Вы находитесь на странице: 1из 126

Head Office : B-32, Shivalik Main Road, Malviya Nagar, New Delhi-110017

• Sales Office : B-48, Shivalik Main Road, Malviya Nagar, New Delhi-110017
Tel. : 011-26691021 / 26691713

Page Layout : Prakash Chandra Sahoo

Typeset by Disha DTP Team

DISHA PUBLICATION
ALL RIGHTS RESERVED

© Copyright Author

No part of this publication may be reproduced in any form without prior permission of the publisher. The author and the
publisher do not take any legal responsibility for any errors or misrepresentations that might have crept in. We have
tried and made our best efforts to provide accurate up-to-date information in this book.

For further information about the books from DISHA,


Log on to www.dishapublication.com or email to info@dishapublication.com
STUDY PACKAGE IN PHYSICS FOR JEE MAIN & ADVANCED
Booklet No. Title Chapter Nos. Page Nos.
Ch 0. Mathematics Used in Physics
Ch 1. Units and Measurements
Units, Measurements &
1 Ch 2. Vectors 1-202
Motion
Ch 3. Motion in a Straight Line
Ch 4. Motion in a Plane
Laws of Motion and Ch 5. Laws of Motion and Equilibrium
2 203-318
Circular Motion Ch 6. Circular Motion
Ch 7. Work, Energy and Power
Work Energy, Power &
3 Ch 8. Collisions and Centre of Mass 319-480
Gravitation
Ch 9. Gravitation
4 Rotational Motion Ch 1. Rotational Mechanics 1-120
Ch 2. Properties of Matter
Properties of Matter &
5 Ch 3. Fluid Mechanics 121-364
SHM
Ch 4. Simple Harmonic Motion
Ch 5. Thermometry, Expansion &
Calorimetry
6 Heat & Thermodynamics Ch 6. Kinetic Theory of Gases 365-570
Ch 7. Laws of Thermodynamics
Ch 8. Heat Transfer
Ch 9. Wave – I
7 Waves 571-698
Ch 10. Wave –II
Ch 0. Mathematics Used in Physics
8 Electrostatics Ch 1. Electrostatics 1-216
Ch 2. Capacitance & Capacitors
Ch 3. DC and DC circuits
9 Current Electricity Ch 4. Thermal and Chemical effects of 217-338
Current"
Ch 5. Magnetic Force on Moving
Charges & Conductor
Ch 6. Magnetic Effects of Current
10 Magnetism, EMI & AC Ch 7. Permanent Magnet & Magnetic 339-618
Properties of Substance
Ch 8. Electromagnetic Induction
Ch 9. AC and EM Waves
Ch 1. Reflection of Light
Ch 2. Refraction and Dispersion
11 Ray & Wave Optics Ch 3. Refraction at Spherical Surface, 1-244
Lenses and Photometry
Ch 4. Wave optics
Ch 5. Electron, Photon, Atoms,
Photoelectric Effect and X-rays
12 Modern Physics 245-384
Ch 6. Nuclear Physics
Ch 7. Electronics & Communication
Contents
Contents
Study Package Booklet 9 - Current Electricity
3. DC and DC Circuits 217-306 4. Thermal & Chemical Effects of Current
307-338

3.1 Introduction 218 4.1 Thermal effect of current :

3.2 Ohm’s law 221 joule’s law 308

3.3 Electricity from chemicals : cell 226 4.2 Electrical appliances 310
4.3 Seebeck effect 315
3.4 Electromotive force (emf) 227
4.4 Peltier effect 317
3.5 Circuit analysis 229
4.5 Thomson effect 318
3.6 Electrical instruments 250 4.6 Chemical effect of current 318
Review of formulae and 4.7 Faraday’s law of electrolysis 319

important points 256 Review of formulae and


important points 320
Exercise 3.1- 3.6 258-286
Exercise 4.1-4.6 322-332
Hints & solutions 287-306
Hints & solutions 333-338
218 ELECTRICITY & MAGNETISM
3.1 INTRODUCTION
When you turn on a light, you are using the sort of electricity that flows along the wires,
like water flows along pipes. This is called current electricity. It is usually made up of
billions of electrons flowing along a wire. These electrons do not move along a wire by
themselves. They have to be pushed by potential difference, produced by the battery.

Inside a wire
An electric current consists of billions of free electrons, flowing along a wire. The
electrons “hop” from one atom to the next travelling in short bursts. Individual
electrons move only fractions of a milimetre each second.
But like pushing a row of railway wagons, they at
the speed of light, 300,000 kilometres
per second.

Electrons Plastic
moving covering
along or insulation
stops electrons
escaping from wire

Each
atom has
a central nucleus
and on area where
electrons orbit

Direct current, DC
In DC, all the electrons moves in the same direction, in all the time that the electricity
flows. This type of current is produced by the batteries in torches cars and similar
devices.
Alternating current, AC
In AC, the direction of electron movement changes many times each second. The electrons
move one way, then the other, and so on.
Electric current
In an isolated metallic conductor, the free electrons move randomly like the molecules of
a gas and so the net rate of flow of charge through any hypothetical plane is zero. If
potential difference is applied across the ends of the conductor, an electric field is set up
at every point within the conductor. This field exerts force on free electrons in opposite
Fig. 3.1 to direction of field and will give them a resultant motion. This flow of electrons constitutes
an electric current. It can be defined as the rate at which charge passes through any
specified surface area. Thus
dq
i = .
dt

SI unit of electric current is C/s. 1 C/s = 1A.


We can get the charge that passes through any cross-section of the wire in time interval
from t1 to t2 by integration. i.e.,
Fig. 3.2 t2
q = i dt .
t1
DC AND DC CIRCUITS 219
More about electric current
1. The current is the same for all cross-sections of a conductor of non-uniform
cross-sections. It is similar to the water flow. Charge flows faster where the
conductor is smaller in cross-section and slower where the conductor has larger
cross-section.
2. As electrons move faster than positive ions, so the net current is due to electrons. Fig. 3.3
This type of current is known as conduction current.
3. In liquids the flow of current occur due to both types of ions. In gases it is due to
positive ions and electrons. In semiconductors, it is due to flow of holes and
electrons.

1C 1
1A = 19 electrons/s
s 1.6 10
Fig. 3.4
18
= 6.25 10 electron/s
4. Current constitutes by an electron: Let an electron is moving on a circular path of
radius r with a speed v. The current due to its motion at any section of path,
q
i =
t

e ev
=
2 r 2 r
v Fig. 3.5
5. If charge q is distributed over the ring uniformly or non-uniformly, current at any
section of ring
q q
i =
t 2 /

q
=
2
(a)
6. Direct current (dc) and alternating current (ac) :

Current density ( J )
It is a characteristic of a point inside a conductor rather than the conductor as a whole.
It is defined as the current per unit area through an infinitesimal area normal to the
direction of current flow. (b)
1. The current density at a point P is given by Fig. 3.6
di
J = nˆ
dA
2. If cross-sectional area is not normal to the current, then
di (a)
J = or i = J dA
dA cos
3. For uniform distribution of current
i
J = .
A (b)
4. It is a vector quantity. Its SI unit is A/m2. Fig. 3.7
220 ELECTRICITY & MAGNETISM
Drift velocity
When an electric field is applied in an conductor, a force acts on each electron in the
direction opposite to the field. The electrons get bised in their random motion in favour
of the force. As a result, the electrons drift slowly in this direction. If the electron drifts
a distance in a long time t, we can define drift velocity (average velocity) as :

Fig. 3.8 Dotted path represents the vd =


t
motion of electron with field E . If is the average time between two successive collisions, then the distance drifted in
this time
1 2 1 Ee 2
= at
2 2 m

eE
Drift velocity vd = at
m
–4
The order of drift velocity is 10 m/s. It is very small in comparison to thermal speed
( 105 m/s ) of electron at room temperature.
Now consider a cylindrical conductor of cross-sectional area A placed in electric field
E . Take a small length vd t of the conductor. The volume of this portion is Avd t. Let
n is the number of free electrons per unit volume of the wire, then total free electrons in
this volume is nAvd t .
The charge crossing this area q = nAvd t e
q
or i = neAvd
t
i
and J = ne vd
Fig. 3.9 A
More about drift velocity
i
We can write vd .
neA
(i) If a constant current flows through a conductor of non-uniform cross-section,
1
then vd
A
i V /R
(ii) Also, vd =
neA neA
V V
=
ne
neA
A
Clearly if conductors of different cross-sectional area are connected with same p.d., the
drift velocity is independent of cross-sectional area.
Microscopic view of Ohm's law E J :
eE
We have got vd =
2m
and J = ne vd

ne 2
From above equations, we get J = 2m E
DC AND DC CIRCUITS 221

2m
or E = J
ne 2
or E = J Ohm's law
2m
where , is called resistivity of the material.
ne 2
1
The conductivity of material is defined as = .

1
The quantity is called conductance.
R

3.2 OHM'S LAW


If the physical conditions of conductor (temperature, length etc.) do not change, then
the current in the conductor is directly proportional to the potential difference across the
conductor.
Resistivity ( )
It can be defined as the ratio of electric field applied to current density.Thus
E
=.
J
Resistivity is the property of material which does not depend on size and shape of the
conductor. SI unit of resistivity is Ohm-meter ( -m). A perfect conductor would have
zero resistivity and a perfect insulator have infinite resistivity. Metals and alloys have
the lowest resistivities and hence they are the best conductors. It is a familiar fact that
good electrical conductors, such as metals, are good conductors of heat, while poor
electrical conductors such as plastics are also poor thermal conductor.
Variation of resistivity with temperature
The resistivity of metallic conductors increases with increasing temperature as

t = 0 [1 t t2 ]

where 0 = resistivity of conductor at 0 C


t = resistivity of conductor at t C (a)
t = is the temperature difference between t and 0 C,
and & are temperature coefficients of resistance
, and positive for metals and negative for non-metals.

Note: The resistivity of the alloy manganin is practically independent of the


temperature.
(b)
Resistance
Fig. 3.10
In the relation E J , it is difficult to measure E and J directly. It is, therefore very
convenient to put this relation in macro parameters like current and potential difference.
To do this let us consider a conductor of length and cross-sectional area A as shown
in fig. 3.11.
We can write E = J
V i
As E = and J
A Fig. 3.11
V i
=
A
222 ELECTRICITY & MAGNETISM

or V = i =iR
A

This relation is often referred to as Ohm's law.

where R is called resistance of the specimen and is equal to R .


A
The SI unit of R is ohm ( ). Its symbol is .
The material obeying Ohm's law is called ohmic conductor or a linear conductor. If ohm's
law is not obeyed, then the conductor is non-ohmic or non linear.
Fig 3.12 (a) is a curve for metalic conductor. The straight line shows that the resistance
of conductor is the same no matter what applied voltage is used to measure it. fig.3.12
(b) is not straight line and the resistance depends on voltage applied, the resistance is
called dynamic resistance and the conductor is called non-ohmic conductor.
Thermistor
Thermistor is the heat sensitive resistor. It is usually made by semiconducting material
for which V – i plot is not linear. The temperature coefficient of thermistor is negative and
Fig. 3.12
usually large. It is of the order of 0.04 / C.
Thermistors are used for making resistance thermometers which can measure very low
temperature.

Note:
1. Resistance depends on length of wire not on the shape of the wire.
All the wires have same resistance between A & B. is the straight length of wire
between its ends.
Fig. 3.13 2. V = i R is the general relation between i and V which is applicable to both type of
conductors, ohmic as well as non-ohmic.
Variation of resistance with temperature
As the resistance of any specimen is proportional to its resistivity, which varies with
temperature, and also resistance varies with temperature. Therefore we have

Rt = R0 (1+ t t2 )
Fig. 3.14
for small t, Rt R0 (1+ t )
Temperature coefficient of resistance ( )
If R1 and R2 are the values of resistance at temperature t1 and t2 respectively, then
R1 = R0 ( 1 t1 ) and R2 = R0 ( 1 t2 )
R1 1 t1
R2 = 1 t2

R2 R1
or = .
R1t2 R2t1

Note:
We have assumed to be constant for all temperature. But actually it varies with temp.
If Rt is the resistance at any temperature, then Rt = R0 ( 1 t ), differentiating above
equation w.r.t. temp, we get
1 dRt
= R0 dt .
DC AND DC CIRCUITS 223
Resistance of conductor of non-uniform cross-section
Consider a conductor of length and radius at its ends are r1 and r2. The resistance of
element under consideration
dx
dR =
r2
r2 r1
where, r = r1 x
Fig. 3.15

dx
dR = 2
r2 r1
r1 x

Resistance of whole conductor

dx
R = dR
2
0 0 r2 r1
r1 x

or R = .
r1r2

Why conductor offers resistance?


Resistance means the hinderance offered to the flow of charge. Electrons in their motion
collide with the positive ions and themselves, due to which resistance in motion occurs.
The resistance mainly occurs due to collisions of electrons with the positive ions.
Super conductors
Kamerlingh found that mercury offers zero resistance at 4.2 K. This phenomenon is
called super conductivity and the metal is called superconductor. Certain alloys become
superconductors at rather high temperature. The resistance of material in the
superconducting state is zero and the currents once established in closed
superconducting circuits persist for weeks, even though there is no battery in the circuit.
Stretching a wire
Consider a wire of length , radius of cross-section r and is of resistance R. It is stretched
to length '. Let its resistance becomes R '. Assuming volume of material remains constant
after stretching or compression, and so
Fig. 3.16 Resistance-temperature graph
r2 = r '2 '
for superconductor

' r2
or =
r '2

'
We have, R = 2
and R '
r 2
r'

' r2
R' = R
r '2

' '
= R
224 ELECTRICITY & MAGNETISM

2
or = '
R' R

r2 r2
Also R' = R
r '2 r '2

4
r
or R' = R
r'

% change in resistance, if there is small change in length


R
100 = 2 100
R
Resistors in series and parallel
Series : The resistors are said to be in series, if they provide a single path between the
points. The current is same in each resistor in series, but p.d. across resistor is proportional
to its resistance. Let us consider two resistances R1 and R2 connected in series. For
series connection
V = V1 + V2
(a) Since i is same in both the resistors,
iR = iR1 + iR2

or R = R1 + R2
(b) The above equation is true for any number of resistors in series. Thus for n-resistors
Fig. 3.17 R = R1 + R2 + ......+Rn
Parallel : The resistors are said to be in parallel between the points, if each provides an
alternative path between the points. The potential difference is the same across each
resistor, but current divide in inverse ratio of their resistances. For parallel connection of
resistors
i = i1 + i2

V V V
=
R R1 R2

1 1 1
or =
R R1 R2

1 1 1 1
Fig. 3.18 For n-resistors in parallel = .....
R R1 R2 Rn
Effective value of
(i) In series : Suppose 1 and 2 are the temperature coefficient of resistance of
Fig. 3.19 the resistors R1 and R2 respectively. Let be their effective value.

=
At 0°C R01 R02 R0 = R01+ R02
At t°C R01 (1 1t ) R02 (1 2t ) R0 (1 t)
Their equivalent resistance at any temperature in series is
R t = R1t + R2t
DC AND DC CIRCUITS 225
or R0 ( 1 t) = R01 ( 1 1t ) + R02 ( 1 2t )
or (R01 + R02) ( 1 t) = (R01 + R02) + ( R01 1 R02 2 )t
or (R01 + R02) + (R01 + R02) t = (R01 + R02) + (R01 1 + R02 )t
2
R01 1 R02 2
After solving, we get =
R01 R02
(ii) In parallel :

At any temperature t C Fig. 3.20


1 1 1
Rt = R1t R2t

1 1 1
or =
R0 1 t R01 1 1t R02 1 2t

1 1 1
or = R01 1
R01 R02 1t R02 1 2t
1 t
R01 R02

R01 R02 1 1
or =
R01 R02 1 t R01 1 1t R02 1 2t

1
R01 R02 1 t 1 1 1 1
= 1 1t 1 2t
R01 R02 R01 R02
Solving by Binomial theorem, we have
1 1 1 1
1 t 1 t = 1 1t 1 2t
R01 R02 R01 R02

1 1 1 2
or t = t
R01 R02 R01 R02

1 R02 2 R01
or = R01 R02

Ex. 1 The region between two concentric spheres of radii r1 and The resistance between the spheres
r2 is filled with a conducting material of conductivity . Find r2 r2
1 dr
resistance between the spheres. R = dR
r1
4 r1 r2
Sol.
Choose an element of thickness dr at a r2 r1
distance r from the centre. Its resistance = Ans.
4 r1r2
dr 1 dr
dR = 2
Ex. 2 Find resistance of the annular disc between centre and
4 r 4 r2
periphery of the disc. The inner and outer radius of the disc is a and
b respectively. The thickness of the disc is t and conductivity of its
Fig. 3.21 material is .
226 ELECTRICITY & MAGNETISM
Sol. dr 1 dr
The resistance of a cross-section perpendicular to the current flow on dR =
2 rt 2 rt
any radius r and thus of area 2 rt is given by Total resistance

b
1 dr
R =
a
2 rt

1 b
or R = n Ans.
2 t a

Fig. 3.22

3.3 ELECTRICITY FROM CHEMICALS : CELL


If we want to flow the charge through a resistor, we must set-up a potential difference
across it. Let us connect each end of the resistor to a conducting sphere, with one
Fig. 3.23 sphere charged positively and other negatively as in fig. 3.23.
The problem with such a device is that charge flows till their potential become equal.
When that happens, the flow of charge stops.Therefore, to maintain a steady flow of
charge we need a device which can maintain potential difference between the terminals
of the resistor. Such a device is called emf device. It may be a battery, electric generator
or solar cells etc.
The simplest unit for making electricity is called a cell. It makes electricity for chemical
Electric animal reactions and works like a pump to push electrons along wires. A battery has two or more
Muscles produce tiny electrical signals
as they work. In the electric eel, these cells and some types, such as car batteries, are rows or "batteries" of single cells, hence
muscles form large blocks along the our common name "battery" for single and multiple cells. In a primary cell, as electricity
body. They produce powerful surges of is produced, the chemicals are slowly used up. Eventually, the chemicals run out and the
electricity, hundreds of volts, like a battery can not make electricity any more. In a secondary cells, the chemicals can be
“living battery”
replenished or reformed by recharging the cell with electricity.

Negative terminal Car batteries


Positive Also called an accumulator, a vehicle batteries can be recharged. The
terminal chemical reaction which has taken place to make electricity can be
Acid-proof reversed by putting electricity back in, so the battery can be used
casing again. In a vehicle the recharging is carried out by an alternator, which
is driven by the engine. Most car batteries have six linked cells, each
with an output of about two volts. Each cell consists of lead plates,
lead dioxide plates and sulphuric acid. Electricity is produced in the
Plates sit in bath
of strong sulphuric acid reactions between the plates and the sulphuric acid.
Lead dioxide plate
Lead plate
DC AND DC CIRCUITS 227
3.4 ELECTROMOTIVE FORCE (EMF)
Emf of a source is defined as the workdone per unit charge that the source does in
moving charges from its lower potential terminal to its high potential terminal. i.e.,
dW
= .
dq Fig. 3.24
The term electromotive force is somewhat confusing in that concept to which it refers as
not a force, but a work per unit charge.
For a source in open circuit, the potential difference V, is equal to the electromotive force.
Thus
V =
Under closed circuit condition, the source offers some resistance to the flow of charge.
This resistance is called internal resistance. Because of this internal resistance the potential
across the terminals of source is decreased by i r.
p.d across the terminals of the cell
V = –ir
Now consider the case in which source is being charged by an other source like alternator.
When this is the case, the current within the source is from positive terminal to negative
terminal. The terminal voltage is then greater than the emf and so p.d. across the
terminals of the source Fig. 3.25
V = + i r.

What happens inside a source of emf ?


1. Consider a source of emf which maintains a potential difference across its terminals.
Let us consider the charges inside the source. In an open circuit, the charges are
in equilibrium, therefore the net force on each charge must be zero. i.e.,
Fig. 3.26
F = 0 = Fe + Fn
where F e is the force exerted by field produced by the terminals of the source.
This field is electrostatic in nature. The other force ( F n ) which is equal in
magnitude to F e is the force exerted by field, is of nonelectrostatic in nature,
whatever its origin is. Thus we can write
Ee + En = 0
Fig. 3.27
or Ee = En

2. The electrostatic potential difference V is defined as the work done per unit
charge done by the electrostatic field E e on a charge moving from positive
terminal to negative terminal of the source. And the work done by E n , per unit
charge, when a charge moves from negative terminal to positive terminal of the
source, is called electromotive force of the source.
When E e E n , we have V =

3. Now suppose that the terminals of a source are connected by a conducting wire
forming a complete circuit. The driving force on the free charges in the wire is due
to the electrostatic field E e field set up by the charged terminals of the source.
This set up a current in the wire from positive terminal to negative terminal of the
source. The charges on the terminals decreases slightly and hence the electrostatic
Fig. 3.28
228 ELECTRICITY & MAGNETISM

fields both within the wire and source decrease also. As a result E e becomes

smaller than E n . Hence positive charges within the source are driven towards
the positive terminals, and there is a current within the source from negative to
positive terminal of the source. As E e is some what less than E n in closed
circuit condition, correspondingly V is less than . The difference is equal to the
work per unit charge done by the resultant field, which is simply ir.
V = – ir
Short circuit : If the terminals of a source are connected by a conductor of zero
resistance (practically negligible resistance), the source is said to be short
circuited.
The short circuit current can be obtained as
V = 0= –ir

Fig. 3.29 or i =
r
Open circuit : When no current is drawn from the source, it is said to be an open
circuit. Thus
i = 0
V = – ir = – 0 × r
Fig. 3.30 or V =
At a glance
1. For an ideal source of emf (cell) whose internal resistance (r) is zero, VAB =
or
2. A practical cell which has some internal resistance (r).
Fig. 3.31 VAB =
3. When cell is discharging :
VAB = ir
Fig. 3.32 4. When cell is charging :
VAB = ir
5. Short circuit : VAB = 0
Fig. 3.33 or 0 = – ir i
r
6. Open circuit : i = 0
VAB = – ir = – 0 =

Fig. 3.34
Difference between a cell and a capacitor
1. Capacitor can provide variable (decreasing) potential in a circuit for short time.
While cell can provide constant potential for a long time (many years).
Fig. 3.35 2. The energy stored between plates of capacitor is electrical while energy stored
between terminals of cell is chemical.
3. The plates of capacitors are usually made of same material. The terminals of cell
always be of different materials.
4. The electric field between the plates of capacitor is non zero. But electric field
between the terminals of cell is zero in open circuit.
Fig. 3.36
DC AND DC CIRCUITS 229
3.5 CIRCUIT ANALYSIS
Ohm's law can give the solution of the circuit which has one loop or circuit with many
loops which can be reduced to a single loop. To analyse complicated circuits Kirchhoff
developed a method which is based on two laws. These are :
Kirchhoff's first law or junction rule
According to it, the algebraic sum of currents at any junction is equal to zero. i.e.,
i 0.
or
The sum of currents entering at any junction is equal to the sum of currents leaving the
junction.

Fig. 3.37
Kirchhoff's first law is based on conservation of charge; the magnitudes of the currents
in the branches must add to give the magnitude of the current in the original conductor,
so that i1 = i2 + i3.
Kirchhoff's second law or loop rule
According to it the algebraic sum of potential drop across all circuit elements in a closed
loop is equal to zero. i.e., V 0 .
Second law is based on conservation of energy.
(a) For a circuit having cells and resistors, we can wirte
iR = 0.
(b) For a circuit having cells, capacitors and resistors, we have
q
iR = 0.
C
Guidelines and sign conventions
The guidelines and sign conventions, we have used in previous chapter can also be
used here also.
1. Assume the currents in each branch of the circuit. At each junction, they must
satisfy junction rule. The current in any resistor is due to the net response of all
the sources present in the circuit.
2. You may choose any close loop, which may have cell or not. But for a circuit Fig. 3.38
having only one cell, choose a close loop, which include this cell.
3. (a) The potential drop across any resistor in the direction of flow of current can
be taken as negative and positive for reverse direction of current.
(b) The potential drop across the capacitor is taken as negative from its positive
plate to its negative plate and positive for reverse sense.

Fig. 3.39
Note: See the sign of charge of first coming plate of the capacitor. In the above
case, the first coming plate when moves from left to right across the capaci-
tor is positive which has charge (q).
(c) The emf of the cell is taken as negative from positive terminal to negative
terminal of the cell and positive for reverse sence.
Fig. 3.40
230 ELECTRICITY & MAGNETISM

Note: In the final calculations, some of the currents/all currents may be of


negative sign. Do'nt be bother about their sign. The negative sign indicate that
the actual direction of current will be opposite of assumed direction of current. In
further calculations you have to use the value of current with the negative sign.
Grouping of cells : Battery
1. Series grouping :
Fig. 3.41
(a) Right grouping of cells : When each cell in series delivers the current in same
direction.

The equivalent emf eq = 1 2

The equivalent resistance req = r1 + r2


(b) Wrong grouping of cells : When cells deliver current in opposite directions.

The equivalent emf eq = 1 2 ( 1 2)


Fig. 3.42
The equivalent resistance req = r1 + r2
In a series of 5 identical cells, if one of the cells is wrongly connected :
The net emf of rightly connected cells = 4
The net emf of wrongly connected cell = –
The equivalent emf of all cells in series = 4 –
Fig. 3.43 or eq = 3
The equivalent internal resistance req = 5r
Thus when one cell is wrongly connected, the net or equivalent emf is
decreased by 2 . But internal resistance is always be additive.
For a battery of n-identical cells, if m cells are wrongly connected then effective
emf
eq = (n – 2m)
req = n r
(c) For a battery of n-identical cells :
Net or equivalent emf eq = n
and equivalent resistance Req = n r + R

eq
Net current i = Req
Fig. 3.44

n
=
nr + R

(i) If R << n r, then i . This is the current in the resistor due to only one cell.
r
Therefore series grouping of cells in such a case when external resistance is
very small in comparison to internal resistance is of no use.

(ii) If R>> nr, imax n . This time the current in resistor R is n times the current
R
due to one cell. Therefore series grouping of cells is useful in this case.
DC AND DC CIRCUITS 231
2. Parallel grouping
(a) For a battery of n-identical cells, all are rightly connected
Equivalent emf eq =
r
and equivalent resistanceReq = R
n
eq
i = Re q

= r
R
n
r
(i) If R >> ,i . This is the current in the resistor due to only one cell.
n R
Therefore such grouping of cells is of no use.
r
(ii) If R , imax n . That time the current in resistor R is n times the current
n r
Fig. 3.45
due to one cell. In such a case parallel grouping of cells is useful.
(b) Battery having cells of different emf
(i) Two cells are rightly connected : Both the cells delivering currents in same
sence.
Method I :
In closed loop ACDBHGA
1 i1 r1 iR = 0
1 R
or i1 = –i
r1 r1

2 R
Similarly i2 = i
r2 r2

1 2 1 1
and i1+ i2 = – iR
r1 r2 r1 r2

1 2 1 1
or i = iR
r1 r2 r1 r2
Fig. 3.46
1 2
r1 r2
or i =
1 1
1 R
r1 r2

1r2 2 r1
r1r2
=
r1r2 R r1 r2
r1r2

1r2 2 r1
=
r1r2
r1 r2 R
r1 r2

1r2 2 r1 / r1 r2 eq
=
r1r2 Re q
R
r1 r2
232 ELECTRICITY & MAGNETISM

= 1r2 2 r1
eq
r1 r2
Method II :
(i) In closed loop ACDBFEA
1 ir1 i r2 2 = 0

1 2
or i = r1 r2

Fig. 3.47 Now VAB = 1 i r1

1 2
= 1 r1
r1 r2

or = VAB = 1r2 2 r1 r1r2


eq , and req
r1 r2 r1 r2
(ii) If cells are wrongly connected : The equivalent emf of them can be easily
obtained by any of the methods discussed above. Or otherwise putting – 2 is
place of 2, so we have
1r2 2 r1
eq =
r1 r2
Fig. 3.48
r1r2
req = .
r1 r2
3. Mixed Grouping :
Let there are n-identical cells in a row and m such rows are in parallel. The total
number of cells in the circuit is mn.
Equivalent emf eq = n
nr
and equivalent resistance Req = R
m
eq
Net current i =
Req

n
=
nr
R
m
mn
=
nr mR
mn
= 2
nr mR 2 nr mR
The current i will be maximum, when denominator in above expression is minimum.
It to be minimum when
nr mR = 0
nr
or = R
m
or total internal resistance = total external resistance
mn mn
then i ma x = .
2 nr mR 2 rR
Fig. 3.49
DC AND DC CIRCUITS 233
Ex. 3 Find the equivalent emf of the three cells which are Sol.
connected as shown in fig. 3.50 The cells can be connected in the following manner

Fig. 3.50 Fig. 3.54


Sol.
1 1 2 1 3 1
(a) The equivalent emf of cells 1 and 2 is 1 = V , r1
1 1 2 2
1r2 2 r1
= r1 r2 4 1 3 1 1 1
2 = V , r2
1 1 2 2

1r2 2 r1
Now = r1 r2
Fig. 3.51
eq= VAB = 3+
3 1 1 1
1r2 2 r1
= 3+ 2 2 2 2
r1 r2 = 1 1
2 2
r1 r2
and Req = r3
r1 r2 = 1V Ans.
(b) The equivalent emf of cells 1 and 2 is Ex. 5 Two sources of current of equal emf are connected in
1r2 2 r1 series and have different internal resistances r1 and r2 (r2 > r1).
r1 r2 Find the external resistance R at which the potential difference
across the terminals of one of the sources (which one in particular)
2r1 1r2 become equal to zero.
r1 r2
Sol.
The current in the circuit is

2
i =
Fig. 3.52 r1 r2 R
eq = V AB = 3+

2 r1 1r2
= 3
r1 r2

r1 r2
and Req = r3 .
r1 r2
Fig. 3.55
Ex. 4 Find equivalent emf of four different cells are connected
Since i is same in both the cells
in parallel. Their emfs and internal resistances are shown in fig.
V1 = –ir1 and V2 = –ir2
3.53.
Since both the cells are giving the currents therefore V1 and V2 can not be
negative.
Since r2 > r1, therefore firstly V2 can be zero

2
0 = r2
r1 r2 R

Fig. 3.53 or R = r2 r1 . Ans.


234 ELECTRICITY & MAGNETISM
The method of principle of superposition Finding the potential difference :
Complicated circuit can be solved by using the principle of
superposition. According to this principle, when a number of cells
Ex. 7 Consider the following circuit and find potential difference

are present in a circuit, the current in any resistor is same as the VA – VB.
superposition of the currents due to all the cells, acting one at a
time, the others being absent.
Ex. 6 Analyse the following electrical circuit.

Fig. 3.59

Fig. 3.56 Sol.


Sol. The currents in different resistors can be obtained as follows: The net
current drawn from the source
Method I :
Assuming first only 10V is present in the circuit, and then only 20V is 10
present in the circuit. Find current in each case. Finally superpose the
i = 2A.
6 6
currents due to both the cells. It can be understand from the fig. 3.55. 1 1
6 6

This current will divide equally in branches OAP and OBP because they
have equal resistances.

Fig. 3.60
Method I : VA–VB = (VO–VB) – (VO–VA)
= 4 1 2 1 2V Ans.
Fig. 3.57 Method II :
Method II : Connect a hypothetical cell between A and B, taking A as positive terminal.
Now apply loop rule in close loop OABO, to get VAB = VA–VB. Thus
By using Kirchhoff's method.
2 1 VAB 4 1= 0
or VAB = 2V Ans.

Ex. 8 See the circuit shown in fig. 3.61. Find the current through
the switch S when it is closed.

Fig. 3.58
Assuming the currents in resistors as shown in fig. 3.59. In closed loop
ABEFA, we have
–2 i1–2(i1+i2)+10 = 0
or 2 i1 + i2 = 5 ....(i)
In closed loop BCDEB, we have
–20 + 2i2 + 2(i1+i2) = 0 Fig. 3.61
or i1 + 2i2 = 10 ...(ii) Sol.
Solving equations (i) and (ii), we get Let i1 and i2 be the currents in the branches A O and B O respectively. By
i1 = 0, and i2 = 5A junction rule the current in switch S is
i = i1+ i2
DC AND DC CIRCUITS 235
VO 0 20 VO 10 VO i1 = i1 + i3
or = which gives i3 = 0
2 2 4
or VO = 10 V (2 resistor is in open circuit, therefore current in it will be zero)
VO 0 To get the potential difference VBA = VB – VA, connect a hypothetical cell
Current, i =
2 between A and B keeping B as positive terminal.
10 0 Now in close loop A A' B' B D E F A, we have
= 5A . Ans.
2 + VBA + 5 + 5 i1 = 0
Ex. 9 Find out the value of current through resistance for
or VBA = –5 – 5 i1
the given circuit. Also find the potential difference between A and = 5 5 2
B. = –15 V. Ans.
Ex. 10 Find the reading of the ammeter A as shown in fig. 3.64.

Fig. 3.62
Sol.
Let i1 and i2 be the currents in 5 and 10 resistors respectively..
Fig. 3.64
Sol.
The above circuit can be reduced as follows.

Fig. 3.63
In close loop ACEFA,
–5i1 + 10 = 0 Fig. 3.65
or i1 = 2A The reading of ammeter A is i, where
In close loop GHBDG
12
–20 + 10i2 = 0 i = 3A
or i2 = 2A 3 1
Let i3 is the current in 2 resistor. At junction E, The current distribution in all the resistors is also shown in the fig. 3.65.

More about short


When two points of a circuit are connected together by a resistanceless wire, they are
said to be short-circuited. Since 'short' has theoretically zero resistance, so it gives rise
to two important facts :
(i) no voltage can exist across it because V = ir = i 0 0 ,
(ii) current through it is very large, theoretically infinity.

Short in series circuit


Consider a circuit with three resistors in series with a source of emf as shown in fig. 3.66 (a)
The equivalent resistance of the circuit R = 1 + 2 + 3 = 6 and resultant current
12
i= 2 A which is same in all the resistors.
6
Now 3 resistor has been shorted out by a wire CD as shown in fig. 3.66 (b) so that
RCD = 0. Fig. 3.66
236 ELECTRICITY & MAGNETISM
The equivalent resistance of circuit R ' = 1 + 2 = 3 and current in the resistors
12
i'= 4 A in 1 and 2 resistors, but no current in 3 resistor..
3
Short in parallel circuit
Consider a circuit as shown in fig. 3.67 (a).
The equivalent resistance of circuit
1 1 1 1
=
R 1 2 3
6
which gives R =
11
12
currents in resistors are i1 = 12 A
1
12
i2 = 6A
2
12
i3 = 4A
3
Fig. 3.67 Now suppose a short is placed across 3 resistor as shwon in fig. 3.67 (b). The entire
current will pass through the short.
R AB = 0
i1 = i2 = i3 = 0.
But short draws infinite current.

Ex. 11 In the circuit shown in fig. 3.68 E, F, G and H are cells Sol.
of emf 2, 1, 3 and 1 V respectively. The resistances 2, 1, 3 and 1 The proposed currents are shown in fig. 3.68 (b)
are their respective internal resistances. Calculate (a) the potential In close loop ABDA, we have
difference between B and D and (b) the potential differences across –2 + 2i + 2i1 + 1i + 1 = 0
the terminals of each of the cells G and H. or 3i + 2i1 = 1 ...(i)
In close loop BCDB, we have
1 + 1(i – i1) + 3 (i – i1) – 3 – 2 i1 = 0
or 2i – 3i1 = 1 ...(ii)
After solving equations (i) and (ii), we get
1
i1 = A
13
5
and i = A
13
(a) The potential difference
VD – VB = 2i1
1 2
= 2 V
13 13

2
or VB – VD = V
13
(b) The current in the cell is
5 1 6
i–i1 = A
13 13 13
Since cell G is discharging and H is charging
6
VG = ir 3 3
13

Fig. 3.68 21
= V Ans.
13
DC AND DC CIRCUITS 237

6 Ex. 13 Twelve cells each having the same emf are connected
and VH = ir 1 1
13 in series and are kept in a closed box. Some of the cells are wrongly
connected. This battery is connected in series with an ammeter
19 and two cells identical with each others. The current is 3A when
= V. Ans.
13 the cells and battery aid each other and 2A when the cells and
Ex. 12 A part of a circuit in steady state along with the currents battery oppose each other. How many cells are wrongly connected ?
flowing in the branches, the values of resistances etc. is shown in Sol.
fig. 3.69. Calculate the energy stored in the capacitor. Let m cells are wrongly connected in the battery and is the emf of each
cell, then
net = (12 – 2m)
When two cells aid the battery, then current

12 2m 2
3 = ...(i)
R
where R is the total resistance of the circuit.
When two cells opposes the battery, then

12 2m 2
2 = ...(ii)
R
Solving above equations, we get
m = 1
Hence one cell is wrongly connected in the battery.

Ex. 14 Find the charges on the four capacitors of capacitances


Fig. 3.69 1 F, 2 F, 3 F and 4 F shown in fig. 3.71.
Sol.
After charging fully there is no current in arm of capacitor. By using
junction rule the currents in the resistors are shown in fig. 3.69.

Fig. 3.71
Sol.
The current will flow through the resistors only. Let i1 and i2 be
the currents in branches AB and CD respectively, then
6
i1 = 2A
1 2
Fig. 3.70 6
The p.d. across the capacitor is and i2 = 1A
3 3
VP–VQ = (VP – VR) + (VR – VS) + (VS – VQ)
(i) p.d. across 1 F capacitor
= 5 3 1 3 2 1
VAE = 1 2 = 2V
= 20 V
charge on the capacitor = 1 VAE = 2 C
The energy stored in the capacitor is
(ii) p.d. across 2 F capacitor
1
U = CV 2 V EB = 2 2 = 4V
2 charge on the capacitor = 2 VEB = 8 C
1 (iii) p.d. across 3 F capacitor
6
= 4 10 202 VCF = 3 1=3V
2
charge on the capacitor = 3 VCF = 9 C
= 800 J. Ans.
(iv) p.d. across 4 F capacitor
V FD = 4 1=4V
charge on the capacitor = 4 VFD =16 C. Ans.
238 ELECTRICITY & MAGNETISM
Ex. 15 A long round conductor of cross-sectional area S is
1 a a
1 2
made of material whose resistivity depends only on a distance r = rdr
R dR
0 0

from the axis of the conductor , where is a constant. Find (where a is the radius of conductor)
r2
(a) the resistance per unit length of such a conductor 2
(b) the electric field strength in the conductor due to which a
a
2 rdr 2
a
2 a4 a2
3
= r dr
current i flows through it. 0 0
4 2
Sol. r2
(a) Consider a cylindrical element of radii between r and (r + dr). Its
resistance S2
=
2
dR =
2 rdr
2
Thus R = . … (ii)
1 2 rdr S2
or = ...(i)
dR
2
The resistance per unit length of wire R =
S2

2
(b) Equation (ii) can be written as R . Compare with
S S

2
R , we get .
S S

Fig. 3.72
i 2 2 i
By Ohm's law E j .
S S 2
Since all such resistance are in parallel S

Wheatstone bridge
It is a special type of resistance network, commonly used for comparing resistances or
unknown resistance can be measured in terms of three known resistances. It consists of
four resistances connected in the form of a bridge.
Analysis
Let four resistors P, Q, R and S are connected as in fig. 3.74.
The proposed currents are as shown in fig. 3.74.
At junction A i = i1 + i2 ...(i)
In close loop A B D A;
–P i1– G ig + R i2 = 0 ...(ii)
and in close loop B C D B;
– Q (i1–ig) + S (i2 + ig) + G ig = 0 ...(iii)
Fig. 3.73 In close loop A B C E F A;
–Pi1 – Q(i1 – ig) + V = 0 ...(iv)
After solving above equations we can get i1, i2 and ig. The equivalent resistance between
A and C is
V V
RAC = i i1 i2

On substituting the values of i1 and i2, we can get RAC.


Balanced Wheatstone bridge
For balanced bridge; VB = VD, and so no current flows through the galvanometer (ig = 0),
the equations (ii) and (iii) reduce to
– P i1 + R i 2 = 0
DC AND DC CIRCUITS 239

i1 R
or i2 = ...(v)
P
and – Q i1 + S i2 = 0
i1 S
or i2 = ...(vi)
Q
Now from equations (v) and (vi), we have

R S
=
P Q
Fig. 3.74
P R
or =
Q S
The balanced bridge now effectively reduces as :
The equivalent resistance between A and C
1 1 1
RAC =
P Q R S

P R
The relation = can be written as
Q S

P Q
= .
R S
That is if positions of the galvanometer and battery are interchanged, then there will still
no deflection in the galvanometer. Fig. 3.75

Ex. 16 Find the equivalent resistance between the points A We have VA – VB = (VA – VC) + (VC – VB)
and B of the circuit shown in fig. 3.76. = 5 i1 + 10(i – i1) = 10 i – 5 i1 ...(i)
Also VA – VB = (VA – VC) + (VC – VD) + (VD – VB)
= 5 i1 + 5(2 i1 – i) + 5 i1
= 20 i1 – 5i ...(ii)
From equations (i) and (ii), we get
3
i1 = i
5
Substituting this value in equation (i), we get
3i
VA – VB = 10 i – 5
5
= 7i
V A VB
or = 7
Fig. 3.76 i
Sol. or RAB = 7.
Method I : Method II :
The given circuit is not balanced bridge therefore we have to find the
equivalent resistance either by circuit analysis by Kirchoff's law or by
other method.
Suppose current i enters through the junction A into the circuit. Let i1 goes
to the 5 resistor and the rest i – i1 goes to 10 resistor. By symmetry,,
the current i, coming out from the junction B will be composed of a part i1
from 5 resistor and i–i1, from 10 resistor. By junction rule the
current in middle 5 resistor is (2i1 – i) as shown in fig. 3.76 (b).
Fig. 3.77
240 ELECTRICITY & MAGNETISM
Supposing a source is connected between the terminals A and B. The Solving equations (iv) and (v), we get
current distribution is shown in fig. 3.77.
9V
At junction A, i = i1 + i2 i1 =
105
V V
Resistance between A and B, RAB = 6V
i i1 i2 i2 =
105
In close loop A C D A,
– 5 i1 – 5 i3 + 10 i2 = 0 ...(i) V
Also i3 =
35
or – i1 – i3 + 2 i2 = 0
In close loop C B D C,
V V
– 10 (i1 – i3) + 5 (i2 + i3) + 5 i3 = 0 RAB =
i1 i2 9V 6V
or – 2 i1 + i2 + 4 i3 = 0 ...(ii) 105 105
Now in close loop A C B E F A
– 5 i1 – 10 (i1 – i3) + V = 0 = 7 .

V
or – 3 i1 + 2 i3 = ...(iii)
5 Note:
From equations (i) and (iii), we get
6V 9V
V i1 i3 i2and i2 i3 i1
– 5 i1 + 4 i2 = ...(iv) 105 105
5
Thus the current in 5 resistor connected symmetrically is same and
From equations (ii) and (iii), we get
currents in 10 resistors also remains same.
2V
4 i1 + i2 = ...(v)
5

Delta-star transformation
In solving networks, having many loops by the applications of Kirchhoff's method, one
sometimes experiences great difficulty due to large number of simultaneous equations
that have to be solved. However, such complicated networks can be simplified by
successively replacing delta meshes by equivalent star system and vice versa. Suppose
we have three resistances R1, R2 and R3 connected in the form of delta as shown in fig.
3.78
(a) These three resistances can be replaced by three resistances R12, R23 and R13 in
the form of delta between the same terminals as shown in fig. 3.78 (b).

Fig. 3.78
R1 R2
where R 12 = .
R1 R2 R3

R1 R3
R 13 = R1 R2 R3

R2 R3
Fig. 3.79 and R 23 = .
R1 R2 R3

Note: In any network there may be many deltas. Students are advised to transform
the delta into star which has symmetry. For example in a network of three deltas,
transform middle delta into star.
DC AND DC CIRCUITS 241
Ex. 17 Take the previous problem and find its equivalent Sol. Method I : By junction removal method
resistance between the terminals A and B by using delta-star
transformation method.

Fig. 3.80
Sol.
We can simplify the circuit by transforming delta A C D into star as
follows.

Fig. 3.84
Because of symmetry about dotted line PQ, the currents in left part of
circuit are same as the right part of the circuit. The current distribution in
the resistors may be as in fig. 3.84 (b). It is very clear from the figure that
resistors having current i3 can be removed from the junction Q without
affecting the currents in other resistors as shown in fig. 3.84 (b). The
resulting circuit can be simplified as :
Fig. 3.81

Fig. 3.82
R1R2 5 10 5
R12 = ,
R1 R2 R3 5 10 5 2

R1R3 5 5 5
R13 = ,
R1 R2 R3 5 10 5 4

R2 R3 10 5 5
R23 = ,
R1 R2 R3 5 10 5 2
Fig. 3.85
5 9 8
RAB = 7 2
2 2 3 8
RAB = 8
7
Ex. 18 Find the equivalent resistance of the circuit shown in 3
2
fig. 3.83 between the points A and B. Each resistor has a resistance Method II :
1 . Since network of resistors is symmetrical about dotted line PQ, therefore
it has equal resistances about PQ. We can break it into two equal parts as:

Fig. 3.83 Fig. 3.86


242 ELECTRICITY & MAGNETISM
The resistance of each half =R, which is equal to

8
R AB
7

4
R Fig. 3.87 Fig. 3.90
7
Ex. 19 Find resistance between A and B in the following
The equivalent resistance between A and B networks of resistances. Each resistance is equal to r.
Sol.
Solution in brief are:
Fig. 3.88 1.

RAB = 2R

4 8
= 2
7 7
Method III : By delta-star transformation :

Fig. 3.91
r
Ans :
2

2.

Fig. 3.92
4r
Ans :
5

Fig. 3.89
3.

Fig. 3.93
Ans : RAB = 4.1
4. RAB = ? . Each resistance is r

Fig. 3.94
2r
Ans. .
3
DC AND DC CIRCUITS 243
5. Ex. 20 Find the equivalent resistance of the given network of
resistors between the points A and B.

Fig. 3.95
10r
Ans. .
9 Fig. 3.100
6.
The circuit now can be simplified by delta-star transformation.
Balanced Wheatstone bridge : Sol.
Junctions D and F are equipotential and so they can be connected together
without affecting the circuit. The simplified circuit is shown in fig. 3.101.
(i)

(ii)

Fig. 3.101
5
r
Ans. RAB =
11
Ex. 21 There is an infinite wire grid with square cells as
(iii) Ans : r shown in fig. 3.102. The resistance of each side is r. Find the
equivalent resista nce of the whole grid between any two
neighbouring points such as A and B.

Fig. 3.96
Incircle resistor can be neglected.

r
7. Ans :
4

Fig. 3.97

Fig. 3.102
Sol.
Let x is the resistance between A and B. The circuit now can be reduced as
shown in fig. 3.103 (c) and (d). The resistance between A and B is also r,
4r
8. Ans : so
3

Fig. 3.98

9.

Fig. 3.99 Ans. R 3


r Fig. 3.103
5
244 ELECTRICITY & MAGNETISM

1 1 1
= R = r + 0.48 r + 0.48 r = 1.96 r
x rx r
3 R 1.96 r
r x RAF =
2 2
r = 0.98 r Ans.
After solving, we get x .
2
Ex. 22 A five pointed regular star has been soldered together
form of a uniform wire. The resistance of the section CL is r;
(a) Find the resistance of the section FL.
(b) What is the equivalent resistance across the terminals A and Fig. 3.106
F? Ex. 23 Twelve equal wires, each of resistance r ohm are
connected so as to form a skeleton cube. Find the equivalent
resistance between the diagonally opposite points 1 and 7.

(a) (b)
Fig. 3.107
Sol.
Fig. 3.104 Connect a source between points 1 and 7.
Sol. The network is symmetrical about the diagonal 1-7. Therefore current in
resistors are distributed symmetrically about the diagonal. The current
Suppose length of A L C =
The length of the sectionFL = 2 ML distribution is shown in fig. 3.107 (b).
= 2 LC sin 18° Choose a close loop 1-2-3-7-9-10-1, we have
= 2 sin 18° i i i
= 0.62 r r r V = 0
3 6 3
(a) Since resistance of wire is proportional to its length, therefore
resistance of section FL, r' = 0.62 r. V 5
(b) The given network of resistors is symmetrical about AF, so it can or = r
i 6
be break into two identical parts, each with a resistance R. HK has
no current because V H = V K. Therefore its resistance can be V 5
neglected. The resistance of left part R can be obtained as : or R17 = r Ans.
i 6
Ex. 24 Twelve equal wires each of resistance r are joined to
form a skeleton cube. Find the equivalent resistance between two
corners on the same edge of the cube.

Fig. 3.105

Fig. 3.108
DC AND DC CIRCUITS 245
Sol. Method I : potential. So must be 4 and 5. If points of equal potential are joint by a
Connect a source between points 1 and 2. Let current i enters through wire, the currents in the circuit do not change. The given network of
point 1 into the network. The network is symmetrical about dotted line. resistors can be reduced successively as shown in fig. 3111 .
The currents above and below dotted line are symmetrically distributed
as shown in fig. 3.109.

Fig. 3.109
By junction rule at 1, we have
i = x + 2y
V V
R12 = = ...(i)
i x 2y
In close loop 1–2–9–10–1, we have
V
–rx + V = 0 or x ...(ii)
r
In close loop 1–4–3–2–1
– ry – rz –ry + rx = 0
or x – 2y – z = 0 ...(iii)
In close loop 4-8-7-3-4
– r (y – z) – r 2 (y – z) – r(y – z) + rz = 0
or – 4 (y – z) + z = 0
or – 4y + 5z = 0 ...(iv)
From equations (iii) and (iv), we get
– 4y + 5 (x – 2y) = 0 Fig. 3.109
or 5 x = 14 y
V 7r
Since x = r
5 7
r R 12 = r Ans.
7r 12
5 V r
y = 5
14 r
V V Ex. 25 Twelve wires, each having resistance r, are joined to
Now R12 = V 5V form a cube as shown in fig. 3.112. Find the equivalent resistance
x 2y 2
r 14r between the ends of a face diagonal such as 1 and 8.
7 Sol.
= r Ans.
12
Method II :

Fig. 3.112
V5 = V6
Fig. 3.110
Our previous knowledge reveals that points 3 and 6 must be at the same V3 = V4
246 ELECTRICITY & MAGNETISM
Sol.
(a) In open circuit, there is no current in the resistors. Therefore,
VA = 18 V and VB = 0
VA – VB = 18 – 0 = 18 V Ans.
(b) It is clear from (a), that A is at higher potential.

3r
R18 Ans.
4
Infinite ladder : Find the equivalent resistance of the (c)
infinite network as shown in fig. 3.113.

Fig. 3.116
Fig. 3.113 When switch is closed, the current starts flowing in the resistors.
Let equivalent resistance between the points is R. The equivalent between The circuit can be realised as in fig. 3.116
the points remain R if one more cell is connected with the network. i.e.,
18
i = 2A
6 3
We have VO – VP= 6 2 = 12 V
and VB – VP = 3 2 = 6 V
Fig. 3.114 Since VP = 0, VB = 6V Ans.
rR (d) With the switch S open each capacitor is charged with potential 18
R = r r V. Therefore charge on them :
r R
Charge on 3 F capacitor = 3 18 = 54 C
rR Charge on 6 F capacitor = 6 18 = 108 C
= 2r
r R When switch S is closed, the p.d. across 3 F capacitor
or Rr + R2 = 2 r2 + 2Rr + Rr = 3 2 = 6V
or R2 – 2 Rr – 2r2 = 0
and charge stored = 3 6 = 18 C
2 The p.d. across 6 F capacitor
2r 2r 4 2r 2
R = = 6 2 = 12V
2
and charge stored = 6 12 = 72 C
= r 3r Now change in charge on 3 F capacitor
Retain only positive sign, because R always positive. = 18 – 54
3 1 r.. = – 36 C
R = Ans.
and change in charge on 6 F capacitor
Ex. 26 (a) What is the potential difference between points = 72 – 108
A and B in fig. 3.115 when switch S is open ? = – 36 F
(b) Which point, A or B, is at the higher potential ? Ex. 27 A dc source with internal resistance R0 is loaded with
(c) What is the final potential of point B when switch S is closed ?
three identical resistances R interconnected as shown in fig. 3.117.
(d) How much does the charge on each capacitor change when S At what value of R will the thermal power generated in the circuit
is closed? be the highest?

Fig. 3.117
Sol.
The given arrangement of the resistors is equivalent to the three resistors
in parallel (fig. b). Thus equivalent resistance between the terminals
R
RAB =
Fig. 3.115 3
DC AND DC CIRCUITS 247
For the maximum thermal power, the net external resistance of the circuit
is equal to the internal resistance. Thus
R
= R0
3
or R = 3 R0 Ans.
Ex. 28 Find the equivalent resistance across the terminals A
and B of a tetrahedron ADCB shown in fig. 3.118. The resistances
of all the edges of the tetrahedron are equal.

Fig. 3.121
Sol.
From symmetry considerations the potentials of points C and D are
equal. Therefore, this circuit can be replaced by joining the junctions C
and D.
The equivalent resistance between C and O,
R 3R
2 2 3R
RCO =
R 3R 8
Fig. 3.118
2 2
Sol.
From symmetry considerations it can be easily understand that there is R 3R 7R
Resistance of arm ACO : = RAC + RCO =
no current in the edge CD, and so is can be ignored for the equivalent 2 8 8
resistance. Now resistance between A and O
The resistances AD and DB are in series and so equal to 2R. Similarly 7R
resistances AC and CB are in series and so equal to 2R. Now 2R, 2R and R
8 7R
R are in parallel across the terminals A and B. Thus RAO = 7R 15
R
8

1 V V 15V
1 1 1 Thus the current i = Ans.
R AB = 2 R 2R R
RAO 7R 7R
15

or RAB =
R
Ans. Ex. 30 Determine the resistance RAB between points A and B
2 of the frame formed by nine identical wires of resistance R each
(see fig. 3.122).

Fig. 3.119
Ex. 29 Determine the resistance RAO between points A and O
of the frame formed by eight identical wires of resistance R each
(see fig. 3.120).

Fig. 3.122
Sol.
This circuit can not be simplified by connecting or disconnecting junction
or can not be reduced to series or parallel connection. So it require circuit
analysis by Kirchoff's law. Let us apply a p.d. V across the terminals A
and B. The current in different resistors are shown in fig. 3.123

Fig. 3.120

Fig. 3.123
248 ELECTRICITY & MAGNETISM
By junction rule, As resistance can not be negative, so
i2 + i3 = i4 + i5 ... (i)
Now use Kirchhoff's second law in closed loops ACEA, CDEC and A C
2
D B G H A, we have kR1 kR2 R2 R2 kR1 kR2 4kR1R2
– i1R – i3R + i2R = 0 ...(ii) R AB
– i5R + i4R + i3R = 0 ...(iii) 2k
and – i1R – i5R – i2R + V = 0 ...(iv) Ans.
Solving above equations, we get Ex. 32 An electric circuit is shown in fig. 3.126. The cells are
6i1 i1 3i1 4i1 of negligible internal resistances. Find
i2 = , i3 , i4 , i5
5 5 5 5
(a) the current in 3 resistance and the cell of 8 volt
V (b) the charge on the capacitor.
and i1 =
3R
V V
RAB = 6i1
i1 i2 i1
5

5 V 5V
=
11 i1 V
11
3R
15 R
= Ans.
11
Ex. 31 The circuit diagram shown in fig. 3.124 consists of a
very large (infinite) number of elements. The resistances of the Fig. 3.126
resistors in each subsequent element differ by a factor k from the
resistances of the resistors in the previous elements. Determine Sol.
the resistance RAB between points A and B if the resistances of the
At steady state there is no current in the branch of capacitor. The currents
first element are R1 and R2.
in the different branches of the circuit are shown in fig. 3.126. At junction
H
i = i1 + i2 ...(i)
Applying Kirchhoff's loop rule in close loop A B C H A
+ 4 + 3i2 – 5i1 = 0
or 5i1 – 3i2 = 4 ...(ii)
Fig. 3.124
Sol. In close loop H C D G H
From symmetry considerations, we can remove the first element from – 3i2 – 10i – 6i + 8 = 0
the circuit; the resistance of the remaining circuit between points C and D or – 3i2–16 (i1+i2) + 8 = 0
will be RCD = k RAB. Therefore, the equivalent circuit of the infinite chain
will have the form shown in fig. 3.125. or 16i1 + 19i2 = 8 ...(iii)
Solving equations (ii) and (iii), we get
i2 = 0.51 A
and i1 = 1.11 A
The current in 8V cell
i = i1 + i2
Fig. 3.125 = 1.11 + 0.51 = 1.61 A
R2 kR AB To get the p.d. across the capacitor, join point J and D by a hypothetical
Thus RAB = R1 cell VJE, which obviously be the p.d. across the capacitor. In close loop
R2 (kRAB )
G D J F G, we have
R AB R2 kR AB = R1 R2 kRAB kR2 RAB – 8 + 6i + VJE – 3 = 0
2 or –8+6 1.61 + VJE – 3 = 0
R2 RAB kRAB = R1 R2 kR1 RAB kR2 RAB
or VJE = 1.33 V
2
or kRAB R2 kR1 kR2 R AB R1R2 0 Thus the charge on the capacitor
2 Q = CV
R2 kR1 kR2 R2 kR1 kR2 4kR1R2
RAB = 6 1.33
2k
= 7.98 C Ans.
DC AND DC CIRCUITS 249
Ex. 33 A wire forms a regular hexagon and the angular points Ex. 34 Find the resistance between the terminals A and B in
are joined to the centre by wires each of which has a resistance fig. 3.128.
Sol.
1 of the side of the hexagon. Find the current entering at one
The delta FDE may be replaced by its equivalent star as shown in fig.
n 3.128 (b). In fig. 3.128 (c), two series resistances along CES and CDS
angular point and leaving it by opposite point if r is the resistance have been replaced by a single resistance. The equivalent resistance
of any one side of hexagon.
3R 6R
RAB = 2R
2 7

Fig. 3.127
Sol.
Suppose r ' is the resistance of each wire connected to centre, where
r
r' .
n
Because of symmetry the currents in upper half and lower half of the
circuit are equal. Using Kirchhoff's laws, we have
i1 = i3 + i4 ...(i)
and i = 2 i1 + i2 ...(ii) Fig. 3.128
In close loop ABOA, we have Ex. 35 Using Kirchhoff's law, find the magnitude and polarity
of voltage V shown in fig. 3.127. The directions of the two current
r
– i1 r – i4 r + i2 = 0 sources are as shown.
n n
or ni1 + i4 = i2 …(iii)
In close loop BCOB, we have
r r
– i3r + i4 i4 = 0
n n
or 2 i4 = n i3 ...(iv)
From equations (i) and (iv), we get

2i2 ni1
i3 = and i4
n 2 n 2
From equation (iii) Fig. 3.129
Sol.
ni1
ni1 = i2 Suppose the directions of current in the circuit are as shown in fig. 3.129
n 2 (b). At junction A
i1 + i3 + 8 = i2 + 30
n 2 or i1 – i2 + i3 = 22 ...(i)
i1 = i2
n n 3
V V V
where i1 = , i2 = 6 and i3 =
Putting the value of i1 in equation (ii), we get 2 4
Substituting these values in equation (i), we get
2 n 2 n 4 n 1
i = i2 i2 i2 V V V
n n 3 n n 3 = 22
2 6 4

n n 3 V V V
i2 = i. Ans. or = 22
n 4 n 1 2 6 4
250 ELECTRICITY & MAGNETISM
or V = 24 V a, and the resistance of unit length of the wire is .
V 24
i1 = = = 12 A
2 2
V 24
i2 = = = – 4A
6 6
V 24
i3 = = = 6A Ans.
4 4
Ex. 36 In fig. 3.130, all the resistors have a resistance of 4.0
and all the batteries are ideal and have an emf of 4.0V each. What
is the current through resistor R ?

Fig. 3.131
Sol.
The resistance of the network between A and B will not change if we add one
more triangle across C and D. As the side CD = AB/2, and so its resistance
will be RAB/2. The equivalent network can be drawn as in fig. 3.131 (b). Thus
RAB
R
RCD 2 . Again R/2 and R/2 are in series with RCD.
RAB
R
2

Fig. 3.130 R AB
R
Sol. RAB
R R 2
RAB
Taking a close loop a b c d e f g h j k l m n o p q r s a and assuming current R R
Net becomes R 2 , Finally = 2
i in the direction of movement of loop, we have RAB RAB
+4–4–4–4–i 4 = 0 R R
2 2
or i = 2A. Ans. R R
RAB
R
Ex. 37 Determine the resistance RAB between points A and B 2
of the frame made of thin homogeneous wire (fig. 3.131), assuming
that the number of successively embedded equilateral triangles R 7 1 7 1
(wide sides decreasing by half) tends to infinity. Side AB is equal to On solving, we get RAB = a .
3 3

3.6 ELECTRICAL INSTRUMENTS


Meter bridge
The most simple practical application of Wheatstone bridge is meter bridge. It consist of
one meter long straight and uniform wire of manganin or constantan stretched along a

P R
meter scale. We know the balanced bridge condition, i.e. . After getting balance
Q S
Fig. 3.132

condition we can measure unknown resistance by S Q


R. Let balance point is at a
P

Q 1 1
distance of from A, then S R
P

Post office box


It is the compact form of Wheatstone's bridge originally designed to measure the
resistances of electric cables and telegraph wires in post offices. It consists of three arms
AB, BC and AD of the Wheatstone bridge. The unknown resistance S is connected in the
fourth arm CD. Two spring keys K1 and K2 are also provided, which when pressed make
internal connections with terminals A and B respectively. Key K1 is known as battery
DC AND DC CIRCUITS 251
key and key K2 as the galvanometer key. To determine the unknown resistance, the
connectors are made as shown in fig. 3.133. Unknown resistance can then be calculated
Q
by S R.
P
Potentiometer
It is an ideal device which is used to measure emf of a cell or to compare emf of the cells.
It is also used to measure internal resistance of a cell. It consists of a long wire usually 10
m in length and made up of manganin or constantan and a battery of known emf.

Potential gradient across the wire


It is the potential drop per unit length of the potentiometer wire. In the given figure
eR0
VAB = i R0 = R0 Rh
Fig. 3.133
V AB e R0
potential gradient = V/m
L R0 Rh L
Why long wire used in potentiometer ?
We known that sensitivity of an instrument is inversely proportional to i,
1
sensitivity
i Fig. 3.134
1
and i
R
sensitivity R
Since R
sensitivity .
1. Emf of the cell : Connect the cell whose emf, is to be determined and find balancing
length with the help of galvanometer. Let it is from A.
R0
The resistance of length of wireR AJ=
L

e R0
Potential difference across it VAJ = i RAJ =
R0 Rh L
The emf of the cell = VAJ
eR0 Fig. 3.135
= k
R0 Rh L
i.e., emf of cell

Note: It should be remembered that < e .


2. Comparision of emf's of cells : The given cells are connected in turn to the
galvanometer by means of two-way key and the position of balance point on the
potentiometer wire is obtained for each cell. Let 1 and 2 be the balancing lengths
with the cells 1 and 2 respectively, then

1 1
=
2 2 Fig. 3.136
3. Determine internal resistance of the cell : Initially key K ' remains open and
balancing length 1 is obtained. Which determine the emf of cell i.e.,
= k 1 ....(i)
252 ELECTRICITY & MAGNETISM
Now key K is closed. Again find the new balancing length. Let it is 2. In close

loop 1-2-3-4-1, i' =


R r
e
The p.d. across resistor R iR R
R r
This p.d. must equal to the p.d. across 2 length of potentiometer wire.
R
= k 2 ...(ii)
R r
From equations (i) and (ii), we have Fig. 3.137

k 1
R
= k 2
R r
or 1 R = (R + r) 2

1 2 R
r = .
2

Ex. 38 Consider the potentiometer circuit arranged as in fig. Let i is the current in the galvanometer
3.136. The potentiometer wire is 600 cm long. (a) At what distance
from A should jockey touch the wire to get zero deflection in the
galvanometer ? (b) If the jockey touches the wire at a distance of
560 cm from A, what will be the current in the galvanometer ?

Fig. 3.138

Sol.
(a) Let C is the balancing point and is the distance of jockey at which
deflection is zero in the galvanometer. The p.d., across length of
potential wire
= i RAC
Fig. 3.139
15r In close loop 1-2-3-4-1
VAC =
16r 600
– /2 – ir + 14 r (i0 – i) = 0
This p.d. is equal to the emf of cell. That is or 14 i0r – 15 ir = /2 ...(i)
In close loop 1-4-5-6-7-1
15r
/2 = –14 r (i0 – i)–i0 r –i0r + = 0
16r 600
– 16 i0r + 14 ir = – ...(ii)
which gives = 320 cm Ans.
Solving equations (i) and (ii), we get

(b) The resistance of 560 cm length of wire 3


i = . Ans.
15r 22r
= 560 14 r
600
DC AND DC CIRCUITS 253
Galvanometer
Galvanometer usually uses to detect the direction of current. When used for this purpose
its resistance becomes immaterial. If it uses to measure current or potential without any
modification, its resistance affect the circuit parameters. Therefore to make it a useful
instrument, galvanometer is to be modified according to requirement. Let ig is the full
scale deflection current of galvanometer, and G is its resistance then potential difference
across it
V = ig G
Ammeter
It is used to measure direct current. A galvanometer can be changed into an ammeter by
connecting small resistance S, called shunt in parallel with the galvanometer. To measure Fig. 3.140 Galvanometer
the current in a circuit, an ammeter must be placed in series in the circuit so that the
current to be measured actually passes through the ammeter.
Figure shows an ammeter in which shunt S is connected in parallel with galvanometer
Resistance of ammeter
SG
RA = <S ...(i)
S G
If i is the current to be measured, then
S (i – ig) = G ig

S = ig
G ...(ii)
i ig

S
or ig = i ...(1)
S G
Equation (1) is the working equation of an ammeter
Fig. 3.141
i SG
Also ig =
G S G
RA
or ig = i …(2)
G
Let us consider a circuit having a source of emf without the aid of ammeter, actual
current i = /R.
When ammeter is connected to measure the current, its value changed due to resistance
of ammeter. Let it is i'.

i' = <i
R RA

i' = i , if RA = 0 (ideal ammeter)


R
Percentage error in measuring current by ammeter

i i' R R RA RA
100 = 100 100
i R RA Fig. 3.142
R
Conversion of an ammeter into other ammeter
It is same as the conversion of galvanometer into ammeter. If i1 is the initial range of
ammeter and i2 is the final range, then
S
i1 = i2 S RA

S
ig = i … (i) Fig. 3.143
S G
254 ELECTRICITY & MAGNETISM
If s1 and s2 are the sentivities of ammeters, then
s1 i2
s2 = i1
The equation (1) now can be written as
i1 s2 S
i2 = s1 S RA ...(3)

Voltmeter
It is used to measure potential difference between two points of a circuit containing
direct current source. A galvanometer can be changed into a voltmeter by connecting a
very large resistance (R0) in series with the galvanometer. The voltmeter is always
connected in parallel across the points.
The potential difference across R = potential difference across voltmeter
or V = ig RV = ig (G + R0)

or R0 = V
G ...(4)
ig
The resistance of voltmeter RV = G + R0
The actual potential difference across resistor R
V = iR
When it is measured with a voltmeter of resistance RV, it is
R RV
Fig. 3.144 V = i
R RV
iR
= .
R
1
RV
If RV , V'=iR=V
Thus, the resistance of ideal voltmeter is infinite.
Percentage error in measuring potential difference with a voltmeter
iR
iR
R
1
V V' RV
100 = 100
V iR

R
RV R
= 100 100
R RV R
1
RV

1
= 100 .
R
1 V
R
for RV , % error = 0
Conversion of one voltmeter into other voltmeter : It is like to
change galvanometer into voltmeter.
V = ig RV = ig (G + R0)

Fig. 3.145 V2 ig ( RV1 R0 )


DC AND DC CIRCUITS 255
Ex. 39 An electrical circuit is shown in fig. 3.146. Calculate Sol.
the potential difference across the resistor of 400 ohm as will be Given G = 30 , ig = 2 mA = 2 × 10–3 A.
measured by the voltmeter V of resistance 400 ohm either by (a) If S is the required shunt connected in parallel with the
applying Kirchhoff's rules or otherwise. galvanometer, then

S
ig = i
S G

S
or 2 × 10–3 = 0.3
S 30

30
After solving, we get S = Ans.
149
(b) To convert galvanometer into voltmeter a high resistance R0 is
connected in series with galvanometer, and so
Fig. 3.146 V = ig (G + R0)
Sol. The given circuit with the voltmeter reduces to a balanced or 0.2 = 2 × 10–3 (30 + R0)
Wheatstone's bridge.
or R0 = 70 Ans.

Ex. 41 A galvanometer having 30 divisions has a current


sensitivity of 20 A / division. It has a resistance of 25 ohm. How
will you convert it into an ammeter measuring upto 1 A? How will
you now convert this ammeter into a voltmeter reading upto 1 V?

Sol.
The full scale deflection current
ig = 30 × (20 × 10–6)
= 6 × 10–4 A.
If S is the required value of the shunt connected in parallel with the
galvanometer, then

S
ig = i
S G

Fig. 3.147 S
or 6 × 10–4 = 1
The effective resistance across the source = 150 S 25
Current drawn from the cell
150
10 1 After solving, we get S = 0.0150 Ans.
i = A 9994
150 15
The resistance of the ammeter
The current is equally divided into both the branches. Therefore current
SG
1 RA =
in 200 is A S G
30
0.0150 25
1 20 = 0.0150
The p.d. across 200 = 200 V. Ans. 0.0150 25
30 3
To convert this ammeter into the voltmeter, we can use
Ex. 40 A galvanometer has a resistance of 30 and current of
V = ig (RA + R0)
2 mA is needed to give full scale deflection. What is the resistance
Here V = 1V, ig = 1A
needed and how is it to be connected to convert the galvanometer
(a) into an ammeter of 0.3 A range? 1 = 1 (0.0150 + R0)
(b) into a voltmeter of 0.2 V range? or R0 = 0.985 Ans.
256 ELECTRICITY & MAGNETISM

Review of formulae & Important Points


1. Electric current : An electric current i in a conductor is defined
1 1 1
by (ii) In parallel : = ...
R R1 R2
dq
i = . 8. Changing the size of the resistor :
dt If the conductor is stretched from length to , then its resistance
By convention, the direction of electric current is taken as the changes from R to R as
direction in which positive charge carriers would move.
2
'
R = R
2. Current density : Current density is related to the current as

i = J dA In terms of radius of cross-section of the conductor


4
r
where dA is a vector perpendicular to a surface element of area R = R .
dA, and the integral is taken over any surface cutting across the r'
9. EMF of the cell : It is the work done in circulating unit charge in
conductor. J has the same direction as the velocity of the moving
the entire circuit. Thus
positive charges.
dW
3. Drift speed : When an electric field E is established in a conductor, = .
the charge carriers (assumed positive) acquire a drift speed vd in dq
Potential difference across the terminals of the cell
the direction of E; the velocity is related to current density J as
V = ir
J = ne vd
where r is the internal resistance of the cell.
4. Resistance of a conductor : If V is the p.d. applied across the In case when cell is charging, the p.d. across the terminals of the
conductor and i is the corresponding current, then its resistance is cell
defined as :
V = ir.
V 10. Combination of cells :
R = .
i
(i) In series :If n identical cells each of emf and internal
Similarly we can define resistivity and conductivity of a
resistance r are connected in series, then current in external
material :
resistor R
1 E
= n
J i =
nr R
Also E = J In case when nr < < R,
If n is the number of free electrons per unit volume and is the
relaxation time, then i n
R
2m
=
ne 2 (ii) In parallel : i =
r
The resistance R of a conducting wire of length L and uniform R
n
cross-section A is
L In case R < < nr,
R = .
A n .
i
5. Change in or R with temperature : If R0 is the resistance of r
a wire at temperature 0°C, then resistance at any temperature t is (iii) Series-parallel : If n cells are connected in series and m
Rt ; R0 1 t cells are in parallel, then
where is the temperature coefficient of resistance. It can be n
defined as i =
nr
R
1 dRt m
=
R0 dt For maximum current
6. Ohm s law : Under given physical conditions the current i nr
produced in the conductor is proportional to the applied potential = R.
m
difference across the conductor.
7. Combination of resistances : 11. In a series of five identical cells each of emf and internal resistance
(i) In series : R = R1 + R2 + ... r, if one of the cells is wrongly connected, then net emf
net = 3 and rnet = 5r
DC AND DC CIRCUITS 257
Similarly in a series of n cells if m cells are wrongly connected,
then R2 R3
R23 = R1 R2 R3
net = n 2m and rnet = nr

12. If two cells of emfs 1 and 2 and internal resistances r1 and r2


are connected in parallel, then p.d. across the combination

1r2 2 r1
= r1 r2

For any two junctions RAB in delta is equal to RAB in star, similarly
RAC and RBC.
16. Metre bridge : It is used to find unknown resistance. If be the
13. Circuit analysis by Kirchhoff s laws :
(i) Junction rule : At any junction, the algebraic sum of currents balanced length and R is the known resistance, then unknown
is equal to zero. i.e., resistance
i = 0 1
(ii) Loop rule : In a close loop, the algebraic sum of p.d. across all S = R .
circuit elements is equal to zero, i.e.,
= 0. 17. Potentiometer : It is an ideal device of finding emf of the cells,
V
internal resistance of the cell etc.
For a circuit having resistors and cells
iR = 0

Sign conventions :

14. Wheatstone bridge : It is the combination of four resistances in


the form of bridge. For the balanced bridge with the resistors P, Q,
R and S If R0 is the resistance of the potentiometer wire, then emf of the
cell
P R P Q
= or
Q S R S e = ,
0
where is the balancing length and 0 is the length of the
potentiometer wire.
Internal resistance

1 2
r = R ,
2
The equivalent resistance across the terminals of the cell
where 1 and 2 are the balancing lengths without R and with R.
1 1 1 18. Ammeter : Galvanometer of resistance G and full scale deflection
Req = current ig can be converted into an ammeter of range i by connecting
P Q R S
a shunt of resistance S, such that
15. Delta–star transformation : A combination of three resistors in
the form of delta can be effectively converted into star. A delta of S
ig = i
three resistors R1, R2 and R3 is equivalent to a star with three S G
resistors R12, R13 and R23, where
SG
Resistance of ammeter RA =
R1R2 S G
R12 = R1 R2 R3 19. Voltmeter : A galvanometer of resistance G and full scale deflection
current ig can be converted into a voltmeter of range V by connecting
R1R3 a large resistance R0 in series, such that
R13 = R1 R2 R3 ig G R0 .
V =
258 ELECTRICITY & MAGNETISM

MCQ Type 1 Exercise 3.1


Level -1
Only one option correct
1. The value of the resistance as measured across terminals A and B in
figure would be (a) i, vd (b) i, j
(c) j, E, vd (d) i, j and E.
6. Figure shows four situations in which positive and negative charges
move horizontally through a region and gives the rate at which
each charge moves. The situations according to the effective current
through the region, greatest first :

(a) 5k (b) 10 k
(a) A > B > C > D (b) (A = B) > C > D
(c) 15 k (d) 20 k
(c) A < B < C < D (d) (A = C) > D > B
2. The equivalent resistance between A and B is 7. The equivalent resistance between A and B is :

(a) 10 (b) 20
(a) R1+ R2 + R3 (b) R2
(c) 15 (d) none
R2 R4 R1 R4
(c) R1 (d) R R 8. The figure shows three cylindrical copper conductors along with
R2 R4 1 4 their face areas and lengths. Rank them according to the current
3. The time constant of the circuit is : through them, greatest first

(a) RC (b) RC / 2
(c) 2RC (d)
(a) i1 = i2 = i3 (b) i1 > i2 > i3
4. In the figure, the resistors are in series or parallel. Select the right
(c) i1 < i2 < i3 (d) (i1 = i3) < i2
trend : 9. Figure shows a rectangular solid conductor of edge lengths L, 2L,
and 3L. A certain potential difference V is to be applied between
pairs of opposite faces of the conductor as shown in figure : left-
right, top-bottom and front-back. In which pair current is maximum
:

(a) series, series, series (b) series, series, parallel


(c) series, parallel, parallel (d) none
5. The figure here shows conduction electrons moving leftward (a) left-right (b) top-bottom
through a wire. Select the quantity/(ies) which have direction (c) front-back (d) equal in all
rightward :

Answer Key 1 (d) 2 (d) 3 (c) 4 (c) 5 (d)


Sol. from page 287 6 (d) 7 (a) 8 (d) 9 (b)
DC AND DC CIRCUITS 259

10. A resistance of 2 is connected across one gap of a metre-bridge


(the length of the wire is 100 cm) and an unknown resistance,
(b)
greater than 2 , is connected across the other gap. Where these
resistances are interchanged, the balance point shifts by 20 cm.
Neglecting any correction, the unknown resistance is
(a) 3 (b) 4

(c) 5 (d) 6 (c)

11. A wire of resistance 4 is bent to form a circle. The resistance


between A and B is

(d)

16. The current from the battery in circuit diagram shown is

(a) 4 (b) 3

3
(c) (d) none of these
4
12. When a piece of aluminium wire of finite length is drawn through
a series of dies to reduce its diameter to half its original value, its
(a) 1 A (b) 2 A
resistance will become
(c) 1.5 A (d) 3 A
(a) two times (b) four times
17. In the circuit shown below, the cell has an e.m.f. of 10 V and
(c) eight times (d) sixteen times internal resistance of 1 ohm. The other resistances are shown in
13. A steady current flows in a metallic conductor of non-uniform the figure. The potential difference VA – VB is
cross-section. The quantity/quantities constant along the length
of the conductor is/are
(a) current, electric field and drift speed
(b) drift speed only
(c) current and drift speed
(d) current only
14. A strip of copper and another of germanium are cooled from room
temperature to 80 K. The resistance of
(a) 6 V (b) 4 V
(a) each of these increases
(c) 2 V (d) –2 V
(b) each of these decreases
18. In the given figure, potential difference between A and B is
(c) copper strip increases and that of germanium decreases
(d) copper strip decreases and that of germanium increases
15. Express which of the following setups can be used to verify Ohm's
law

(a)
(a) 0 (b) 5 volt
(c) 10 volt (d) 15 volt

Answer Key 10 (a) 11 (c) 12 (d) 13 (a) 14 (d)


Sol. from page 287 15 (a) 16 (a) 17 (d) 18 (c)
260 ELECTRICITY & MAGNETISM
19. If each resistance in the figure is of 9 then reading of ammeter (a) P and Q (b) Q and R
is (c) P and R (d) Any two points
24. The total current supplied to the circuit by the battery is

(a) 5 A (b) 8 A
(c) 2 A (d) 9 A
20. Two wires of equal diameters, of resistivities 1 and 2 and lengths
1 and 2, respectively, are joined in series. The equivalent resistivity (a) 1 A (b) 2 A
of the combination is (c) 4 A (d) 6 A
1 1 2 2 1 2 2 1 25. A cell of e.m.f. is connected with an external resistance R, then
(a) (b) p.d. across cell is V. The internal resistance of cell will be
1 2 1 2

1 2 2 2 1 1 – 2 2 V R V R
(c) (d) (a) (b)
1 2 1 2 V
21. In the arrangement of resistances shown below, the effective
resistance between points A and B is V R V R
(c) (d)
V
26. The potential difference in open circuit for a cell is 2.2 volt. When
a 4 ohm resistor is connected between its two electrodes the
potential difference becomes 2 volt. The internal resistance of the
cell will be
(a) 1 ohm (b) 0.2 ohm
(c) 2.5 ohm (d) 0.4 ohm
(a) 20 (b) 30
27. Eels are able to generate current with biological cells called
(c) 90 (d) 110 electroplaques. The electroplaques in an eel are arranged in 100
22. Calculate the equivalent resistance between A and B rows, each row stretching horizontally along the body of the fish
containing 5000 electroplaques. The arrangement is suggestively
shown below. Each electroplaques has an emf of 0.15 V and internal
resistance of 0.25

9
(a) (b) 3
2
5
(c) 6 (d)
3
23. Six equal resistances are connected between points P, Q and R as
shown in the figure. Then the net resistance will be maximum
between

The water surrounding the eel completes a circuit between the


head and its tail. If the water surrounding it has a resistance of
500 , the current an eel can produce in water is about
(a) 1.5 A (b) 3.0 A
(c) 15 A (d) 30 A

Answer Key 19 (a) 20 (a) 21 (a) 22 (a) 23 (a)


Sol. from page 287 24 (c) 25 (b) 26 (d) 27 (a)
DC AND DC CIRCUITS 261

28. Find out the value of current through 2 resistance for the given
circuit

(a) IR = IG (b) IP = IG
(c) IQ = IG (d) IQ = IR
(a) 5 A (b) 2 A
33. For the post office box arrangement to determine the value of
(c) zero (d) 4 A
unknown resistance, the unknown resistance should be connected
29. A 2 volt battery, a 15 resistor and a potentiometer of 100 cm
between
length, all are connected in series. If the resistance of potentiometer
wire is 5 , then the potential gradient of the potentiometer wire is
(a) 0.005 V/cm (b) 0.05 V/cm
(c) 0.02 V/cm (d) 0.2 V/cm
30. In the diagram shown, the reading of voltmeter is 20 V and that of
ammeter is 4 A. The value of R should be (Consider given ammeter
and voltmeter are not ideal)

(a) B and C (b) C and D


(c) A and D (d) B1 and C1
34. In a metre bridge experiment null point is obtained at 20 cm from
(a) equal to 5 one end of the wire when resistance X is balanced against another
(b) greater than 5 resistance Y. If X < Y, then where will be the new position of the
(c) less than 5 null point from the same end, if one decides to balance a resistance
(d) greater or less than 5 depends on the material of R of 4X against Y
31. AB is a wire of uniform resistance. The galvanometer G shows no (a) 50 cm (b) 80 cm
current when the length AC = 20 cm and CB = 80 cm. The resistance
(c) 40 cm (d) 70 cm
R is equal to
35. A moving coil galvanometer of resistance 100 is used as an
ammeter using a resistance 0.1 The maximum deflection current
in the galvanometer is 100 A. Find the minimum current in the
circuit so that the ammeter shows maximum deflection
(a) 100.1 mA (b) 1000.1 mA
(c) 10.01 mA (d) 1.01 mA
36. Potentiometer wire of length 1 m is connected in series with 490

(a) 2 (b) 8 resistance and 2V battery. If 0.2 mV/cm is the potential gradient,
then resistance of the potentiometer wire is
(c) 20 (d) 40
(a) 4.9 (b) 7.9
32. In the circuit shown P R , the reading of the galvanometer is
same with switch S open or closed. Then (c) 5.9 (d) 6.9

Answer Key 28 (c) 29 (a) 30 (c) 31 (c) 32 (a)


Sol. from page 287 33 (c) 34 (a) 35 (a) 36 (a)
262 ELECTRICITY & MAGNETISM
37. Twelve wires of equal length and same cross-section are connected shown in the figure. If R1 is the initial resistance and R2 that after
in the form of a cube. If the resistance of each of the wires is R,
then the effective resistance between the two diagonal ends would R2
connecting the two identical conductors, then is :
be R1
(a) 2R
(a) 1.4
(b) 12 R (b) 0.4
(c) 0.5
5
(c) R (d) 0.6
6
43. A voltmeter has a resistance G ohm and range V volt. The value of
(d) 8R
resistance used in series to convert it into voltmeter of range nV
38. The potential difference across 8 ohm resistance is 48 volt as
volt is :
shown in the figure. The value of potential difference across X and
Y points will be (a) nG (b) (n – 1) G

G G
(c) (d)
n (n 1)

44. In the circuit shown, each resistance is 2 . The potential V1 as


indicated in the circuit, is equal to

(a) 160 volt (b) 128 volt 2


(c) 80 volt (d) 62 volt
39. In the circuit element given here, if the potential at point B, VB = 0, 2 2
2
then the potentials of A and D are given as
5V
12V
V1
(a) VA 1.5 V , VD 2V 2
(b) VA 1.5V , VD 2V
(c) VA 1.5V , VD 0.5V (a) 11 V (b) 11V

(d) VA 1.5V , VD 0.5V (c) 9 V (d) 9V


40. The current in a conductor varies with time t as 45. The potential difference, in volt, across the resistance R2 in the
i = 2t + 3t2 where i is in ampere and t in seconds. Electric charge circuit shown is :
flowing through a section of the conductor during t = 2 sec to t = 3
sec is
(a) 10 C (b) 24 C
(c) 33 C (d) 44 C
41. As the switch S is closed in the circuit shown in figure, current
passed through it is (a) 5 (b) 7.5
(c) 10 (d) 15
46. The length of a wire of a potentiometer is 100 cm, and the emf of
its standard cell is volt. It is employed to measure the e.m.f. of a
battery whose internal resistance is 0.5 . If the balance point is
obtained at = 30 cm from the positive end, the emf of the battery
is ( where i is the current in the potentiometer)
(a) 4.5 A (b) 6.0 A 30 30
(a) (b)
(c) 3.0 A (d) zero 100 100.5
42. What will be the change in the resistance of a circuit consisting of 30 30 0.5i
five identical conductors if two similar conductors are added as (c) 100 0.5 (d)
100

Answer Key 37 (c) 38 (a) 39 (c) 40 (b) 41 (a)


Sol. from page 287 42 (d) 43 (b) 44 (d) 45 (b) 46 (a)
DC AND DC CIRCUITS 263
47. In the figure shown, the capacity of the condenser C is 2 F. The (a) 0.5 A (b) 1 A
current in 2 resistor is (c) Zero (d) 0.25 A
R1 R3
51. In the circuit, the values of resistances are such that R RG ,
2
where RG is the internal resistance of the galvanometer. The reading
of the galvanometer

R1 R2

K
(a) 9A (b) 0.9 A R3 G
1 1
(c) A (d) A
9 0.9
48. In the given circuit, it is observed that the current I is independent
of the value of the resistance R6. Then the resistance values must (a) always shows zero value
satisfy (b) increases when the switch K is ON
(c) increases when the switch condition is charged from ON to
OFF
(d) remains constant whether the switch K is ON or OFF.
52. If 1, 2, 3 are the conductances of three conductors, of equal size
then their equivalent conductance, when they are joined in series,
will be
1 1 1
(a) 1+ 2+ 3 (b)
1 2 3

(a) R1R2 R5 R3 R4 R6 3 1 2 3
(c) (d) none of these
1 2 2 3 1 3
1 1 1 1
(b) 53. In the given diagram the current through the battery and the charge
R5 R6 R1 R2 R3 R4 on the capacitor respectively in steady state are
(c) R1R4 R2 R3
6V
(d) R1R3 R2 R4 R5 R6
49. In the shown arrangement of the experiment of the meter bridge if 1
AC corresponding to null deflection of galvanometer is x, what 2
would be its value if the radius of the wire AB is doubled
3
0.5µF 4

(a) 1 A and 3µC (b) 17 A and 0 µC


6 12
(c) A and µC (d) 11 A and 3µC
7 7
54. In the circuit shown, the point 'B' is earthed. The potential at the
(a) x (b) x / 4
point 'A' is
(c) 4 x (d) 2 x
50. Current i as shown in the circuit will be 5 7 B
A
3 10
i
50 V C
6 3
E D
(a) 14 V (b) 24 V
2V 2 (c) 26 V (d) 50 V

Answer Key 47 (b) 48 (c) 49 (a) 50 (b)


Sol. from page 287 51 (d) 52 (c) 53 (d) 54 (b)
264 ELECTRICITY & MAGNETISM
55. The effective resistance across the points A and I is R
C
2 2 i1 1 r1

B D
2
i2
A 2 2 r2
H I 2
2
2 (a) 1 – (i1 + i2)R – i1r1 = 0 (b) 2 – i2r2 – 1 – i1r1 = 0
G 2 E (c) – 2 – (i1 + i2)R + i2r2 = 0 (d) 1 – (i1 + i2)R + i1r1 = 0
60. In the given circuit the current I1 is
2 2
30
F
(a) 2 (b) 1 I1
40 I
(c) 0.5 (d) 5
56. Two wires of equal diameters, of resistivities 1 and 2 and lengths 40V
I2
l1 and l2, respectively, are joined in series. The equivalent resistivity 40
of the combination is
80V
1l1 2l2 1l2 2l1
(a) (b) (a) 0.4 A (b) – 0.4 A
l1 l2 l1 l2 (c) 0.8 A (d) – 0.8 A
61. The batteries, one of emf 18 volt and internal resistance 2 and
1l2 2l1 1l1 2l2
(c) (d) the other of emf 12 volt and internal resistance 1 , are connected
l1 l2 l1 l2 as shown. The voltmeter V will record a reading of
57. Find the equivalent resistance across AB.
V
A
2 18V 2
2
2
2 2 12V 1

B (a) 15 V (b) 30 V
(c) 14 V (d) 18 V
(a) 1 (b) 2
62. Shown in the figure below is a meter-bridge set up with null
(c) 3 (d) 4
deflection in the galvanometer. The value of the unknown resistor
58. Thirteen resistances each of resistance R ohm are connected in the
R is
circuit as shown in the figure below. The effective resistance between
A and B is 55 R

R R
G
R R
R R 20 cm

A R B
(a) 220 (b) 110
R R R (c) 55 (d) 13.75
R
63. The resistance of a galvanometer is 50 and it shows full scale
R R deflection for a current of 1 mA. To convert it into a voltmeter to
measure 1V and as well as 10 V (refer circuit diagram) the resistance
R1 and R2 respectively are
4R
(a) 2R (b) R1 R2
3 G
2R
(c) (d) R
3
V1 V2
59. See the electrical circuit shown in this figure. Which of the following
equations is a correct equation for it? (a) 950 and 9150 (b) 900 and 9950
(c) 900 and 9900 (d) 950 and 9000

Answer Key 55 (b) 56 (a) 57 (a) 58 (c) 59 (a)


Sol. from page 287 60 (b) 61 (c) 62 (a) 63 (d)
DC AND DC CIRCUITS 265
64. A resistance of 4 and a wire of length 5 metres and resistance 5
are joined in series and connected to a cell of e.m.f. 10 V and (a) (b)
internal resistance 1 . A parallel combination of two identical
2 2
cells is balanced across 300 cm of the wire. The e.m.f. E of each cell
(c) (d)
is
4 10 V 66. A group of N cells whose emf varies directly with the internal
resistance as per the equation EN = 1.5 rN are connected as shown
1 in the figure below. The current I in the circuit is
3m 1 I
E 5 5m 2
r1 r2
G
E
N rN r3
(a) 1.5 V (b) 3.0 V
3
(c) 0.67 V (d) 1.33 V
65. Two resistance R1 and R2 are made of different materials. The r4
temperature coefficient of the material of R1 is and of the material 4
of R2 is – . The resistance of the series combination of R1 and R2 (a) 0.51 A (b) 5.1 A
will not change with temperature, if R1/R2 equals (c) 0.15 A (d) 1.5 A

Answer Key 64 (b) 65 (d) 66 (d)


Sol. from page 287

Level -2
Only one option correct 1. The equivalent resistance of the network between points A
1. The time constant of the circuit is : 4R
and B is .
3
2. The current in resistor 3 R is zero.
3. The potential difference across R is equal to the potential
(a) RC / 2 (b) RC difference across 2R.
(c) 2RC (d) zero Which of the above statement (s) is/are correct ?
2. The equivalent resistance between A and B is :
(a) 1 alone (b) 2 alone
(c) 2 and 3 (d) 1, 2 and 3.
4. What is the effective resistance between the terminals A and B of
the mesh shown in figure?

33 66
(a) (b)
21 45
8 (a) 2R (b) R
(c) (d) none
7 (c) R/2 (d) R / 3
3. Consider the following statements regarding the network shown 5. The temperature coefficient of resistance for a wire is 0.00125/°C.
in the figure : At 300K its resistance is 1 ohm. The temperature at which the
resistance becomes 2 ohm is
(a) 1154 K (b) 1100 K
(c) 1400 K (d) 1127 K

Answer Key 1 (b) 2 (b) 3 (d)


Sol. from page 290 4 (c) 5 (d)
266 ELECTRICITY & MAGNETISM
6. In the adjoining circuit, the battery 1has an e.m.f. of 12 volt and 11. In the given circuit, with steady current, the potential drop across
zero internal resistance while the battery has an e.m.f. of 2 volt. the capacitor must be
If the galvanometer G reads zero, then the value of the resistance X
in ohm is

(a) V (b) V/ 2
(a) 10 (b) 100 (c) V/ 3 (d) 2V / 3
(c) 500 (d) 200 12. A wire of length L and 3 identical cells of negligible internal
7. Two sources of equal emf are connected to an external resistance R. resistances are connected in series. Due to current, the temperature
The internal resistances of the two sources are R1 and R2(R2 > R1). of the wire is raised by T in a time t. A number N of similar cells
If the potential difference across the source having internal resistance is now connected in series with a wire of the same material and
R2 is zero, then cross-section but of length 2L. The temperature of the wire is
(a) R R1R2 / R1 R2 raised by the same amount T in the same time t. the value of N is
(a) 4 (b) 6
(b) R R1R2 / R2 R1 (c) 8 (d) 9
13. The effective resistance between points P and Q of the electrical
(c) R R2 R1 R2 / R2 R1 circuit shown in the figure is

(d) R R2 R1
8. In the given figure, battery is balanced on 55 cm length of
potentiometer wire but when a resistance of 10 is connected in
parallel with the battery then it balances on 50 cm length of the
potentiometer wire then internal resistance r of the battery is

(a) 2Rr / (R + r) (b) 8 R(R + r) / (3R + r)


(c) 2r + 4R (d) 5R / 2 + 2r
14. The two ends of a uniform conductor are joined to a cell of e.m.f.
and some internal resistance. Starting from the midpoint P of the
conductor, we move in the direction of current and return to P. The
potential V at every point on the path is plotted against the distance
(a) 1 (b) 3 covered (x). Which of the following graphs best represents the
resulting curve?
(c) 10 (d) 5
9. A voltmeter has a range 0 – V with a series resistance R. With a
series resistance 2R, the range is 0 –V . The correct relation between
V and V is
(a) V = 2 V (b) V > 2V
(c) V >> 2V (d) V < 2V (a) (b)
10. A microammeter has a resistance of 100 and full scale range of
50 A. It can be used as a voltmeter or as a higher range ammeter
provided a resistance is added to it. Pick the correct range and
resistance combination
(a) 50 V range with 10 k resistance in series
(b) 10 V range with 200 k resistance in series
(c) 10 mA range with 1 resistance in parallel (c) (d)

(d) 10 mA range with 0.1 resistance in parallel

Answer Key 6 (b) 7 (d) 8 (a) 9 (d) 10 (b)


Sol. from page 290 11 (c) 12 (b) 13 (a) 14 (b)
DC AND DC CIRCUITS 267
15. A voltmeter connected to the terminals of a battery reads 6 V. (a) 0 V (b) 15 V
When the lamps are lighted with the battery, the voltmeter reads 4 (c) 7 V (d) 3 V
V. If the resistance of the lamps is 10 , the internal resistance of 20. What is the resistance between P and Q of the following circuit ?
the battery is : Each resistance is of 1 :
(a) 0.5 (b) 10 a b c d Q
P
(c) 3 (d) 5
16. Three batteries of negligible internal resistance and three resistors
of 4, 8 and 12 are connected as shown in figure here. The current
through the 12 resistor is :
f e
A B C (a) 1 (b) 2
3 a
6V – 4 +
– 8V (c) 3 (d)
+ 2
– 10V
21. Point a in figure is maintained at a constant potential 100 k
+
of 300 V above ground. The reading of a voltmeter
8 D 12 of the proper range, and of resistance 3 × 104 , b
when connected between point b and ground is
(a) 0.57 A (b) 1.09 A
nearly : 200 k
(c) 0.04 A (d) 1.14 A
(a) 10 V
17. In the given circuit Ia, Va and the power supplied by the 15 V
(b) 42 V
battery respectively are :
(c) 50 V
800 3k (d) 62 V
22. If energy stored in the capacitors C1 and C2 are same, then what is

+

15 V 50 V C1
– + the value of C ?
Va Ia 2

+

1200
(a) 7 mA, 8.4 V and 58.8 mW
(b) 0.01 mA, 2 V and 0.02 mW
(c) 7 mA, 8.4 V and 105 mW
(d) 0.01 mA, 8.4 V and 105 mW
18. In the given network of four resistances, the equivalent resistance
is :
7

Req 6 25 1
3 (a) (b)
36 36
4
4 9
(a) 20 (b) 5.4 (c) (d)
9 4
(c) 12 (d) 4.5 23. The key K is connected in turn to each of the contacts over short
19. In the part of a circuit shown in the figure, the potential difference identical time intervals so that change in charge on the capacitor
between G and H (VG – VH) will be : over each connection is small. The final charge q0 on the capacitor
is
G
2A 1A 3V 1

4 1
C
2 5V 2
– + + –
3V 2V 1
1

3A (a) 2.5 C (b) 8 C


H
C
(c) 5C (d)
2

Answer Key 15 (d) 16 (d) 17 (c) 18 (d) 19 (c)


Sol. from page 290 20 (d) 21 (d) 22 (a) 23 (a)
268 ELECTRICITY & MAGNETISM
24. In the given mesh, each resistor has resistance R. The effective
1
resistance between the terminals A and B is (a) (b) 0.25 m
R 5m
A (c) 0.50 m (d) none of these
R
28. Potential of certain points in circuit are maintained as marked.
R
R What is reading of voltmeter (If ammeter reads zero)?

V
R + 7.5 V 1
B 10 V
3R R 1
(a) (b)
8 2
2 2 5V
(c) R (d) 2 R
25. The total current drawn from the battery is: 5V A
8
5V 0V – 5V

24V
(a) 10 V (b) 2.5 V
6
(c) 5V (d) 20 V
29. The terminal network shown in the figure consists of 6 resistors.
The points A, C and E all are at potential 20 V while points B, D
12 and F are at potential –10 volt then potential of junction O will
(a) 3A (b) 45 A
(c) 6 A (d) 9 A F E
26. Plates of a parallel plate capacitor C have charges CV and 3 CV on
its plates. If switch is closed at t = 0. Then initial rate at which heat
energy is produced in resistance R is 2r
r
CV 3 CV 2r
A D
r O
r
2r

B C
R V

V2 4V 2 (a) zero (b) 10 V


(a) (b)
R R (c) 15 V (d) –5 V
2 2 30. All wires have same resistance and equivalent resistance between
9V 16V
(c) (d) A and B is R0. Now keys are closed, then the equivalent resistance
R R will become
27. A battery of emf E0 = 6 V is connected across a 2 m long uniform
wire having resistance 4 /m. The cells of small emf 1 = 2 V and K1

2 = 3 V having internal resistance 2 &1 respectively are


connected as shown in the figure. The null point will be obtained at B

0 =6V 8

A B
2
r2 K2

G 7 R0 7 R0
X Y
(a) (b)
r1 3 9
1
R0
(c) 7 R0 (d)
3

Answer Key 24 (a) 25 (d) 26 (c) 27 (d)


Sol. from page 290 28 (a) 29 (b) 30 (b)
DC AND DC CIRCUITS 269
31. Find equivalent resistance between A & B in the following circuit 36. In following circuit, key is closed at time t = 0, then what will be
current through battery at that time?

A R R R R R B

R 2
3R 2R
(a) (b) 2
2 3
(c) 2 R (d) 3 R
32. Two wires are of same length and same area of cross-section. If
first wire has resistivity 1 and temperature coefficient of K 6V 2
resistance 1 but second wire has resistivity 2 and temperature
coefficient of resistance 2. Their series equivalent resistance is (a) 3 A (b) 1.5 A
independent of small temperature changes. Then (c) 2 A (d) 6 A
(a) 1+ 2 =0 (b) 1 1 = 2 2 37. A capacitor is charged up to V volts. The space between two
(c) 1 1 + 2 2 = 0 (d) 1 2 + 2 1 = 0 plates of capacitor is filled with dielectric medium of constant K
33. A metal wire has coefficient of linear expression 1 and temperature and conductivity . The time after which charge on capacitor
coefficient of resistivity 2. Apparent temperature coefficient of
resistance will be 1
becomes times its initial charge is
(a) 2 (b) 2 – 1 e
(c) 2 + 1 (d) 2– 2 1 K 0 0
34. In the circuit shown the ammeter A reads a current of i1 amp. If (a) (b)
key K1 is opened and K2 is closed ammeter reads i2, then K

2k k 0A
(c) (d)
d2
R 38. An ammeter and a voltmeter are connected in series to a battery.
E A When a certain resistance is connected in parallel to voltmeter
reading of voltmeter becomes half while reading of ammeter becomes
R R
double. What is the ratio of voltmeter resistance and ammeter
resistance?
K1 R R K2 1 3
(a) (b)
3 2
(a) i1 > i2 (b) i1 < i2
(c) i1 = i2 (d) depend on the value of R 2 3
(c) (d)
35. A capacitor is initially connected to a battery of emf 3V. At 1 1
t = 0, switch is thrown to B state. Now charge on capacitor at any C1
instant is given by 39. In the shown network charges in capacitors are same, then C is
2
3V
C1

R A 1 2 3
C
B

C2
V
R 2V R

t
t 5
(a) (b) q C V 2V e RC (a) (b) 1 : 1
q C V 2V e RC 3
(c) 1:3 (d) 1 : 5
t 2t
(c) q C V 2V e 2CR (d) q C V 2V e RC

Answer Key 31 (b) 32 (c) 33 (b) 34 (c) 35 (c)


Sol. from page 290 36 (a) 37 (a) 38 (c) 39 (a)
270 ELECTRICITY & MAGNETISM
40. What is potential difference between A and B? 44. The equivalent resistance of the circuit between A and B is:

R R
2.8
R R
10 V
A B
A B
2 5
2.5 2.5 R R

R R
3 5
R R
(a) 8.4 V (b) 2.4 V (a) (b)
8 4
(c) 4.2 V (d) 7.2 V (c) 2 R (d) 4 R
41. In following circuit, resistance of potentiometer wire is 10 . Power 45. If equivalent resistance between points A and X is 5 and equivalent
consumption in potentiometer wire is same when jocky is placed resistance between A and B is 10 then R2 is
at 10 cm from end A or end B. Internal resistance of cell(r) is R1
A
r

2V 3
X
A 1m B
2V
(a) 5 (b) 6 B
(c) 2 (d) 3 R2
42. The effective resistance between the terminals A and B is. (a) 2 (b) 5
(c) 3 (d) 10
46. The current in 2 resistor in the circuit shown is:
6
10

10

4.5 V
5
2
3
10

3
5 10
A

B
5
6
1 1
(a) 5 (b) 10 (a) A (b) A
(c) 15 (d) 20 4 2
43. The reading of ammeters A1 and A2 for the circuit shown 3
(c) A (d) 2 A
4
6 47. The resistance between points A and B in the network shown in
figure is: A
3 6
A2 6 4
A1 6 (a) 2.46
6 3
6 (b) 4.23 3 5

(c) 5.13 5 2 4
18V

(a) 1 A, 1A (b) 1 A, zero (d) none of these


B
(c) zero, 1A (d) zero, zero

Answer Key 40 (a) 41 (d) 42 (a) 43 (b)


Sol. from page 290 44 (a) 45 (b) 46 (a) 47 (d)
DC AND DC CIRCUITS 271
48. A wire has linear resistance (in Ohm/m). Find the resistance R 52. To verify Ohm’s law, a student is provided with a test resistor RT,
between points A and B if the side of the larger square is d. a high resistance R 1 , a small resistance R 2 , two identical
A galvanometers G1 and G2, and a variable voltage source V. The
(a) d/ 2 correct circuit to carry out the experiment is
G1
(b) 2 d
R2
(c) 2 d
(d) none of these B (a) RT R1
49. For the shown circuit the effective resistance between the points
A and B will be V

2R
R1
R R
(b)
R/2 R/2
2R 2R
R R
A B
4R
R R R R
(c)
2R
(a) 2 R (b) 4 R
(c) R (d) R/2
50. What is the voltage across resistor A in the following circuit? Each
resistor has a resistance of 2 M and the capacitors have
capacitances of 1 F. The battery voltage is 3V. (d)

53. In the network shown in figure, the equivalent resistance between


the point A and B is :
1 3 1
(a) 3
A B
(b) 2
2 4
(c) 4
(d) 1
(a) 0 V (b) 0.5 V 54. A meter bridge is set up as shown, to determine an unknown
(c) 0.75 V (d) 1.5 V resistance X using a standard 10 ohm resistor. The galvanometer
51. There is an infinite wire grid with cells in the form of equilateral shows null point when tapping-key is at 52 cm mark. The end-
triangles. The resistance of each wire between neighbouring joint corrections are 1 cm and 2 cm respectively for the ends A and B.
connections is r. The equivalent resistance of a whole grid between The determined value of X is
the points A and B as shown in figure is:
(a) 10.2 ohm
r
(a) 10
3 X
(b) 10.6 ohm
r
(b)
2 (c) 10.8 ohm
r A B
(c) (d) 11.1 ohm
4
(d) r

Answer Key 48 (a) 49 (c) 50 (d) 51 (a)


Sol. from page 290 52 (c) 53 (d) 54 (b)
272 ELECTRICITY & MAGNETISM

MCQ Type 2 Exercise 3.2


Multiple correct options 15
1. When no current is passed through a conductor: (a) Equivalent resistance between A and C is .
2
(a) the free electrons do not move
(b ) the average speed of a free electron over a large period of time (b) Equivalent resistance between A and C is 7 .
is zero (c) Equivalent resistance between B and D is 3 .
(c) the average velocity of a free electron over a large period of (d) Equivalent resistance between B and C is 3 .
time is zero 5. The resistance of the galvanometer G in the circuit's 25 . The
(d) the average of the velocities of all the free electrons at an meter deflects full scale for a current of 10 mA. The meter behaves
instant is zero. as an ammeter of three different ranges. The range 0 –10 A, if the
terminals O and P are taken; range 0–1A between O and Q; range
2. A battery of emf E0 5V and internal resistance 5 is connected is 0–0.1 A between O and R, then :
across a long uniform AB of length 1 m and resistance per unit
length 5 m–1. Two cells of E1 = 1 V and E2 = 2V are connected G
as shown in the figure. :
R1 R2 R3
5V 5
+ 10A 1A 0.1 A
O P Q R
P
A B (a) R1 = 0.25 (b) R2 = 0.25
1V 1
(c) R3 = 2.5 (d) R1 = R2 + R3
G 6. AB is a part of another large circuit. Also VQ – VB = 12 V. Then :
2V 2
10V 6V 3V
2 4 2 2 1
(a) The null point is at A A B
i Q
(b) If jockey is touched to point B the current in the galvanometer 1
will be going towards B
(c) When jockey is connected to point A no current is flowing (a) current, i = 3 A (b) current, i = 5 A
through 1 V battery. (c) VA – VB = 12 V (d) VA – VB = 24 V
(d) The null point is at distance of 8/15 m from A 7. In the circuit shown;
3. In the circuit shown, the current in i1
(a) i1 = 2 A
10V 2 2 i3
(b) i2 = 9 A
10 20 30
i
1 (c) i3 = 5 A i2
8
10V
(d) i = 11 A 60V

4 8. A battery of unknown emf is connected across resistances as


shown in figure. The voltage drop across 8 resistor is 20 V.
Then :
(a) 4 resistance is 2.5 A (b) 4 resistance is 0 A
(c) 8 resistance is 2.5 A (d) 8 resistance is 0 A V i A
4. Five resistors are connected as shown in the figure. Then : 8 11
B 11 15
i1 i2
5 10
A B
C 13
A 5 C
(a) The reading of ammeter is 2.5 A.
(b) The reading of ammeter is 0.7 A.
10 5 (c) The emf of the battery is 48 V.
D (d) The emf of the battery is 67.3 V.

Answer Key 1 (c, d) 2 (a, b) 3 (a, d) 4 (b, c)


Sol. from page 296 5 (b, c) 6 (a, d) 7 (b, d) 8 (b, d)
DC AND DC CIRCUITS 273

Statement Questions Exercise 3.3


Read the two statements carefully to mark the correct option out of the options given below:
(a) If both the statements are true and the statement - 2 is the correct explanation of statement - 1.
(b) If both the statements are true but statement - 2 is not the correct explanation of the statement - 1.
(c) If statement - 1 true but statement - 2 is false.
(d) If statement - 1 is false but statement - 2 is true.

1. Statement - 1 9. Statement - 1
There is no current in the metals in the absence of electric field. The resistivity of a semiconductor increases with temperature.
Statement - 2 Statement - 2
Motion of free electrons are random. The atoms of a semiconductor vibrate with smaller amplitude at
2. Statement - 1 higher temperature.
A stream of positively charged particle produces an electric field E 10. Statement - 1
at a centrain distance from it. In the following circuit emf is 2V and internal resistance of the cell
Statement - 2 is 1 and R 1 , then reading of the voltmeter is 1V.
A current currying conductor produces an electric field 2E at the Statement - 2
same distance.
3. Statement - 1
The electric bulb glows immediately when switch is on.
Statement - 2
The drift velocity of electrons in a metallic wire is very high.
4. Statement - 1
The connecting wires are made of copper.
Statement - 2
The electrical conductivity of copper is high.
5. Statement - 1 2
V ir where 2V , i 1A and R 1 .
Electric field outside the conducting wire which carries a constant 2
current is zero. 11. Statement - 1
Statement - 2 Drift speed vd is the average speed between two successive
Net charge on conducting wire is zero. collisions.
6. Statement - 1 Statement - 2
Kirchoff s junction rule follows from conservation of charge. If is the average distance moved between two collision and t is
Statement - 2
Kirchoff s loop rule follows from conservation of momentum. the corresponding time, then vd lim .
t 0 t
7. Statement - 1
12. Statement - 1
Two non-ideal batteries are connected in series. The equivalent
When a current is established in a wire, the free electrons drift in
emf is larger than either of the two.
the direction opposite to the current and so the number of free
Statement - 2 electrons in the wire continuously decrease.
The equivalent emf of the batteries will be average of the two. Statement - 2
8. Statement - 1 Charge is a conserved quantity.
Two non ideal batteries are connected in parallel. The equivalent 13. Statement - 1
emf is smaller than either of the two emfs.
A moving coil ammeter can not measure AC.
Statement - 2
The equivalent internal resistance is smaller than either of the two Statement - 2
internal resistances. AC does not show thermal effect.

Answer Key 1 (a) 2 (c) 3 (c) 4 (a) 5 (a) 6 (c) 7 (c)


Sol. from page 297 8 (d) 9 (d) 10 (a) 11 (c) 12 (d) 13 (c)
274 ELECTRICITY & MAGNETISM
14. Statement - 1 15. Statement - 1
In the figure the resistors are in parallel because they are along In a meter bridge experiment, null point for an unknown resistance
parallel lines. is measured. Now, the unknown resistance is put inside an enclosure
Statement - 2 maintained at a higher temperature. The null point can be obtained
The resistors are in series because current is not divided between at the same point as before by decreasing the value of the standard
then. resistance.
Statement - 2
Resistance of a metal increases with increase in temperature.

Answer Key 14 (d) 15 (d)


Sol. from page 297

Passage & Matrix E xercise 3. 4

3. The current density


Passage for Q.1 to Q.3
2 2e 4 2e
Two large parallel plates are located in vacuum. One of them serves as a (a) 0 a 3/ 2 (b) 0 a 3/ 2
cathode, a source of electrons whose initial velocity is negligible. An 3 m 9 m
electron flow directed toward the opposite plate produces a space charge
2 e
(c) a 3/ 2 (d) none of these
causing the potential in the gap between the plates to vary as V a x4 / 3 , 3
0
m
where a is a positive constant, and x is the distance from the cathode.
Passage for Q. 4 to Q.6
The gap between the plates of a parallel plate capacitor is filled up with
two dielectric layers 1 and 2 with thicknesses d1 and d2, permittivities
1 and 2 , and resistivities 1 and 2. A DC voltage V is applied to the
capacitor, with electric field directed from layer 1 to layer 2.

1. The electric field between the plate at a function of x is :


(a) E a x1/ 3 (b) E a x2 / 3

4a 1/ 3
(c) E x (d) none of these
3
4. The electric field E1 in dielectric layer 1 is :
2. The volume density of space charge as a functions of x
V d1V
4 1/ 3 (a) E1 (b) E1
(a) a x /3 (b) ax d1 d1 d 2
0 9
4 4 1V 1V
(c) a 0 x2 / 3 (d) a 0 x4 / 3 (c) E1 (d) E1
9 9 1d1 2d 2 d1 2

Answer Key 1 (c) 2 (c) 3 (b)


Sol. from page 297 4 (c)
DC AND DC CIRCUITS 275
5. The electric field E2 in dielectric layer 2 is :
Passage for Q. 10 to Q. 12
V d 2V
(a) E2 (b) E2 In the circuit shown in figure E, F, G and H are cells of emf 2, 1, 3 and 1
d2 d1 d 2 volt and their internal resistances are 2, 1, 3 and 1 ohm respectively.

2 V 2V
(c) E2 (d) E2
1 d2 1d1 2 d2

6. The surface density of extraneous charge at the boundary between


the dielectric layers to be zero, the condition is :
(a) 1 1 2 2 (b) 1 2 2 1

1 2
(c) 1 2 (d) 1
1 2

Passage for Q. 7 to Q. 9
A capacitor C is uncharged and capacitor C0 has a charge Q0. These are
connected in series with a source of emf in the circuit shown in figure.
The switch S is closed at t = 0. 10. If i is the current in the 2 resistor, then

5 6
(a) i A (b) i A
13 13

1 1
(c) i A (d) i A
13 26
11. The potential difference across 4 resistor is :

1 2
(a) V (b) V
7. If i0 is the current in resistor R at t = 0, then 13 13
Q0 5 3
(c) V (d) V
C 13 26
(a) i0 (b) i0
R R 12. The potential difference across the cell G is :
(a) 1.61 V (b) 1.46 V
Q0 (c) 1.02 V (d) zero.
C0
(c) i0 (d) zero
R Paragraph for question Q. 13 & Q. 14
8. If i is the instantaneous current, then Electrical resistance of certain materials, known as superconductors,
changes abruptly from a nonzero value to zero as their temperature is
1 1 di C C0 lowered below a critical temperature T C(0). An interesting property of
(a) i R 0 (b) i iR 0
C C0 dt C superconductors is that their critical temperature becomes smaller than
TC (0) if they are placed in a magnetic field, i.e., the critical temperature
1 1 Ri TC (B) is a function of the magnetic field strength B. The dependence of
(c) i 0 (d) none of these
C C0 2 TC (B) on B is shown in the figure.

9. The instantaneous current i in the resistor R is :


t TC (B)
t
RC RC0
(a) i i0e (b) i i0e
TC (0)
t (C C0 )
t
RCC0
(c) i i0e R (C C0 ) (d) i i0e

O B

Answer Key 5 (d) 6 (s) 7 (c) 8 (a)


Sol. from page 297 9 (d) 10 (d) 11 (b) 12 (a)
276 ELECTRICITY & MAGNETISM
13. In the graphs below, the resistance R of a superconductor is shown 15. V measured between B and C is
as a function of its temperature T for two different magnetic fields
B1 (solid line) and B2 (dashed line). If B2 is larger than B1 which of I I I I
(a) – (b) –
the following graphs shows the correct variation of R with T in a ( a b) a ( a b)
these fields?
I I I
(c) – (d)
2 a 2 ( a b) 2 ( a b)
16. For current entering at A, the electric field at a distance ‘r’ from A
(a) (b) is

O I I
(a) (b)
8 r2 r2

I I
(c) (d)
2 r2 4 r2
(c) (d)
Passage for Q. 17 to Q. 19
In the circuit shown AB is a 10 uniform slide wire 50 cm long. 1 is 2 V
14. A superconductor has TC (0) = 100 K. When a magnetic field of accumulator of negligible internal resistance. R1 and R2 are 15 and 5
7.5 Tesla is applied, its TC decreases to 75 K. For this material one respectively. When K1 and K2 are both open, the galvanometer shows no
can definitely say that when deflection when AJ = 31.25 cm when K1 and K2 are both closed the
(a) B = 5 Tesla, TC (B) = 80 K balance length AJ = 5 cm.
(b) B = 5 Tesla, 75 K < TC (B) < 100 K
(c) B = 10 Tesla, 75K < TC (B) < 100 K
(d) B = 10 Tesla, TC (B) = 70K K1
1

Paragraph for question Q. 15 & Q. 16


Consider a block of conducting material of resistivity shown in the J
A B
figure. Current I enters at A and leaves from D. We apply superposition
principle to find voltage V developed between B and C. The calculation 2
G
is done in the following steps:
K2
(i) Take current I entering from A and assume it to spread over a
hemispherical surface in the block.
(ii) Calculate field E(r) at distance r from A by using Ohm’s law E =
j, where j is the current per unit area at r. 17. The emf of the cell 2 is
(iii) From the r dependence of E(r), obtain the potential V(r) at r. (a) 0.5 V (b) 1V
(iv) Repeat (i), (ii) and (iii) for current I leaving D and superpose
(c) 1.5 V (d) 2V
results for A and D.
18. The internal resistance of the cell 2.
I V I (a) 7.5 (b) 8
(c) 10 (d) 2
19. The balance length AJ when K2 is open and K1 is closed
a b a
(a) 12.5 cm (b) 13.5 cm
A B C D
(c) 14.5 cm (d) 15.5 cm

Answer Key 13 (a) 14 (b) 15 (a) 16 (c)


Sol. from page 297 17 (a) 18 (a) 19 (a)
DC AND DC CIRCUITS 277

20. Column I gives certain situations in which a straight metallic wire of resistance R is used and Column II gives some resulting effects.
Column I Column II
A. A charged capacitor is connected to the ends of (p) A constant current flows through the wire
the wire
B. The wire is moved perpendicular to its length with (q) Thermal energy is generated in the wire
a constant velocity in a uniform magnetic field
perpendicular to the plane of motion
C. The wire is placed in a constant electric field that (r) A constant potential difference develops
has a direction along the length of the wire between the ends of the wire
D. A battery of constant emf is connected to the (s) charges of constant magnitude appear at
the ends of the wire ends of the wire.
21. A galvanometer of resistance 99 is converted into an ammeter using a shunt of 1 and connected as shown in figure. The ammeter reads
3A. The same galvanometer is converted into a voltmeter by connecting a resistance of 101 in series. If reading is fond to be 4/5 of the full
scale reading.

12 V r 12 V r
2
A
2
V

Column I Column II
A. Internal resistance of the cell (r) in ohm is (p) 10
B. Range of ammeter in ampere is (q) 0.05
C. Range of voltmeter in volts is (r) 1.01
D. Full scale deflection current of the galvanometer is (s) 5
(t) 11.1
22. Consider a network of resistances each of value of R as shown in figure.

B
R R

A C
R
R R

Column I Column II
A. Equivalent of net work between A and C is (p) same
B. Equivalent resistance between A an B (q) 5/8 R
C. Potential of B and D when voltage source is (r) R
applied across A and C is
D. Potential of B and D when voltage source is (s) 2R
applied across A and B is
(t) different

Answer Key 20 A-q ; B-r ; C-s ; D-p, q, r 21 A-r ; B- q ; C - p ; D -q


Sol. from page 297 22 A-r ; B -(q) ; C - (p) ; D-(t)
278 ELECTRICITY & MAGNETISM
23. Match the following
Column I Column II
A. Electric conductivity of a conductor depends on (p) Dimensions (length and area of cross-section)
B. Conductance of a conductor depend on (q) Temperature
C. For a given conductor and at a given temperature, (r) Nature of conductor
current density depends on
D. For a given potential difference a applied across (s) Electric field strength
a conductor, current in it will depend on (t) None of the above
24. Column I gives some electrical circuits, with points A and B indicated in the circuit. Column II gives possible values of potential difference
between the points VA– VB and VB – VC. Match appropriately.
Column I Column II
A. Current I in branch BD does not change, even (p) VA – VB = 0
if key K is closed

C k D

B
B. The circuit is in transient state (q) VA – VB = positive
A C B

C. Dielectric constant of dielectric is varying (r) VA – VB = negative


with y
y
+Q –Q

B C
(capacitor with dielectric)
A x

D. Space between capacitor plates is filled with (s) VB – VC = 0


three dielectric slabs of different dielectric
constants (k1 k2 k3)

+Q –Q

k1 A k2

k3 B C

(t) VB – VC = positive

Answer Key 23 A-q, r ; B-p, q, r ; C-s ; D-p, q, r 24 A-p ; B- p, q, r, t ; C - p, t ; D -p, q, r, t


Sol. from page 297
DC AND DC CIRCUITS 279
25. Column I gives some electrical circuits in steady state. Column II gives some statements regarding the circuits.
Match appropriately.
Column I Column II

V R
5 2.5V
A. Y (p) Reading of voltmeter is 2 V

A 20
2µF
X

C = 2µF
X A Y

B. (q) Reading of ammeter is 0.1 A


R
V

2V

C = 2µF
X V A
R

3V 1V 10
C. (r) Current through R is zero

2V 0V 0V 2V

20
20 2V
2V
2V
V 20

D. 2V 2µF (s) Charge on capacitor is 2µC


20
2V
X 20
A Y
2V
20 2V
20

(t) Point marked Y is at lower potential relative to point marked X

Answer Key 25 A-p, q, r, t ; B-p, r, t ; C-p, q, r, s ; D- q, t


Sol. from page 297
280 ELECTRICITY & MAGNETISM

Subjective Integer Type Exercise 3.5


Solution from page 299
1. The current through a wire depends on time as i = (20 + 4t). Find 6. Find the current supplied by the battery in the circuit shown in
the charge crossed through a section of the wire in 10 second. figure.
Ans. 400 C 8
2. The switch S shown in figure is kept closed for a long time and is
then opened at t = 0. Find the current in the middle 10 resistor
at t = 1.0 ms.
25 F +
24V

10 S
12 V
+ – 10 12
Ans. 11 mA. Ans. 5 A.
3. In the circuit shown in figure, find the value of unknown resistor 7. Find the effective resistance between the points A and B in figure.
R. 3
E D
3

3
5A
F 6 C

6
8A R 6
– 3A 3

3
20 40V 10
A 3 B
+ Ans. 2

1
+ 100V 8. The ammeter current in the circuit shown in figure is . Find
x
Ans. 4 . the value of x.
4. What are the values of V1 and V2 in the circuit shown in figure ?
2V
+ –
2
6 4V – 10
+ A
V1
+
24V 4 10

Ans. x = 7
9. Find the potential difference V1 – V2 between points 1 and 2 of
the circuit shown in figure if R1 = 10 , R2 = 20 , 1 = 5.0 V and
2
2 = 2.0 V. The internal resistances of the current sources are
negligible.
V2
R1 1
Ans. V1 = 8V, V2 = – 4V. - +
5. Compute the value of battery current in the circuit shown in
figure. All the resistances are in ohm. 1 2
6 R2
2
- +
i 4
( 1 2 ) R1
+ Ans. V1 V2 1 4V .
4 12V ( R1 R2 )
– 10. When two identical batteries of internal resistance 1 each are
8 connected in series across a resistor R, the rate of heat produced in
R is J1. When the same batteries are connected in parallel across R,
2
the rate is J2. If J1 = 2.25 J2 then the value of R in is
Ans. 4
Ans. 6A.
DC AND DC CIRCUITS 281

Subjective Exercise 3.6


Solution from page 301

1. The current in a conductor and the potential difference across its 5. Find the currents through the three resistors shown in figure.
ends are measured by an ammeter and a voltmeter. The meters
4
draw negligible currents. The ammeter is accurate but the voltmeter
has a zero error (that is, it does not read zero when no potential
difference is applied). Caculate the zero error if the readings for
two different conditions are 1.75 A, 14.4 V and 2.75 A, 22.4 V.
Ans. 0.4 V.
2. Determine the values of currents i1 and i2 in the circuit shown in 4 6
figure.
4V 2V
+ – – +
i2
Ans. Zero in the upper 4 resistor and 0.2 A in the rest two.
6. Find the current measured by the ammeter in the circuit shown in
3A 5A figure.
i1 1A 10 10 10

8A 4A 50 50
10 10 10
Ans. i1 = – 6A, i2 = 9A.
3. Figure shows an arrangement to measure the emf and internal
6V A
resistance r of a battery. The voltmeter has a very h e i g h + –
resistance and the ammeter also has some resistance. The voltmeter
Ans. 0.4 A.
reads 1.52 V when the switch S is open. When the switch is closed
7. Find the current in each branch of the circuit shown in figure.
the voltmeter reading drops to 1.45 V and the ammeter reads 1.0 A.
Find the emf and the internal resistance of the battery.

V
1 2
r 1
+ –
S
A 2 1

Ans. 1.52 V, 0.07 .


4. The resistance of the rheostat shown in figure is 30 . Neglecting 14V
+ –
the meter resistance, find the minimum and maximum currents
through the ammeter as the rheostat is varied.
Ans. 6A, 4A, 2A, 4A, 6A (top to bottom).
5.5 V 8. Find potential difference Vxy in the circuit shown in figure.
+ –
2 1 4V
A X + –
10
– 2V 3 3 5
+

30 +

4V
20
Y

Ans. 0.15 A, 0.83 A. Ans. 3.7 V.


282 ELECTRICITY & MAGNETISM
9. Two capacitors are charged in series by a 12 V battery (see figure). 13. Find the equivalent resistance of the resistors network across points
(a) What is the time constant of the charging circuit ? A and B.
(b) After being closed for the length of the time determined in (a)
the switch S is opened. What is the voltage across the 6 F 2 15
A
capacitor ?
B
10
a 8
S
3 F

+ 10
5 20
– 12V,1 30 40
6 F

b Ans. 22.5 .
14. Consider an infinite ladder network shown in figure. A voltage is
Ans. (a) 12 s (b) 2.53 V. applied between points A and B. If the voltage is halved after each
10. Calculate the equivalent resistance between A and B of the circuit section, find the ratio R1/R2. Suggest a method to terminate it after
shown in figure. The value of resistances is in ohm. a few sections without introducing much error with attention.

R1 R1 R1 R1 R1
A
4 3 4
R2 R2 R2 R2 R2
3

2 5 2
3

B
5
A B Ans. 1/2.
15
15. Determine the current through the battery in the circuit shown in
6 15 9 6
figure :
9

(a) Immediately after the key K is closed and


12 9 12 (b) in a long time interval, assuming that the parameters of the
circuit are known.
Ans. 6.75 .
K
11. At what value of the resistance Rx in the circuit shown in figure
+ –
will the total resistance between points A and B be independent of
the number of cells ?
C1
2R 2R 2R 2R
A R1 A C2
B
R2
R R R R Rx
R3
B

Ans. Rx = R ( 3 – 1).
Ans. (a) (b) .
12. Figure shows an infinite circuit formed by the repetition of the R ( R1 R3 )
same link, consisting of resistance R1 = 4.0 and R2 = 3.0 . Find
16. A 5.0 F capacitor having a charge of 20 C is discharged through
the resistance of this circuit between points A and B.
a wire of resistance 5.0 . Find the heat dissipated in the wire
R1
between 25 to 50 s after the connections are made.
R1 R1 R1
A Ans. 4.7 J.
17. A capacitor of capacitance 8.0 F is connected to a battery of emf
R2 R2 R2 6.0 V through a resistance of 24 . Find the current in the circuit.
(a) just after the connections are made and
B (b) one time constant after the connections are made.
Ans. (a) 0.25 A (b) 0.09 A.
4 R2 R1 18. How many time constants will elapse before the power delivered
Ans. R 1 1 6 . by the battery drops to half of its maximum value in an RC circuit ?
R1 2
Ans. 0.69.
DC AND DC CIRCUITS 283
19. The circuit shown in figure is made of a homogeneous wire of 23. Find the current in the 2 resistor shown in figure.
constant cross section. Find the ratio Q12 / Q34 of the amounts of
heat liberated per unit time in conductors 1-2 and 3-4.
A

3 4
12V

+
6


3 4
4

6V

2

+
1 2
B + – C
3 6V
- +
Ans. 1.85 A.
32. When the circuit of figure is in steady state, what would be the
Q12
Ans. 11 6 2 . p.d. across the capacitor ? Also find the discharge current at the
Q34 instant S is opened.
20. In figure circuit section AB absorbs energy at a rate of 50 W when
a current i = 1.0 A passes through it in the indicated direction.
8
4
i
X
A R=2.0 B
1 F
(a) What is the potential difference between A and B ?
4
(b) Emf device X does not have internal resistance. What is its 2
emf ?
(c) What is its polarity (the orientation of its positive and negative S 24V
terminal) ? + –

Ans. (a) 50 V , (b) 48 V, (c) B is connected to negative terminal.


21. In the given circuit =3 =2 = 6V Ans. 8V, 1.8 A.
1 2 3
24. A varying voltage is applied to the clamps AB such that the voltage
R1 = 2 R4 = 6 , R3 = 2 R2 = 4 and C = 5 F
across the capacitor plates varies as shown in figure.
Find the current in R3 and energy stored in C.
C
R1 1 A V
- + C
C
2 R2 R3
+ - R

3 R4
- + B t
t0 2t0 3t0 4t0 5t0
D
Ans. 1.5 A, 4.0 J.
22. Find the currents i1, i2 and i3 in the circuit shown in figure. Plot the time dependence of voltage across the terminals C and D.

10 V

i3
10 10 10

+
i1
+

100V 10 i2
– – 50V Ans.
t
t0 2t0 3t0 4t0 5t0

Ans. i1 = 3.75 A, i2 = 0, i3 = 1.25 A.


284 ELECTRICITY & MAGNETISM
25. A thin uniform wire AB of length 1 m, an unknown resistance x and 29. In the circuit shown in figure the emf of the source is equal to =
a resistance of 12 are connected by thick conducting strips, as 5.0 V and the resistances are equal to R1 = 4.0 and R2 = 6.0 .
shown in figure. A battery and a galvanometer (with a sliding The internal resistance of the source equals R = 0.10 . Find the
jockey connected to it) are also available. Connections are to be currents flowing through the resistances R1 and R2.
made to measure the unknown resistance x using the principle of
Wheatstone bridge. Answering the following questions :

+
– R R1 R2

x 12

A B C D R2 I1R1
Ans. I1 I .2 A, I 2 0.8 A.
(a) Are there positive and negative terminals on the galvanometer ? ( RR1 R1R2 R2 R) R2
(b) Copy the figure in your answer book and show the battery 30. In the circuit shown in figure the sources have emf’s 1 = 1.0 V and
and the galvanometer (with jockey) connected at appropriate
2 = 2.5 V and the resistances have the values R1 = 10 and R2 =
points. 20 . The internal resistances of the sources are negligible. Find a
(c) After appropriate connections are made, it is found that no potential difference VA – VB between the plates A and B of the
deflection takes place in the galvanometer when the sliding capacitor C.
jockey touches the wire at a distance of 60 cm from A. Obtain
the value of the resistance of x. R1
1
Ans. (a) There are no positive and negative terminals on the – +
galvanometer because only zero deflection is needed in it.
C
(b) The battery and galvanometer with jockey connections are 1
– +
shown in figure. A B
2 R2
– +
Battery
+ – (
x C 12 1 2 ) R1
Ans. VA VB 0.5V .
A Jockey J B D ( R1 R2 )
31. N sources of current with different emf’s are connected as shown
G
in figure. The emf’s of the sources are proportional to their internal
Galvanometer
resistances, i.e. = R, where is an assigned constant. The lead
wire resistance is negligible. Find
(c) 8 .
(a) the current in the circuit;
26. A long cylinder with uniformly charged surface and cross-sectional
(b) the potential difference between points A and B dividing the
radius a = 1.0 cm moves with a constant velocity v = 10 m/s along
circuit in n and N – n links.
its axis. An electric field strength at the surface of the cylinder is
equal to E = 0.9 kV/cm. Find the resulting convection current, that
is, the current caused by mechanical transfer of a charge. A
Ans. I = 2 0aEv = 0.5 A.
27. The gap between the plates of a parallel plate capacitor is filled
with glass of resistivity = 100 G .m. The
capacitance of the capacitor equals C = 4.0 nF. Find the leakage
current of the capacitor when a voltage V = 2.0 kV is applied
B

to it.
VC
Ans. I 1.5 A.
0

28. Two sources of current of equal emf are connected in series and Ans. (a) I = ; (b) VA – VB = 0.
have different internal resistances R1 and R2 (R2 > R1). Find the 32. Figure illustrates a potentiometric circuit by means of which we
external resistance R at which the potential difference across the can vary a voltage V applied to a certain device possessing a
terminals of one of the sources (which one in particular?) becomes resistance R. The potentiometer has a length l and a resistance R0,
equal to zero. and voltage V0 is applied to its terminals find the voltage V fed to
Ans. R = R2 – R1, V = 0 in the source of current with internal the device as a function of distance x. Analyse separately the case
resistance R2. R >> R0.
DC AND DC CIRCUITS 285
35. Find the emf and the internal resistance of a source which is
V0 equivalent to two batteries connected in parallel whose emf’s are
equal to 1 and 2 and internal resistances to R1 and R2.
R0
( 1R2 2 R1 ) R1 R2
Ans. , R1 .
x ( R1 R2 ) ( R1 R2 )
R
36. Find the current flowing through the resistance R in the circuit
shown in figure. The internal resistances of the batteries are
V0 Rx V0 x negligible.
Ans. V x
; for R R0 , V .
l
Rl + R 0 (l x) R2
l
33. A 6 volt battery of negligible internal resistance is connected across
a uniform wire AB of length 100 cm. The positive terminal of

+
0 –
another battery of emf 4 V and internal resistance 1 is joined to R1 R3 –

+
the point A as shown in figure. Take the potential at B to be zero.
R
(a) What are the potentials at the points A and C ?
(b) At which point D of the wire AB, the potential
is equal to the potential at C ? [ ( R2 R3 ) 0 R3 ]
Ans. I
(c) If the points C and D are connected by a wire, [ R ( R2 R3 ) R2 R3 ]

37. In the circuit shown in figure the sources have emf’s 1 = 1.5 V, 2
6V
+ – = 2.0 V, 3 = 2.5 V, and the resistances are equal to R1 = 10 , R2 =
20 , R 3 = 30 . The internal resistances of the sources are
negligible. Find :
A B (a) The current flowing through the resistance R1;
D C
(b) A potential difference VA – VB between the points A and B.
1
+ – A
4V

+ –
what will be the current through it ? 1–
+ 3
R2
(d) If the 4 V battery is replaced by 7.5 V battery,
what would be the answers of parts (a) and (b) ?
R1 + R3
– 2
Ans. (a) 6 V, 2 V (b) AD = 66.7 cm (c) zero (d) 6V, – 1.5 V, no
such point D exists
34. Find the magnitude and direction of the current flowing through B
the resistance R in the circuit shown in figure, if the emf’s of the
sources are equal to 1 = 1.5 V and 2 = 3.7 V and the resistances
are equal to R1 = 10 , R2 = 20 , R = 5.0 . The internal [ R3 ( 1 2) R2 ( 1 3 )]
Ans. (a) I1 0.06 A,
resistances of the sources are negligible. ( R1 R2 R2 R3 R3 R1 )

R2 (b) VA – VB = 1 – I1R1 = 0.9 V.


2
+ – 38. Find how the voltage across the capacitor C varies with time t in
figure, after the shorting of the switch Sw at the moment t = 0.
R1
– + SW
1

R
+

R C

( R1 2 R2 1 ) R
Ans. I 0.02 A, the current is
( RR1 R1 R2 R2 R )
directed from the left to the right. 1 2t / RC
Ans. V 1 e .
2
286 ELECTRICITY & MAGNETISM
39. Find a potential difference VA – VB between the plates of a capacitor 40. A constant voltage V = 25 V is maintained between points A and B
C in the circuit shown in figure, if the sources have emf’s 1 = 4.0 of the circuit. Find the magnitude and direction of the current
V and 2 = 1.0 V and the resistances are equal to R1 = 10 , R2 = 20 flowing through the segment CD if the resistances are equal to R1
and R3 = 30 . The internal resistances of the sources are = 1.0 , R2 = 2.0 , R3 = 3.0 and R4 = 4.0 .
negligible.
R1 R2
C
R3 C
A B A B
R3 R4
+

2
D
R2 R1

V R1 R2
Ans. I 1 1.0 A. The current
R2 R 1 R2 R4 ( R1 R3 )
1 1
– + R1R3 ( R2 R4 )

flows from point C to point D.

Ans. VA 2 R3 ( R1 R2 ) 1 R1 ( R2 R3 )]
VB 1.0V .
( R1 R2 R2 R3 R3 R1 )
DC AND DC CIRCUITS 287

Solutions Exercise 3.1Level -1


1. (d) Capacitor offers infinite resistance to dc and so, 4
r
1 1 1 12. (d) R R R (2)4 16 R.
r
RAB 20 13. (a)
or RAB = 20 k 14. (d) The resistance of semiconductors increases with decrease in
2. (d) The effective circuit is shown in figure temperature while that of metals increase.
R4 15. (a) Effective circuit for Ohm's law is shown in figure.
A
A
R1R4 R1
R AB
R1 R4
R V
B
3. (c) Time constant, = C(2 R) = 2 CR

R
16. (a) The equivalent resistance of the circuit across the cell is
C
6 18
R 0.5 8 2 15
R 6 18

4. (c) V 15
Now current, i 1A.
5. (d) The direction of i and j are opposite of the direction of R 15
17. (d) VA – VB = (V0 – VB) – (V0 – VA)
motion of electron, and the direction of electrons are opposite
=1×2–1×4
to that of E . = – 2 V.
6. (d) (A) i = 7 C/s (B) i = 4 – 3 = 1 C/s
(C) i = –2 – 5 = –7 C/s (D) i = –3 + 1 = –2 C/s 1A 4 A 2
It shows, (A = C) > D > B.
7. (a) RAB = 2 + 2 + 2 + 2 + 2 = 10 . 2A
L (1.5L) 3 L ( L 2) L O
8. (d) i1 ; i2 ; i3 .
A A2 A A2 A 1A
L 2 B 4
9. (b) As R , and so it is least along top-bottom. Current will 18. (c) Diode in forward bias offers zero resistance.
A
be maximum. 10 10
R 10 15 k .
R 100 10 10
10. (a) ....... (i)
2 30
Total current i 2 mA. The current in 10 k resistance
R ( 20) 15
and ....... (ii) is 1 mA. Thus VA – VB = (1 × 10–3) × 10 × 103 = 10 V.
2 (80 )
19. (a) The current through ammeter
On solving above equations, we get
i = 5 A.
R=3 .
R 2
1A 1A 1A 1A 1A

100 –
5A
A A
20. (a) R = R1 + R2
1
( 1 2) 1 1 2 2
1 3 3 B or
11. (c) RAB . A A A
1 3 4 3
1 1 2 2
1 2
288 ELECTRICITY & MAGNETISM
21. (a) The effective circuit is shown in figure. 750
Now, i 1.5 A.
30 60 R 512.50
RAB
30 60 750 V
= 20 .

5 10 15 i

A B 512.5
28. (c) 2 is in open circuit, so there is no current in it.
2
29. (a) Current, i 0.1A.
10 20 30 5 15
P.d. across potentiometer wire, V = i R = 0.1 × 5 = 0.5 V.
22. (a) The effective circuit is shown in figure.
0.5
9 9 9 Now, potential gradient 0.005 V cm.
RAB . 100
9 9 2
30. (c) If R' is the resistance of ammeter, then
4 (R + R' ) = 20
3 3 3 R = 5 – R'.
R 20
A B 31. (c) R 20 .
80 80
32. (a) In balanced bridge, IP = IQ and IR = IG.
33. (c) See theory of the chapter.
3 3 3 x 20
34. (a) y 4 x.
y 80
5R If be the distance of new balance point, then
23. (a) The least value of two resistance in series is .
6 4x
P
5R y (100 )
R = 50 cm.
RPQ 6
5R R R/3 SG
R 35. (a) ig i
6 S G
5R 0.1 100 6
. Q R or 100 10 i
11 R/2 100 0.1
24. (c) The equivalent resistance across the cell is, R = 1.5 . or i = 100.1 × 10–3 A.
36. (a) The p.d. across the potentiometer wire
6 V = 0.2 × 10–3 × 100 = 0.02 V.
Current, i 4 A.
1.5 So, we can write,
3 2 0.02
490 R R
R = 4.9
2
6V 37. (c)
5R
Req .
6 6
3 48
1.5 38. (a) Current in 8 resistor, i1 6 , and the current in 24
8

25. (b) As V = – i r 48
resistor, i2 2 A. So, total current = 8 A.
24
V V R( V) Now, Vxy = iR xy = 8 × (3 + 6 + 10 + 1) = 160 V.
r .
i (V R) V 39. (c) VA – VB = 1 × 1.5 = 1.5, VA = 1.5 V
Also VB – VD = –1 × 2.5 + 2
V 2 VD = 0.5 V.
26. (d) = 2.2 V. Also i 0.5 .
R 4 t2 3
Using, V = – i r
40. (b) q i dt (2t 3t 2 )dt
or 2 = 2.2 – 0.5 r
t1 2
r = 0.4
27. (a) The equivalent circuit is shown in figure. 3
t2 t3
5000 0.25 2 3 (32 33 ) (22 23 )
5000 0.15 750V , R 500 512.5 . 2 3 2
100
= 24 C
DC AND DC CIRCUITS 289
41. (a) i = i1 + i2 50. (b)
Vo 0 20 Vo 5 Vo 51. (d) In balance bridge there is no current in galvanometer arm
or (here k arm), and so reading of G remains same.
2 2 4
or Vo = 9 V 52. (c) In series

9 R R1 R2 R3
Thus, i 4.5 A
2 3
or =
i1 i2 A 1A 2A 3A
o
2 4 3 1 1 1
i =
1 2 3
2
3 1 2 3
or =
1 2 2 3 1 3
53. (d) At steady state, there is no current in capacitor arm. So
42. (d) R1 = 5 R and R2 = 3 R equivalent resistance through battery
R2 3R 1 1 1 1 C
0.6 ; R
R1 5R R 1 2 3 11
nV V V 6
43. (b) , R0 (n 1) G. Current i = 11A .
G R0 G 2 R 6 /11
p. d. across capacitor = 6V
12 5
44. (d) i 1A Charge on capacitor = CV = 0.5 × 6 = 3µC.
7
V1 = –5 – 2 × 1 – 2 × 1 1 50
2 54. (b) Current i 2A
= –9 V 25
i
5V 12V Now VA VB i 12 2 12 24 V
V1 As VB 0 , VA 24 V .
45. (b) V = iR2 = 0.5 × 15 = 7.5 V 2
55. (b) The effective circuit is shown in figure.
RAB 1 .
0.5A
0.5A 2 2

2 2
R2=15
A B
2 2

30 2 2
46. (a) emf of the cell, e .
L 100
6
47. (b) i 1.5 A.
2 3 56. (a) R = R1 R2
2.8
2 3 ( 1 2) 1 1 2 2
Current in 2 resistor. or =
A A A
i1 = 0.9 A and i2 = 0.6 A.
2 1 1 2 2
= .
1 2
1.5A
57. (a) The equivalent circuit is shown in figure. So RAB = 1
3
A
R1 R3 2 2
48. (c) or R1R4 R2 R3
R2 R4 2
2 2 2
R1 1 r 1
49. (a) As , so, balanced distance does not
R2 2 r2 2 B
depend on radius of the wire.
290 ELECTRICITY & MAGNETISM
58. (c) The equivalent circuit is shown in figure. So or R1 R2 = 10000
R2 = 9000
2R/3 2R/3
e R
2R 64. (b) E=
A B R Rh r L
RAB =
3
2R/3 2R/3 10 5
= 3 3V .
5 4 1 5
59. (a) In upper close loop, we can write 65. (d) At any temperature,
– (i1 i2 ) R i1r1 0 . or R R1(1 t ) R2 (1 t)
60. (b) Using Kirchoff's loop rule, we have
dR
– 30 I1 + 40 – 40 (I1 + I2) = 0 … (i) For R to be constant, 0
dt
and 40( I1 I 2 ) – 40 – 80 40I 2 0 … (ii)
After solving, we get, I1 = – 0.9 A. or R1 = R2

V1r2 V2r1 18 1 12 2 R1
61. (c) V 14 V
r1 r2 1 2 R2 = .

R 80 66. (d) E1 1.5 1 V , E2 1.5 2V ,.........


62. (a) We have =
55 20 E1 E2 ...... En
Now current I =
R 220 . r1 r2 ...... rn
63. (d) ig (G R1 ) = 1 1.5(1 2 ...... N )
=
or 10 3 (50 (1 2 ...... N )
R1) = 1
= 1.5 A
R1 = 950
Now ig (G R1 R2 ) = 10

Solutions Exercise3.1 Level -2


1. (b) Time constant = CR.
R
A
R
C
R R
R
R
R
B
2. (b) The equivalent circuit is shown in figure. 5. (d) We know that, Rt = R0(1 + t).
66 or 1 = R0(1 + 0.00125 × 300)
Thus, R AB . and 2 = R0(1 + 0.00125 × t)
45
After simplifying, we get t = 1127 K.
6. (b) By Kirchoff's law, we have
2/3
+12 – 500 i + X i = 0
Also –2 + X i = 0
After simplifying above equations, we get
X = 100 .

i 500
G
A B i
3. (d) It is the balance bridge and so, 3R resistor in the circuit is X
ineffective. So, current in this resistance is zero. 12V 2V
2R 4 R 4
R AB R.
2R 4 R 3
4. (c) The effective circuit is shown in figure. Thus 2
7. (d) The current in the circuit, i
R R1 R2 R
RAB .
2
DC AND DC CIRCUITS 291
P.d. across source of internal resistance R2,
V = – i R2 2R 2R

or 2
0 R2
R1 R2 R Q
P
or R = (R2 – R1). r r
, R1 , R2

2R 2R
14. (b)
i R 15. (d) = 6 V,
V 4
Current, i 0.4 A
1 2 R 10
8. (a) Internal resistance, r R
2 Now, V = – ir
or 4 = 6 – 0.4 r
55 50 or r=5 .
10 1
50 10
9. (d) V = ig (G + R)
and V' = i g (G + 2 R)
It shows, V < V' < 2 V i
SG
10. (b) ig i 6V
S G
16. (d) In close loop BCEDB,
S – 4 (i – i' ) + 10 – 12 i + 8 = 0
or 6 ....... (i)
50 10 i and in close loop ABDFA,
S 100
– 4 (i – i' ) + 10 + 8(i) – 6 = 0
Also V = ig (G + R0) On solving above equations, we get
or V = 50 × 10–6(100 + R0) ....... (ii) i = 1.14 A.
On simplifying above equations, we get
V = 10 V, R0 = 200 × 10–3 i B
A C
2V V V
11. (c) Current, i (i – i ) 4 8V
2 R R 3R 6V
Thus p.d. across capacitor, 10V i
V V
VC iR R . F D 12 E
3R 3 8

R 50 15
V 17. (c) i 7 10 3 A
i 5000
i Va iR 7 10 3
1200 8.4 V
V C
3
P Vi 15 7 10 105 mW .
2V 2R 800
i
15V
+
V 2 V 50 V
12. (b)
R
t mC T –
(3 )2 1200 3000
or t mC T ....... (i)
R 18. (d) The equivalent circuit is shown in figure.
( N )2 6 4 7 3
and t 2mC T ....... (ii) R 4.5
2R 6 4 7 3
On solving above equations, we get
N=6 6 7
13. (a) The effective circuit is shown in figure.
2 Rr
RPQ 4 3
R r
292 ELECTRICITY & MAGNETISM
19. (c) – 4 × 2 + 3 – 2 × 2 + 1 × 2 + (VG – VH) = 0
or VG – VH = 7 V. q 12 q22
Given,
20. (d) The effective circuit is shown in figure 2C1 2C2
3
RPQ . C1 q12
2
C2 q22
1 1 1
P Q 2
2C1
=
1 5 / 3C2
1
C1 25
1 or C2 36
21. (d) The effective resistance of the circuit, 23. (a) The final charge
200 30 q0 = Vav × C
R 100
200 30 3 2
= 126 k = C
2
300 = 2.5 C.
Thus, i 2.38 mA.
126 24. (a) The effective circuit is shown in figure.
Now, reading of voltmeter, 3R
RAB
V 2.38 10 3 3
(26 10 ) 61.88 V 8
A
i
R R R
100k R
300V B
R
b
25. (d) The equivalent resistance,

200k V 30k 24
R .
9
24
Thus current, i 9 A.
24 9
C2 8
B 2R i G C
22. (a)
q2 6
24V 12
R C1
i
A H D
q1
2V
F E
26. (c) – +
At steady state the current,

i
3R
In close loop A H D E F A,
q1
0 R V
C1
The initial potential difference between plates of capacitor
or q1 = 2C1 = 3V – V = 2V
Now in close loop BGCEFB
2V V
q2 The current, i =
2R i– 0 R
C2
3V
q2 =
or 2R 0 R
3R C2 The rate of heat energy produced
5C2 2
q2 = 3V 9V 2
3 = i2R = ×R=
R R
DC AND DC CIRCUITS 293
27. (d) Effective emf of two cells in parallel 32. (c) R = R1 + R2
2 r1 1r2 3 2 2 1 4 l
= V = 1 1t 1 2t
r1 r2 = 2 1 3 A
1 2

X terminal of the connection is positive. For neutral point, dR


positive terminal of 0 must be connected to X. For, 0
dt
28. (a) The equivalent circuit is shown in figure.
or 1 1 + 2 2 = 0
1 A 1
V F l 0 2t l0 1 1t
+ 7.5 V 33. (b) R A A0 1 2 1t
10 V = R0 (1 + 2t) (1 + 1t) (1 – 2 1t)
1 1 dR
B C Now R dt = ( 2 – 1)
0
2 2 5V [Neglecting product and higher powers of 1 and 2]

5V A i1 =
2R
34. (c) (i)
5V D 5V E R A

In close loop ABCDEFA,


– 10 – 5 + 5 + V = 0 R R
or V = 10 volt 2
29. (b) The effective circuit is shown in figure.
i
20 Vo Vo 10
r 2r i2
3 3
R A (ii)
r 2r
3 3
20 V – 10 V
A,E,C O R R
V0 = 10 Volt 2
30. (b) If each resistor is r, then
R0 In first case, i1
2R
R0 = 3r or r =
3
3
r In second case, i
r 3 4R
r
A B
When keys are closed, then i2
2R
r
Req 7 35. (c) Initial charge on the capacitor Q0 = C × 3V = 3CV
3
q
7 R0 + i × 2R + V = 0 q 2R
= C B
9 C
31. (b) The equivalent circuit is shown in figure. q
or 2Ri = V i
R C
R
dq q
A R B R or 2R = V V
dt C
R R q
dq t dt
or = 0
q 2R
Q0 V
C
R R
2 A R B 2 q
q
n V
R C Qo t
1 2R
2R C
Req =
3
294 ELECTRICITY & MAGNETISM
39. (a) C1V1 = C2V2
q
q t C1 V2
n V
C Qo 2CR C2 V1

q Q0 t 5i 5
or n V n V
C C 2CR 3i 3

q t C1
or i 3
n V n 2V
C 2CR

q t 2
or n 2V n V i
C 2CR i
1 C2
2V t
n
q 2CR R
V 2V
C
40. (a) The effective circuit is shown in figure.
t 0.8 2A
2V
or e 2CR
q
V 10 V
C i1
A B
2 5
t
q
or 2V = V e 2CR i2
C
3 7.5
t R = 0.8 + RAB
or q = CV – 2CVe 2CR = 0.8 + 4.2 = 5
36. (a) At t = 0, capacitors offer zero resistance for dc and so i1 = 1.2A, i2 = 0.8A
effective resistance of the circuit is R = 2 Now, VAB = i1 × 7
= 1.2 × 7 = 8.4 V
6
i= = 3A. 2 2
2 41. (d) 1 9
r 1 r 9
1 r=3
37. (a) The charge of the capacitor becomes of initial charge in
e 42. (a) The effective circuit is shown in figure.
time, Thus, RAB = 5 .
= CR A K
K 0A 1 d 5 5
=
d A
A 5 B
K 0 d
=
5 5
38. (c) In first case, RA RV
43. (b) Total resistance of the circuit
i A V
RA RV 6 3 6 3
i V R 2 6
6 3 6 3
and V = iRV
In second case, V 18
I 3A
R 6
2i = R
RV R 1A
RA 2A 3
RV R 6 1A
A RV 1A
V RRV V 1A
and 2i 2i 2 1A 6 3 2A
2 R RV

3A

After simplifying above equations, we get 18 V


RV 2 44. (a) All the eight resistors are in parallel, and so
RA 1 R
R AB .
8
DC AND DC CIRCUITS 295
45. (b) As VP = VQ, so current in 3 resistor. The effective circuit
is shown in figure. 1 (2r )( r 2)
R 2r r 2 (on solving)
R1 2 (2 2) r
A P
R d/ 2
2V
49. (c) 2R
X
2V
B Q R/2 R/2
R2
2R 2R
RAB = RAX + RBX 4R
RBX = RAB – RAX A B
or R2 = 10 – 5 = 5 R R R R
46. (a) The equivalent circuit is shown in figure. 2R
1
Thus current i A R
4

6 3 R/2 R/2
2
R R R
3 6 A R B
R R R
R
R R
4.5V 50. (d) 1.5 V
47. (d) RAB = 3.88

A A
7.5 4
6 4

3 5 0.53 2.1

2 0.38
5 4 A
B
B
A This is a DC circuit because the battery is the only source of
6 4 A
7.5 4 voltage. Hence, the capacitors behave like open circuits. An
5 equivalent circuit is then two parallel sets of two identical
5.6 series resistors, see figure. The voltage drop across each
3/2 3/5 parallel branch must be the battery voltage of 3V. Since the
4 resistors are identical there is an equal voltage drop of 1.5 V
1 4
1 across each resistor. In particular there is a drop of 1.5 V
B
B across resistor A.
51. (a) When i current enters into the terminal A, by the symmetry
48. (a) Let each half side has resistance r (= d/2)
i
A current will flow from A to B. Similarly when i current
r r 6
r r i
r leaves the terminal B, current also flows from A to B. So
6
r i
r r net current in the resister between A and B becomes .
3
r r
B
i
Now V AB r
3
r r
ir
or i R AB
A r B 3

r R AB
r
r r 3

r
296 ELECTRICITY & MAGNETISM
52. (c) The following points should be considered while making the 54. (b) At null point
circuit :
(i) An ammeter is made by connecting a low resistance R2 in
parallel with the galvanometer G2. X 10
G2

R2
(ii) A voltmeter is made by connecting a high resistance R1 in A B
series with the galvanometer G1. 52 cm 48 cm
R1
G1 X 10
(iii) Voltmeter is connected in parallel with the test resistor RT. 1 2
(iv) Ammeter is connected in series with the test resistor RT.
(v) A variable voltage source V is connected in series with the Here 1 = 52 + end correction = 52 + 1 = 53 cm
test resistor RT .
53. (d) The equivalent circuit is shown in figure. 2 = 48 + end correction = 48 + 2 = 50 cm
RAB = 1 .
X 10
A 3 B 53 50
1 1
6 53
X 10.6
5

Solutions Exercise 3.2


For 1 A range :
1. (c,d) See theory of the chapter.
2. (a,b) The effective emf the two cells in parallel is R1 R2
ig i
1 2 2 1 R1 R2 R3 G
e 0.
1 2
So null point will be at zero distance from A. When jockey is R1 R2 3
or 10 10 1
touched to B, the current flows through 2 V cell towards B. 25
25
3. (a,d) In close loop through cells, the net emf of the close loop is 9
zero and so current in 8 resistor will be zero. In 4 R2 = 0.25 .
resistor, For range 0.1 A :
10 R1 R2 R3
i 2.5 A. 10 10 3
0.1
4 R1 R2 R3 G
5 On solving, R3 = 2.5 .
5 10
4
2 6. (a, d) Going from B to Q,
4. (b, c) A C VQ – VB = 12 = 1i + 3 + 2 i
5 5 i=3A
2 7. (b, d) x 10
RAC = 7 .
y 20
1 1 1 1
Also RBD 3 .
RBD 15 5 15 z 30
i
25
5. (b, c) For 10 A range : R1 R2 R3
9
60V
R1
ig i 60
R1 R2 R3 G x 6 A,
10
3 R1 60
or 10 10 10
25 y 3 A,
25 20
9
60
1 and z 2 A,
R1 . 30
36 Thus, i1 = y + z = 5 A, i2 = x + y = 9 A.
DC AND DC CIRCUITS 297

20 308 1049
8. (b,d) Current in 8 resistor, i 2.5 A. Total circuit resistance, R = 8 11
8 39 39
= iR
11
Thus, i2 2.5 0.7 A.
11 28 1049
2.5 67.3 V .
39
28 11 308
Resistance between A and C .
39 39

Solutions Exercise-3.3
1. (a) Free electrons in metals more randomly. So net flow of charge 9. (d) The resistivity of semiconductor decreases with increase in
through any section of the metal is zero. temperature.
2. (c) The net charge on current carrying conductor is zero, and so V 2
its electric field is also zero. 10. (a) i 1 A, and V = ir = 1 × 1 = 1 V.
r R 1 1
3. (c) The drift velocity of electrons in metals is order of 10–4 m/s.
11. (c) Drift speed is the average speed between two successive
4. (a)
collision.
5. (a) Statement-2 is the explanation of statement-1. 12. (d) The free electron density in any part of the conductor remains
6. (c) Kirchoff 's loop rule follows from conservation of energy. constant.
7. (c) The equivalent emf of two batteries in series, 13. (d)
e = e1 + e2. 14. (d)
8. (d) The equivalent emf of the two batteries in parallel, 15. (d) With increase in temperature, resistance of metal wire
increases, but balance conduction will not change.
e1r2 r2 r1
e . e may be; e1 e e2.
r1 r2

r1r2
Internal resistance, r . This value is smaller than
r1 r2
either of r1 and r2.

Solutions Exercise-3.4
Passage for Q.1 to Q.3 On solving (i) and (ii), we can get E1 and E2.
6. (a). At the boundary between dielectrics, the surface density of
dV 4ax1/ 3 extraneous charge
1. (c) E ...(i)
dx 3 0 E2 E1. ]
2 1 0 2 0 1
2. (c) If is the surface charge density, then
Passage for Q.7 to Q.9,
E 7. (c) If V is the potential of the capacitor, then
… (ii)
0
Q0
On comparing (i) and (ii), we get V .
C0
4a 1/ 3
= 0 x V Q0 C0
3 The current in the resistor, i .
Volume charge density, R R
8. (a) If q is the charge on each capacitor, then
d 4
= a 0 x2 / 3
dx 9 q q
iR 0
3. (b) Current density, j = v C C0

2eV dq 1 1 di
where v ] or R 0
m dt C C0 dt
Passage for Q.4 to Q.6
4. (c) 1 1 di
or i R 0
5. (d) V E1d1 E2d 2 ...(i) C C0 dt
By equation of continuity 9. (d) After simplifying above equation, we get
E1 E2
t (C C0 )
j1 j2 or ...(ii)
1 2 RCC0
i i0e .
298 ELECTRICITY & MAGNETISM
Passage for Q.10 to Q.12, Now VAJ i R2
The equivalent circuit is shown in figure. See examples.
or iRAJ i R2

E or 10 0.5
0.2 5 5
50 5 r
F 2
H r = 7.5 .
G 19. (a) 2 iRAJ

or 2 10
Passage for Q.13 & Q.14, 0.5 ( AJ )
13. (a) From the given graph it is clear that with increase of the 10 50
magnitude of magnetic field (B), the critical temperature TC AJ = 12.5 cm.
(B) decreases. 20. A-q : Energy stored in capacitor will convert into thermal energy.
Given B2 > B1. Therefore for B2, the temperature at which B-r : Induced emf, e = B v .
the resistance becomes zero should be less. The above C-s : Because of electric force ends of wire will have opposite
statement is true for graph (a). charges.
14. (b) We know that as B increases, TC decreases but the exact D-p, q, r : When battery is connected to wire a constant current
dependence is not known. flows in the wire which produces heating effect.
Given at B = 0, TC = 100 K SG 1 99
and at B = 7.5T, TC = 75 K 21. A-r : Resistance of ammeter, R A 0.99
S G 1 99
At B = 5T, TC should be between 75 K and 100 K.
Passage for Q.15 & Q.16, V
15. (a) Let j be the current density. Current i
R
I 12
Then j 2 r2 I j or 3 ; r = 1.01
2 r2 2 0.99
I B - q : Using kirchoff's loop rule, we have
E j
2 r2 12 – 1.01(i1 i2 ) 2i1 0
a a
' I and 2 i1 – 200 i2 0
Now, VBC E . dr 2
dr
a b a b 2 r After solving above equations, we get

a
i2 0.04 A (i1 + i2) 1.01
I 1 I I
2 r 2 a 2 ( a b) 5
a b ig i2 0.05 A . i1
4
On applying superposition as mentioned we get
2
I I S
VBC 2 '
VBC C-p Now ig i
a (a b ) S G
i2 200
I 1
16. (c) As shown in Answer. E or 0.05 i ; i = 5A
2 r2 100
Passage for Q.17 & Q.19, D-q Also V ig (G R) 0.05(99 101) 10 V
2 10
17. (a) 2 iRAJ 31.25 0.5 V .
10 15 50 R R
18. (a) The effective circuit is shown in figure. 22. A-r : RAC R A C
The current in potentiometer wire,
2 R R
i 0.2 A,
10
B
0.5
and current in R2, i . R
5 r
5R
2V k B-q : R AB A 2R/3
8
E
R
A J
=0.5 V G
C-p : It is the balance bridge and so VB VD .
r
R2=5 D-t : VB VD .
DC AND DC CIRCUITS 299
23. See theory of the chapter. and V 2.5 – 0.1 5 2V
24. A-p : If current in branch BD is not charge even K is closed, this is
or Vx V y 2V .
the condition of balanced bridge and so VA VB .
B-p, r, t There is no current in R and capacitor.
B-p, q, r, t : p.d. depends on emfs of the cells.
C- p, t : In this case p.d. occurs along x-axis, and no p.d. along 2µF
y-axis. X V A
R
D- p, q, r, t : p.d. depends on the values of k1, k2 and k3. i
25. For ideal voltmeter, its resistance is infinite and so there is no 3V 10
current in the branch in which it connected. 1V
i

2V 2V
V 2 –1
R i 0.1 A
C-p, q, r, s :
10
5 i 2.5 V y P.d. across capacitor is
V 3 – 2 1V
20 Charge on capacitor
q CV 2 i
x i = 2µF.
7 2
A - p, q, r, t : D-q, t : i 0.1 A.
7 20
2.5
i 0.1 A
25

Solutions Exercise-3.5
1. We know that, charge In close loop, we have
t i2 R i1 10 100 40 = 0
i dt or 10R 2 10 100 40 = 0
q =
0 R = 4
10 24
4. The current in the circuit i = 2A
20 4t dt 12
=
0 The p.d. across 4 resistor.. V1 V0 = 4 2
As, V0 = 0,
2 10
4t
= 20t V1 8V
2 0
= 400 C Ans. Similarly p.d. across 2 resistor
2. At steady state, the p.d. across the capacitor, V0 = 6 V. When switch V0 V2
= 2 2
with S is opened the current starts flowing from capacitor towards
resistor. The initial current V2 = – 4V Ans.
5. The effective circuit is shown in figure.
V0 6
i0 = 0.6 A
R 10 4
The time constant, = 6
CR 25 10 10
6
= 25 10 5s 4
t/ 4 12 V
We know that, i = i0e
8
= 10 3 / 25 10 5
0.6 e 2
4
= 0.6 e
3 The equivalent resistance, R = 2
= 11 10 A Ans.
3. The current in 20 resistor, i1 12
Current i = 6A
2
40
= 2 A.
20 12 8 96
6. The equivalent resistanceR =
Thus current in R will be, i2 = 8 2 10 A 12 8 20
300 ELECTRICITY & MAGNETISM
24 10.
The current, i 5A = Ans.
96 / 20 Cells connected in series
7. The successive reduction of the circuit is as :
Clearly equivalent resistance, R R
R = 2

3
3 I
3 3
3
6 6 6
6
r r
2E 2r

6
6 E E
3
3 A B 2
6 2E
A B J1 I 2R .R ...(1)
2r R
8. In close loop (1), we have 4 10i 2i1 0, and in close loop (2), we
have Cells connected in parallel
10 i i1 2 2i1 0 R
R
2V ( i– i1)

2 I
2 r
10 E
4V 1
A i1
i E r
10
E r 2
After solving above equations, we have
2
1 E
i1 =
7
A Ans. J2 I 2R R
r ...(2)
9. The current in the circuit, R
2
1 2 5 2 1
i = R1 R2 10 20 10
A Given J1 = 2.25 J2

Thus V1 V2 = iR2 (2 E )2 E2
2 .R 2.25 .R
(2r R) 2 r 2
1 (R )
= 2 20 4V . Ans. 2
10
4 2.25
2 2
(2r R) r
R
2

4[ R 0.5]2 2.25[2 R]2 [ r 1 ]


2 (R + 0.5) = 1.5 (2 + R)
R=4
DC AND DC CIRCUITS 301

S olutions Exercise-2.6
1. If V0 is the zero error in the voltmeter, then we can write 1i1 =
2i2 1 i2 i1 0
V = V0 ir
and 1i1 2i2 1 i2 i1 = 0
or 14.4 = V0 1.75 r
After simplifying, we get i1 6 A, i2 4 A. Ans.
and 22.4 = V0 2.75 r
After simplifying, we get 2
8. i1 = 0.4 A
V0 = 0.4 V Ans. 5
2. At the left junction,
2 x V xy 1 B 4V
3 5 8 i1 = 0
i1 = –6A. i2
Similarly i2 = 9A. 2V
3 5
3. We know that 3
i1 4V
V = ir
i1
or 1.52 = 0 y
A i2
= 1.52 V.
In second case 4
and i2 = 0.5 A
V = ir 8
or 1.45 = 1.52 1 r In close loop x A B y x, we have
r = 0.07 Ans.
3i1 4 3i2 Vxy = 0
4. For maximum current,
20 10 20 Vxy = 3.7 V Ans.
R = 0
20 10 3
V 5.5
9. (a) The equivalent resistance of the circuit
The current, imax = 0.83 A. R = 1 5 6
R 20
3 3 6
For minimum current and C = 2 F
3 6
20 10 110 The time constant, = CR 2 6 12 s.
R = 30
20 10 3 (b) P.d. across both the capacitors together
V 5.5 t/
imin = 0.15 A Ans. V = V0 1 e
R 110
3 / 1
= 12 1 e 12 1
5. Upper 4 resistor is shorted, so current in it is zero. Then other e
two resistors are in series, so = 7.6 V
4 2 If V1 and V2 one the p.d. across 3 F and 6 F capacitors,
i = 0.2 A Ans. then V1 + V2 = 7.6
4 6
6. The effective circuit is shown in figure.
and 3V1 = 6V2
10 10 10
V2 = 2.53 V Ans.

10. The effective circuit may be drawn as :


10 10 10
i1 4 3 4 i1
i2
3 3 i2
6V
2 5 5
The equivalent resistance,R = 15
V 6 A
Current, i = 0.4 A Ans. B
R 15 i3 i3
7. The currents in different resistors are as shown in figure : 6 15
In close loops, we have 9
15 9
i1 i2
1 2 i4 12 9 12 i4

1 (i2 –i1) Clearly the effective resistance R = 6.75 Ans.


2 1
i2 i1

14 V
302 ELECTRICITY & MAGNETISM
2R resistance branch, which is shown in figure.
11. One unit cell consists of .
The current, i = R1
R
K

2R
A A
R1 C1
R Rx Rx
So, = (b) After a long time capacitors will offer infinite resistance and
so effective circuit is as :
B B
The current i = .
R1 R3
2 R Rx R
or 2 R Rx R = Rx
K

After simplifying, we get Rx 3 1 R. Ans.


12. If R is the resistance between A and B, then
R1
RAB = R A'B'
R3
R2 R
or R = R1 16. The p.d. across the capacitor
R2 R
Q 20
R1 V0 = 4V
A C 5
A'
V 4
R2 R The initial current, i0 = 0.8 A.
R 5
B' = CR = 20 × 5 = 100 s.
B The current in the resistor at any time t
i = t/
After substituting the values and simplifying, we get R 6 . i0e
Ans. t2
13. The effective circuit is shown in figure
i 2 Rdt
The heat dissipated, H =
2 2 t1
A A
50
25 3 t2
B B t/ 2
i0e Rdt
8 8 =
t1
50 50
2t / t2
50 e
= i02 R
2/ t1
Thus equivalent resistance,
i02 R 1 1
50 50 / 3 =
R = 2 8 2 e 2t1 / e 2t2 /
50 50 / 3
After substituting and simplifying, we get
= 22.5 . Ans. H = 4.7 J. Ans.
14. The currents in two successive restores are as : 17. (a) The current in the circuit just after connection,
R1 R1 V0 6
A i0 = 0.25 A.
i1 i1 R 24
2 (b) Current in the circuit, one time constant after connection are
i2 mate,
R2 R2
i2 i = t/
i0e
2
B = 0.25 e 1
i1 = 0.09 A. Ans.
Clearly i2 =
2 18. Power delivered by the battery
i1 P = i
or i2 = 2 t/
or P = i0e
R1 1 t/
= . Ans. or P = P0e
R2 2
15. (a) Immediately after the key is closed, the capacitor offers zero P0
For, P = , we have
resistance. The circuit will be completed through least 2
DC AND DC CIRCUITS 303
P0 t/
and 10i3 10 i3 i2 10 i1 i3 = 0 ...(iii)
= P0e
2 After solving above equations, we get
or = 2 et / i3 10
t = 0.693 . Ans.
19. The effective circuit is shown in figure. 3
10 10 10
Resistance of wires, R12 R13 R34 R24 r3 4
i1 ( i 1 – i3 ) ( i 3 – i2 )
( i 1 – i2 )
r 6
and R15 R25 R36 R46 . 100V 1 10 2 50V
2 5
The heat liberated per unit time, i2
2
1 2
V
Q12 = . i1 = 3.75 A, i2 0, i3 1.25 A Ans.
r V
From Ohm's law, we can get current through the conductor 3 – 4 : 23. The current distribution in different branches of the circuit is shown
in the diagram.
V By using Kirchoff's loop rape in four loops, we can get
i34 = .
r 2 3 i2 = 1.85 A.

2 V2 i3
Thus Q34 = i34 r 2
.
r 2 3
6 12V
Therefore required ratio is :
(i1–i2+i3–i )
Q12 2 i

4
Q34 = 2 3 11 6 2. Ans. 4

3
6V (i 1–i 2)
20. (a, b) i1 i2
2
The power of the circuit P = i2 R i
3 6V (i–i1)
or 50 = 12 2 1
24. At steady state, there is no current in the capacitor, so the effective
i circuit is as follows :
A B
R=2 If V1 and V2 are the p.d. across 4 and 8 respectively, then
= 48 V.
Thus p.d. between A and B, V1 V2 = 24
VAB = 48 1 2 50 V . Ans. V1 V2
and =
21. At steady state, there will be no current in the capacitor. 4 8
The current distribution is shown in diagram. In loop (2),
V1 = 8 V and V2 = 16 V
6
i1 = 1.5 A
4 A
In loop (3), 4 8
2 2i2 4i1 3i2 3 = 0
i2 = 1A.
The p.d. across the capacitor, 2
4
F 6 G 6V B
H S

5 F 1 2 24V
2 4
E i2 D C Similarly if V3 and V4 are the p.d. across 4 and 2 respectively,,
2V i1
then
3
i2 V3 V4 = 24
A
3V 3 B
V3 V4
and =
VEG = 2 2i2 2 2 1 0 4 2

1
V3 = 16 V and V4 = 8V.
2
U = CVEG 0 Ans. Thus p.d. across capacitor
2
22. Using Kirchoff's loop rule in loop (1), (2) and (3), we have VA VB = V3 V1 8V . Ans.
100 10i1 10 i1 i3 10 i1 i2 = 0 ...(i) The discharged circuit is shown in figure.
8 10 40
10 i3 i2 50 10 i1 i2 = 0 ...(ii) The effective resistance, R = .
8 10 9
304 ELECTRICITY & MAGNETISM
VA VB 4 8 V VC
Current i = Now i = . Ans.
R 0 0
8V C
8
= 1.8 A. Ans.
i
29. The current in the circuit,
40 4 2
9 i1 i2 2 1
i = .
R1 R2 R
25. During the time interval from 0 to t0, the voltage across the capacitor
is zero, the charge on it is also zero, there is no current through it
1 R1 1 R2
and hence VCD is zero. From t0 to 2t0 , the voltage across the
capacitor, and hence the charge on its plates increases linearly and
hence a current passes through it. This means that voltage VCD
becomes constant. During time interval 2t0 to 3t0 the voltage across R
the capacitor does not change. Hence current does not flow, and
The p.d. across any source is,V = 1 ir.
VCD is zero. Finally from 3t0 to 5t0 , the capacitor is discharged
As r R2 is greater, so p.d. across this cell becomes zero. Thus
the current through the resistor is negative and constant, and its
magnitude is half the value of the current during t0 to 2t0. 2
0 = R2
R1 R2 R
26. For balanced bridge,
RAJ 0.6 12 or R = R2 R1.
RJB
= 30. Using Kirchoff's loop rule in loops (1) and (2), we have
0.4 x
R i1 i2 i1R1 = 0
D
RAJ R JB ( i1+ i2)
i1 i2
p B
A 1 R1 2 R2
1
R
x
12

C
and i1R1 i2 R2 = 0
After solving above equations, and substituting the known values,
x = 8 . Ans. we get
27. Suppose the linear charge density of the cylinder is . Take an i1 = 1.2 A and i2 0.8 A.
element of length dx on the surface of the cylinder, the charge on it
is, dq dx. The electric field at a point on the surface of the 31. In closed loop 123451, 2 iR2 iR1 1 = 0
cylinder is 2 1
or i = R1 R2
E = .
2 0 a R1
1 i
5 4
= 2 0 aE

1
6 3
and dq = 2 0 aE dx A B
i
dq dx i
Thus current, i = 2 0 aE 1 2
dt dt 2 R2
or i = 2 0 aEv. Ans. Now in close loop 12361, we have
V V 2 iR2 VAB 1 = 0
28. The leakage current i = .
R d VAB = iR2
1 2
A
A 1 2 R1
= 0.5V.
R1 R2

n
32. (a) The current, i =
i nR R
R
= . Ans.
R
d
(b) The circuit can be redrawn as :
0 A For upper close loop, we have
We know that, C =
d
d 0
=
A C
DC AND DC CIRCUITS 305
n-cells 1 2
36. The circuit current i = .
R1 R2
A V AB B

The p.d. VA VB = 1 i1R1


(N-n)-cells
1 R1
n i nR = VAB
i
or n R nR = VAB
1 2 A B
= 0. VAB Ans. = 1 R1
R1 R2
33. The resistance of x length of the potentiometer wire is R2
2
R0 x R0 x
Rx = , and R
x 1R2 2 R1 .
= R1 R2 Ans.
The total resistance of the circuit
R0 x R2 i4
R 8 7 6 5
R0 x
RT = R0 x
37. i0 i1
R
0
R1 R3
x xR 1
= R0. i3
R xR0 i2 R
i0
V0 1 2 i1 3 i4 4
The circuit current, i =
x xR
R0 Applying Kirchoff's junction rule, we have
R xR0
i0 = i1 i2 ...(i)
Rx R
The voltage, V = i .
and i1 = i3 i4 ...(ii)
Rx R
After substituting the values, we get Now in the loop 12781,
V0 Rx 0 i2 R1 = 0 ...(iii)
V = R R0 x
x
. Ans.
In loop 1236781,
i3 R3 i1R2 0 = 0 ...(iv)
34. (a) We have, VA VB = 6 V. and in loop 34563,
As VB = 0, VA 6V i4 R 1 i3 R3 = 0 ...(v)
VC = VA 4V 6 4 2V. After solving above equations, we get
(b) The current in the potentiometer wire, R2 R3 0 R3
i4 = . Ans.
6 R R2 R3 R2 R3
i = .
RAB 38. The current in different branches is shown in diagram.
If x is the required length, then In close loops (1) and (2), we have,
VAD = iRAD
A (i1+i2)
6 RAB x
or 4 = RAB 100 i1 i2
1
x = 66.7 cm. Ans. R2
1
(c) As VD VC , so VD VC 0, therefore no current through it. 1 2 R3
(d) Do as above. R1
35. By Kirchoff's I law i = i1 i2 . 2
Using Kirchoff's loop rule in loops (1) and (2), we have
2 i1R2 1 i2 R1 = 0 B

and i2 R1 1 iR = 0 i1R1 1 i2 R2 2 = 0
After substituting the values, and solving, we get,
and 2 i2 R2 1 i1 i2 R3 = 0
i = 0.02 A. Ans.
After solving above equations, and substituting the values, we get
R2
2 i1 = 0.06 A.
i1 1 R1 39. Suppose q is the charge on the capacitor at any instant, and i2 is
i2 the current, then
1
i 2 dq
R i2 = .
dt
306 ELECTRICITY & MAGNETISM
By Kirchoff's first law, In close loop 1-2-8-7-1,

6 Sw 5 i2 4 i1R3 i2 R2 = 0, ...(i)
2
i
i1 and in close loop 4-5-6-7-8-4,
q
1
R –q i2 R2 i3 R1 = 0 ...(ii)
1

R At junction 7, we have
1 2 3
i1 = i2 i3 ...(iii)
i = i1 i2
After solving above equations, we get
dq
= i1
dt 1 R2 R3 2 R2
In close loop 1-2-3-4-5-6-1, i3 = R1R2 R2 R3 R3 R1
q
ir = 0
C Potential difference, VA VB = 2 i3R1
dq q
or i1 R = 0 ...(i)
dt C 2 R3 R1 R2 1R1 R2 R3
= . Ans.
In close loop 2-3-4-5-2, R1R2 R2 R3 R3 R1
q 41. The current distribution is shown in figure.
i1R = 0 ...(ii)
C
In closed loop A-6-5-4-B-C-D-A,
dq 2q
From equations (i) and (ii), we get R =
dt C i2 R3 i2 i3 R4 V = 0 ...(i)
q t
dq dt In closed loop 1-2-3-4-5-6-1,
or =
2q R
0 0 i1R1 i1 i3 R2 i2 i3 R4 i2 R3 = 0 ...(ii)
C
2t i1
C 2 (i1–i3)
or q = 1 e RC 1 3
2 R1 R2
q 2t / RC
V = 1 e Ans. A B
C 2 i3
40. The current distribution is shown in figure. i2
6 4
R3 5 R4 (i2+ i3)
2 C
1 3
i1 R3 A B
D C
V
2
i2
7 4 and closed loop 1-2-5-6-1,
R2 8 R1 i3
i1R1 i2 R3 = 0 ....(iii)
6 5 After substituting the values and simplifying, we get
1 i3
i = 1 A. Ans.
308 ELECTRICITY & MAGNETISM
4.1 THERMAL EFFECT OF CURRENT : JOULE'S LAW
Consider a part of circuit, carrying current i. If dq charge flows between two points
differing in potential by V in time dt, then work done by the electric field
dW = V dq
= V idt
The rate at which energy enters in this part of the circuit
dW
Fig. 4.1 i.e., Power, P = Vi
dt
The SI unit of power is J/s. 1 J/s = 1 W.
The power dissipated in the resistor
If the portion of the circuit is a resistor, the potential difference is given by V = iR and

V2
P = V i = i2 R =
R
The power enters in the resistor appears as thermal energy. On a microscopic scale the
transfer of power in the form of thermal energy can be understood as : Collisions between
Fig. 4.2 the electrons and lattice of ions increase the amplitude of the thermal vibrations of the
lattice and this corresponds to increase in temperature of resistor. This effect is often
called Joule heating effect.

Power output of a source


Consider a source of emf and internal resistance r, connected to an external circuit. The
current i flows out from the source. The power input into the external circuit,
P = Vi
We have, V = – i r,
and hence, P = ( – i r) i
= i – i2r
The product i is the rate at which work is done on the source. The term i2 r is the rate
at which energy is dissipated in the internal resistance of the source. The power output
of the source is the power input to the remaining part of the circuit. Therefore
Fig. 4.3 Pout = i – i2 r
Power input to a source
Let us consider the source of emf and internal resistance r. The current i is flowing into
the source. The power input into the source
P = Vi.
Here we have V = +ir
Therefore Pin = ( + i r) i
or Pin = i + i2 r

Loss of power in transmission lines and cables


Fig. 4.4 When electrical power is transmitted from the generator to the point where it is used,
there is always a loss of energy due to the resistance of the transmission wires owing to
the Joule heating effect.
The power used in home is measured by the product of current i and voltage V between
the power lines, as they enter the home output power PO = V i. If the total resistance of
the wires from generator to the home is R, then power lost = i2 R. Hence the input power
to the home is P i = V i + i2R, and the voltage at the generator must be V + i R. The
electrical efficiency
PO
= Pi
Fig. 4.5

Vi
or
Vi i2 R
THERMAL AND CHEMICAL EFFECT OF CURRENT 309

Note: To keep losses minimum, the power is always supplied at high potential, and
so to keep the current in the power lines as low as possible

Prequired
i =
V
2
Prequired
and Ploss = i2 R = R.
V2
1
From the above equation, it is clear that, Ploss .
V
The supply voltage from the generator may be 33000 V or more.
Maximum power theorem
Consider a battery of emf and internal resistance r is used in a circuit with a variable
external resistance R. The value of R for which the power consumed in R is maximum;

The current in the resistor R is i =


R r
The power consumed in R is
2
R
P = i2 R
2
R r

dP
P to be maximum 0, Fig. 4.6
dR
2 2
dP R r 2R R r
or = 4 =0
dR R r
which gives R = r
2 2
R r
and Pmax = 2 2
R r r r

2
or Pmax = .
4r
Kilowatt-hour Fig. 4.7
It is the commercial unit of electrical energy. It is knows as 1 unit.
Thus 1kW - h = 1000 3600 J
= 3.6 106J

Number of units consumed = total power (W ) time( h) .


1000
Ex. 1 Consider following circuit, find power generated in the
Sol. The power input in the source (electrical power produced)
resistor R.
Pin = i
= 12 2 = 24 W
The power dissipated in the source
= i2 r = 2 2 2
= 8W
The power output of the source
Pout = 24 – 8 = 16 W
The power generated in resistor = 16 W
Fig. 4.8
310 ELECTRICITY & MAGNETISM
4.2 ELECTRICAL APPLIANCES

1. Filament of electrical bulb : It is made of tungsten which has low resistivity and
high melting point.
2. Filament of heating devices : It is made of nichrome which has high resistivity and
high melting point.
3. Standard resistances (Resistance box) : Standard resistances are made of manganin
or constantan. These materials have moderate resistivity and very low temperature
coefficient of resistance.
4. Fuse wire : It is made of tin-lead alloy. It has low resistivity and low melting point.

Rated or design values :


Some of the values like; wattage, voltage etc. are printed on the electrical appliances are
called rated or design values. These values give the informations about resistance and
allowable current etc. For a bulb of 100 W and 200 V,
Its resistance can be obtained by
2
Vdesign
R = ;
Pdesign

1
or R
Pdesign
where Vdesign and Pdesign are the design voltage and power of the bulb.
2202
R =
100
= 484
[Assuming this resistance constant for all values of supply voltage V]
Pdesign
Allowable current i = Vdesign
Fig. 4.9

100 5
= A
220 11

Note: All electrical appliances are design for same voltage; i.e., 220 V..
Power consumed
Power consumed depend on the supply voltage which may be different from rated
power.
2
Vsupply
Pconsumed =
R
For Vsupply = Vdesign ; Pconsumed = Pdesign
Let us now consider two bulbs of wattage P1 and P2 :
For Vsupply = Vdesign = V

V2
Resistance of first bulb R1 =
P1

V2
and resistance of second bulb R2 =
P2
THERMAL AND CHEMICAL EFFECT OF CURRENT 311
1. Bulbs connected in series : In series both the bulbs have same current.
P1 consumed i 2 R1 R1
P2 = 2 R2
consumed i R2
If P1 > P2 then R1 < R2 ,
P1 consumed P2 consumed .
Thus in series low wattage bulb glows brighter than high wattage bulb.

Total power consumed


Fig. 4.10
V2 V2
Pconsumed =
Rtotal R1 R2

V2
=
V2 V2
P1 P2
1 1 1
or =
Pconsumed P1 P2
2. Bulbs connected in parallel : In parallel the voltage across them is same.

V2
P1 consumed R1
P2 =
consumed V2
R2
R2
= R1
If P1 > P2 then R1 < R2 , Fig. 4.11
P1 consumed
P2 consumed
Thus in parallel high wattage bulb glows brighter than less wattage bulb.
V2 V2
Total power consumed, Pconsumed =
Rtotal R1 R2
R1 R2

V2 V2
where R1 = and R2 =
P1 P2
Pconsumed = P1 + P2.

Fuse : It is used in series with the circuit to prevent the electrical appliances from
burning by melting itself to open the circuit.
Fig. 4.12
Let R is the resistance of the fuse wire, then

R =
r2

2 i2
The heat produced in fuse wire H = i R
r2
If h is the heat loss per unit surface area of fuse wire, then heat radiated per second
= h 2 r
Fig. 4.13
312 ELECTRICITY & MAGNETISM

i2
At thermal equilibrium = h 2 r
r2

i2
or h =
2 2r3
According to Newton's law of cooling
h = C
where is the temperature of fuse wire above surrounding, and C is a constant]

i2
= C
2 2r3

i2
or =
2 2 r 3C

The above expression is free from length of the fuse wire. Hence the function of fuse is
independent of its length provided i remains constant.
For given material of fuse wire
i2 r3.
An important explanation
In houses all electrical appliances are connected in parallel with the supply line. Hence
each of them has same potential. But because of resistance of current carrying wires, the
potential across each of them is different. It is common experience that when any high
voltage appliance is switched on, the other get affected (bulb glows less brighter). It is
due to current carrying cables. To understand more clearly consider a simple circuit
having a bulb and a heater connected in paralled with a source.
When only bulb is switched on

200 4
i = A
50 100 3
The power consumed by bulb

2
4 1600
P = i2R = 100 W 177.8W
3 9
Fig. 4.14
Now heater is also switched on, then net current supply

200
i = 2A
100 100
50
100 100

The current flows in the bulb = 1A


The power consumed by the bulb P = i2R
= 12 × 100 = 100 W
It is clear from above calculations that when heater in parallel is switched on, bulb
consumes less power, and therefore becomes dim. If the resistance of current carrying
wire becomes zero, there will no such effect occur.
THERMAL AND CHEMICAL EFFECT OF CURRENT 313
Ex. 2 Two bulbs rated 25 W-200 V and 100 W-200 V are Sol.
connected in series to a 400 V supply. Show with necessary Let H is the heat required to boil the water, then for first coil
calculations, which bulb if any will fuse ? What should happen if
the two bulbs were connected in parallel to the same supply ? V2 V 2t1
H = t1 R1 =
Sol. R1 H
Resistances of the bulbs:
V 2t2
and for second coil R2 =
V2 200 2 H
R1 = 1600 (i) When both the filaments are connected in series
P1 25
V2 V2
2 2 H = t t
V 200 Rtotal R1 R2
and R2 = 400
P2 100

= V2
t
V 2t1 V 2t2
H H

t = t1 + t2
(ii) When both the filaments are connected in parallel

Fig. 4.15 V2 V2
H = t t
P1 25 1 Rtotal R1R2
Allowable currents: i1 = A
V 200 8 R1 R2

P2 100 1 After substituting values of R1 and R2, we get


and i2 = A
V 200 2 1 1 1
= .
(i) When they are connected in series, both have same current t t1 t2
Vsupply 400 1 Ex. 4 A cell of emf and internal resistance r supplies
i = A
R1 R2 1600 400 5 currents for the same time t through external resistance R1 and R2
separately. If the heat developed in the external resistance in the
> i1
two cases is the same, then what should be the internal resistance ?
< i2
It means 25 W bulb will fuse in series. Sol.
(ii) When bulbs are connected in parallel, both the bulb will have same
voltage. Thus In first case i1 = R r
1
Vsupply
and heat produced in R1
i1' =
R1
2
400 1 H1 = i12 R1t R1t
= A R1 r
1600 4
> i1
In second case i2 = R r
Vsupply 2
and i2' =
R2
Fig. 4.16
and heat produced in R2
2
400
= 1A i22 R2t R2t
400 H2 =
R2 r
> i2
The currents in both the bulbs are greater than their allowable
For the condition given,
currents. Therefore both bulbs will fuse in parallel. H1 = H2
Ex. 3 An electric kettle has two heating elements. One brings 2 2

it to boil water in time t1 and the other in time t2. Find the time t or R1t = R2t
R1 r R2 r
taken by water to boil in kettle when
(i) heating filaments are connected in series. After solving , we get
(ii) heating filaments are connected in parallel.
r = R1 R2 . Ans.
314 ELECTRICITY & MAGNETISM
Ex. 5 What amount of heat will be generated in a coil of Substituting 2 t/ t =z
resistance R due to a charge q passing through it if the current in
the coil t
or ln 2 = ln z
(a) decreases down to zero uniformly during a time interval t. t
(b) decreases down to zero halving its value every t seconds? Differentiating above equation, we have
Sol. d t d ln z
(a) As the current decreases uniformly, its value at any time t, assuming ln 2 =
dt t dt
initial current i0
ln 2 1 dz
or =
t z dt

t dz
or dt = .
ln 2 z
Substituting these values in equation (i), we get

t dz
q = i0 z
0
ln 2 z
Fig. 4.17
i0 t i0 (– t ) z 0
i = i0 – t i0 1 =
t t ln 2

t t
t i0 ( t ) t/ t
Thus q = idt i0 1 dt = 2 0
t ln 2
0 0

t i0 t
t2 i0 t =
= i0 t ln 2
2 t 0
2
t/ t
q ln 2 q ln 2 1
2q Thus we have i0 = and i
or i0 = t t 2
t
The heat generated in the time interval 0 to
t 2q t
Hence i = i0 1 1
t t t
H = i 2 Rdt
The heat generated 0
t t 2
2q t
H = i 2 Rdt 1 Rdt q ln 2 1
t/ t
2

0 0
t t = Rdt
0 t 2
t
4q2 R t
3
= 1 q 2 2R
3 t = 2t / t
t 0 2 0
2 t

4q 2 R
= Ans. q2R
3 t = . Ans.
t
(b) According to given condition
t/ t
Ex. 6 A conductor has a temperature independent resistance
1 and a total heat capacity C. At the moment t = 0 is connected to a dc
i = i0
2 voltage V. Find the time dependance of the conductor's temperature
When current decreases according to above equation it becomes T assuming that thermal power dissipated into surrounding space
zero at t = . Therefore charge will flow till infinite time.
to vary as q = k(T - T0 ) , where k is a constant, T 0 is the
t/ t
1 environment temperature (equal to the conductor's temperature at
The charge q = i dt i0 dt
2 the initial moment).
0 0
Sol.
t/ t The rate of heat evolved by the resistance
= i0 2 dt … (i)
= rate of absorption of heat + power dissipated into surrounding
0
THERMAL AND CHEMICAL EFFECT OF CURRENT 315

V2 CdT
k (T T0 ) V2
or = k (T T0 )
R dt
ln R 2 =
kt
2 CdT V
V C
or k (T T0 ) =
R dt R

T t
dT dt k
or = ln 1 (T T0 ) kt
2 C 2 =
T0 V 0 V /R C
k (T T0 )
R
k (T T0 ) kt / C
T or 1 2
= e
2 (V / R )
ln V k (T T0 ) t
R =
C V2
k T0 or k (T T0 ) = 1 e kt / C
R
2 2 kt
ln V k (T T0 ) ln V k (T0 T0 ) = V2 kt / C
R R C or T = T0 (1 e ) . Ans.
kR

4.3 SEEBECK EFFECT

The conversion of electrical energy from thermal energy was discovered by Seebeck in
1826. According to him if the junctions of two different metals are kept at different
temperatures, then there is an electric current in the circuit. This effect is called Seebeck
effect. The emf produced across the junctions is called thermo-emf and the resulting
current is called thermo-electric current, the pair of metals is called a thermo-couple.
Thermoelectric series
Seebeck arranged different metals in a specific order which gives the thermo-electric
series, and showed that when any two metals of the series are used with junctions at
different temperatures, current flows from the metal earlier in the series to the metal later
in the series through the cold junction. Also, the series gives an idea of the relative
magnitude of emf for different thermocouples. Farther apart two metals lie in the series,
larger is the emf produced. The thermo-electric series is as :
Antimony, Fe, Cd, Zn, Ag, Au, Rb, Mo, Cr, Sn, Pb, Hg, Mn, Cu, Pt, Ni, constantan,
bismuth.
Take as an example, a thermocouple madeup of copper and iron. The current will be from Fig. 4.18 Thermocouple made of Cu-Fe.
iron to copper at the cold junction. The largest emf will generate in a thermocouple made
of antimony and bismuth.
Variation of emf with temperature
Suppose a thermocouple is made up of two metals A and B, and the thermo-emf produced
is . Figure shows graphically the variation in thermo-emf as the temperature of the
hot junction changes.
If tC, tn and ti denote the temperature of the cold junction, the neutral temperature and
the inversion temperature respectively, then
tn – tC = ti – tn

tC ti
or tn = … (1)
2
If the cold junction is kept in ice (0°C) and the hot junction at t(°C), then the thermo-emf Fig. 4.19
depends on the temperature as

bAB 2
AB = a AB t t . … (2)
2

where a AB and bAB are constants for a pair of metals A and B. It should be remembered
that the parabolic variation is obtained in the case when either of a and b is negative.
316 ELECTRICITY & MAGNETISM
Neutral and inversion temperature
The temperature of the hot junction at which the thermo-emf is maximum is called the
neutral temperature and the temperature at which the thermo-emf changes in sign is
called the inversion temperature.

Thus for AB to be maximum,

d
= 0
dt

d bt 2
or at = 0
dt 2

or a + bt = 0

t = tn = a. … (3)
b

As both a and b are constants for a thermo-couple and so neutral temperature is constant

d
for any thermo-couple. The quantity is called thermoelectric power or seebeck
dt
coefficient.
The maximum value of thermo-emf

2
a b a
max = a
b 2 b

or = a2 . … (4)
max
b
Coefficients a and b for thermocouple
2
Metal with Pb a(µV/°C) b (µV/°C )
Al –0.47 0.003
Bi –43.7 –0.47

Cu 2.76 0.012
Au 2.9 0.0093

Fe 16.6 –0.030
Ni 19.1 –0.030

Pt –1.79 –0.035
Ag 2.5 0.012

Steel 10.8 –0.016


THERMAL AND CHEMICAL EFFECT OF CURRENT 317
Law of intermediate metal
Suppose AB , AC and BC are the thermo emfs across the thermo-couples made of
A, B; A, C; and B, C respectively. If the temperature difference across the junctions of
all the thermocouples are the same, then

AC = AB BC … (5)

Law of intermediate temperature


t2
Let AB t denotes the thermo-emf of a thermocouple made of metals A and B when the
1
temperatures of the junctions are t1 and t2. Then
t2 t3 t2
AB t = AB t AB t … (6)
1 1 3

This is known as the law of intermediate metals.


Ex. 7 The cold junction of a thermocouple is maintained at Which on solving gives
20°C. No thermo-emf is developed when the hot junction is at = 400°C
540°C. Find the neutral temperature. Thus neutral temperature is 400°C. Ans.
Sol. Giventc = 20°C, ti = 540°C
Ex. 9 Find the emf of a Cu – Fe thermocouple when
tc ti temperature of hot junction is 100°C and that of cold junction is
tn =
2 0°C.

=
20 540
280 C . Ans.
Sol. We know that
2
Ex. 8 The expression for thermo emf in a thermocouple is = at
bt 2
2
2
given by the relation , where
40 is the temperature By law of intermediate metals, we have
20
difference of two junctions. What is the neutral temperature? Cu,Fe = Cu,Pb Pb,Fe Cu,Pb Fe,Pb
2
Sol. Given = 40
20 t2 t2
= aCu t bCu a Fet bFe
2 Pb 2 Pb
For maximum value of ,
1
d = aCu aFe t bCu bFe t2
= 0 Pb 2 Pb
d

d 2 1
or 40 = 0 = (2.76 – 16.65) × 100 + (0.012 + 0.030) × 1002
d 2
20
= – 1.179 mV Ans.
2
or 40 = 0
20

4.4 PELTIER EFFECT


In 1834, Peltier discovered that if a current is allowed to pass through the junction of two
different metals, heat is either evolved or absorbed at the junctions.That means the
junction is either heated or cooled. This effect is known as Peltier effect and the heat
evolved or absorbed is knwon as Peltier heat. Experiments has shown that the peltier
heat transferred at any junction is proportional to the amount of current crossing the
junction and changes its sign when reversing the direction of current. Thus Peltier effect
is reversible. If H is the amount of heat evolved or absorbed when a charge Q is
passed through the junction, then we define Peltier coefficient (Peltier emf) as : Fig. 4.20 Peltier effect

Peltier heat H
=
charge transferred Q
318 ELECTRICITY & MAGNETISM
4.5 THOMSON EFFECT
If a metal has a nonuniform temperature and a current is passed through it, heat is
absorbed or evolved in different sections of the metal. This heat is over and above the
Joule's heat. This effect is called Thomson effect. If a charge Q is passed through a
small section of the metal wire having temperature difference t between the ends, the
H
Thomson coefficient = .
Q t

Explanation of Seebeck, Peltier and Thomson effects


We know that the density of free electrons is different in different metals at same
temperature. Thus when two different metals are joined together, the electrons tend to
Fig. 4.21 (a) Two metals at same diffuse from the side with higher concentration to the side with lower concentration.
temperature Also the density of free electrons varies inside the same metal, if temperature of metal is
not uniform everywhere. Thus electrons tend to diffuse from the higher concentration to
the lower concentration region. So if the junctions are kept at different temperatures, an
Fig. 4.21 (b) A metals with different emf' is developed across the junctions.
temperature It can be easily understand that at the cold junction the electron density is more than the
electron density at the hot junction. It will cause the p.d. between the junctions. Thus, if

AB t , AB t be the emf's at the cold and hot junctions respectively, then thermo
1 2

-emf will be AB AB t
2
AB t .
1
It is clear from the above explanation that Seebeck emf is a combination of Peltier emf and
Thomson emf. Thus

AB = AB t AB t t( A B).
2 1

Fig. 4.22 Difference between Peltier effect and Joule's effect


Peltier effect Joule's effect
1. It is a reversible effect. 1. It is an irreversible effect.
2. It takes place at the junctions. 2. It takes place across the entire metal.
3. It may be heating or a cooling effect. 3. It always be a heating effect.
4. Peltier heat is proportional to the current: 4. Joule heat is proportional to the square

H Peltier i. of the current: H Joule i2.

4.6 CHEMICAL EFFECT OF CURRENT


Electrolyte : The liquid which dissociates into its ions on passing current through it
is called electrolyte.
Electrolysis : The process of decomposition of electrolyte solution into ions on
passing current through it is called electrolysis.
Electroplating : The article to be electroplated is made as the cathode and the metal
to be deposited is made as the anode. A solution of the metal is taken as the electrolyte.
If gold is to be coated then auric chloride is used as electrolyte. In case of ornament of
silver, electroplating is to be done by AgNO3. The Ag+ ions move to the cathode and
receives an electron from the cathode to become neutral Ag atom and deposited on the
Fig. 4.23
cathode.
THERMAL AND CHEMICAL EFFECT OF CURRENT 319
Ag+ + e– = Ag.

The NO3 ion moves to the anode and gives its extra electrons to it. The NO3 ion is
converted to NO3,
NO3– = NO3 + e.
The NO3 so formed reacts with a silver atom of the anode to form AgNO3 which gets
dissolved in the solution. This way, silver anode slowly dissolved in the solution. Thus,
silver anode slowly dissolves and deposited on the cathode with the concentration of
the electrolyte remaining unchanged.

4.7 FARADAY'S LAW OF ELECTROLYSIS


First law : The mass of a substance liberated at an electrode is proportional to the
amount of the charge passing through the electrolyte. Thus
m = zQ
If an electric current of constant magnitude i is passed through an electrolyte for a time
t, then
Q = it
m = zit
where z is a constant called electrochemical equivalent (ECE) of the substance. The SI
unit of ECE is kg/C.
Second law : The mass of a substance liberated at an electrolyte by a given amount
of charge is proportional to the chemical equivalent of the substance. The chemical
equivalent of a substance is

atomic mass
W =
valency

If W1 and W2 are the chemical equivalent of two substances, then from Faraday is II law

m1 W1
m2 = W2 … (i)

Also from I law

m1 z1
m2 = z2 … (ii)

From (i) and (ii)

W1 z1
W2 = z2

W1 W2
or z1 = = constant (F)
z2

W
or = F
z
F is the proportionality constant called Faraday's constant. 1F = 96500 C/eq.
320 ELECTRICITY & MAGNETISM
Ex. 10 A brass plate having surface area 200 cm2 on one side is Sol. Chemical equivalent of oxygen
electroplated with 0.10 mm thick silver layers on both sides using
a 15A current. Find the time taken to do the job. The specific Atomic mass 16
W = = = 8.
gravity of silver is 10.5 and its atomic weight is 107.9 g/mol. valency 2

Sol. The volume of silver deposited Chemical equivalent of hydrogen = 1


V = 2 × vol. of silver deposited on one side By Faraday II law
= 2(200 × 10–4 × 0.1 × 10–3) m3
mO WO 8
= 4 × 10–6 m3 = W
mH H 1
The mass of silver needed
m = V = 10.5 × 103 × 4 × 10–6 mO 1
= 42 × 10–3 kg = 42 g or mH = = g
8 8
Chemical equivalent of silver
107 1
W = = 107 g We have 1g of oxygen = gram – equivalent of hydrogen
1 8
W 107 We know that for liberation of 1 gram - equivalent weight, it requires
z = =
F 96500 1
If t is the required time, then by Faraday I law 96500C of charge. Thus for gram-equivalent, it requires a charge
8
zit = m
42 42 96500 96500
t =
m
= = 1.12 104 C.
zi 107 107 15 8
15
96500 If t is the required time, then
= 42 minute Ans.
Ex. 11 A current of 1A is passed through a dilute solution of Q 1.12 10 4
t = =
i 1
sulphuric acid for some time to liberate 1g of oxygen. How much
hydrogen is liberated during this period ? How long was the current = 1.2 × 104 s
passed? Faraday constant = 96500 C/mole. = 3 hour 20 min Ans.

Review of formulae & Important Points


1. The rate of energy transfer is called power in an electrical device 2
with a potential difference V is R.
P =
P = V i. R r
If the device is a resistor, then we can write

V2
P = i2R .
R
2. Thermal energy produced in time t
Q = Pi = Vit.
For maximum power
In a resistor, electrical potential energy is converted to internal
thermal energy via collisions between charge carriers and atoms. 2
r = R and Pmax =
3. kW-h : It is the commercial unit of electrical energy. 4R
1kW-h = 3.6 × 106 J. 5. Electrical appliances :
1 kW-h is known as 1 unit of electrical energy. The resistance of any electrical appliance of power Pdesign and
Vdesign can be obtained by
power W time h 2
Number of units = Vdesign
1000 R =
Pdesign
Cost of electricity = number of units × cost of one unit The allowable current
4. Maximum power theorem :
Pdesign
Power generated in the resistor R i =
Vdesign
THERMAL AND CHEMICAL EFFECT OF CURRENT 321
6. In houses the electrical appliances are connected in parallel. If 11. Neutral temperature : It is constant for any thermocouple.
appliances of powers P1, P2, ... are connected in parallel across the Neutral temperature
design voltage V, then total power consumed
tc ti
P = P1 + P2 + ... tn =
2
In series :

1 1 a
1 ... Also tn = .
= P P2 b
P 1

7. Fuse wire : In a fuse wire, the change in its temperature for the The maximum value of will occur at tn, which is
constant current i is given by
a2
i 2
max = .
= b
2 2r 3C
12. Law of intermediate metal : For thermocouples made of A, B; B,
For the given material of fuse wire i 2 3 C and A,C
r .
8. Chemical effect of direct current : AB BC = AC

(i) Faraday s I law : The amount of substance deposited or 13. Law of intermediate temperature : For any thermocouple
liberated on any electrode is proportional to the charge flows
t3 t2 t2
in the electrolyte solution. Thus AB t AB t = AB t
1 3 1
m = zq = z it,
where z is called electrochemical equivalent. 14. Thermoelectric power or Seebeck coefficient

(ii) Faraday s II law : If same amount of charge flows in two d


different electrolyte solutions, then the ratio of amounts of S = a bt.
dt
substances deposited is proportional to their chemical
equivalent. Thus 15. Peltier coefficient :

m1 W1 H d
= T .
m2 = W2 Q dt

9. Faraday constant : 16. Thomson coefficient :

W H dS
= F (Faraday constant) = T
Z Q t dt
1F = 96500 C/eq.
10. Seebeck effect : The conversion of thermal energy into electrical
energy is known as Seebeck effect.
The emf across the junctions of two different metals is given by

bt 2
= at ,
2
where a and b are Seebeck s constants.
322 ELECTRICITY & MAGNETISM

MCQ Type 1 E xercise 4. 1

Only one option correct 9. A battery of e.m.f. 10 V and internal resistance 0.5 ohm is connected
1. Which of the following plots may represent the thermal energy across a variable resistance R. The value of R for which the power
produced in a resistor in a given time as a function of the electric delivered in it is maximum is given by
current ? (a) 2.0 ohm (b) 0.25 ohm
(c) 1.0 ohm (d) 0.5 ohm
10. An electric bulb is designed to draw power P0 at voltage V0. If the
voltage is V it draws a power P. Then
2
2 V
V0 P P0
(a) P P0 (b)
V V0
(a) I (b) II
(c) III (d) IV
V V0
2. Two resistors R and 2R are connected in series in the circuit. The (c) P P0 (d) P P0
V0 V
thermal energy developed in R and 2R are in the ratio
(a) 1 : 2 (b) 2 : 1 11. The three resistances of equal value are arranged in the different
(c) 1 : 4 (d) 4 : 1 combinations shown below. Arrange them in increasing order of
3. Two resistors R and 2R are connected in parallel in the circuit. The power dissipation
thermal energy developed in R and 2R are in the ratio (I)
(1) 1 : 2 (b) 2 : 1
(c) 1 : 4 (d) 4 : 1
4. Forty electric bulbs are connected in series across a 220 V supply.
After one bulb is fused, the remaining 39 are connected again in
series across the same supply. The illumination will be (II)
(a) more with 40 bulbs than with 39
(b) more with 39 bulbs than with 40
(c) equal in both the cases
(d) in the ratio of 492 : 392
5. A heater coil is cut into two parts of equal length and one of them
is used in the heater. The ratio of the heat produced by this half
coil to that by the original coil is
(III)
(a) 2 : 1 (b) 1 : 2
(c) 1 : 4 (d) 4 : 1
6. What is immaterial for an electric fuse wire?
(a) Its specific resistance (b) Its radius
(c) Its length (d) Current flowing through it
7. An electric bulb is rated 220 volt and 100 watt. Power consumed
by it when operated on 110 volt is
(a) 50 watt (b) 75 watt (IV)
(c) 90 watt (d) 25 watt
8. Two heater wires of equal length are first connected in series and
then in parallel. The ratio of heat produced in the two cases is
(a) 2 : 1 (b) 1 : 2 (a) III < II < IV < I (b) II < III < IV < I
(c) 4 : 1 (d) 1 : 4 (c) I < IV < III < II (d) I < III < II < IV

Answer Key 1 (a) 2 (a) 3 (b) 4 (b) 5 (a) 6 (c)


Sol. from page 333
7 (d) 8 (d) 9 (d) 10 (b ) 11 (a)
THERMAL AND CHEMICAL EFFECT OF CURRENT 323
12. A 100 W bulb B1, and two 60 W bulbs B2 and B3, are connected to 18. A bulb rated at (100W – 200 V) is used on a 100 V line. The current
a 250 V source, as shown in the figure. Now W1, W2 and W3 are the in the bulb is
output powers of the bulbs B1, B2 and B3, respectively. Then
1
(a) amp (b) 4 amp
4

1
(c) amp (d) 2 amp
2
19. Two electrolytic cells containing CuSO4 and AgNO3 respectively
are connected in series and a current is passed through them until
1 mg of copper is deposited in the first cell. The amount of silver
deposited in the second cell during this time is approximately
[Atomic weight of copper and silver are respectively 63.57 and
107.88]
(a) W1 W2 W3 (b) W1 W2 W3
(a) 1.7 mg (b) 3.4 mg
(b) W1 W2 W3 (d) W1 W2 W3 (c) 5.1 mg (d) 6.8 mg
13. A 1000 W heating unit is designed to operate on a 120 V line. The 20. How much current should be passed through acidified water for
line voltage drops to 110 V. The percentage of heat output drops 100s to liberate 0.224 litre of H2.
by : (a) 22.4 A (b) 19.3 A
(a) 9% (b) 27 % (c) 9.65 A (d) 1 A
(c) 16 % (d) 30 % 21. One junction of certain thermoelectric couple is at a fixed
temperature Tr and the other junction is at temperature T. The
14. Three electric bulbs of 200 W, 200 W and 400 W are shown in thermo electromotive force for this is expressed by E = K(T – Tr)
figure. The resultant power of the combination is
1 1
T0 T Tr . At temperature T T0 , the thermoelctric
200 W 400 W 2 2
power is
1
(a) KT0 (b) KT 0
2
200 W 1 1 2
(c) KT 2 (d) K T0 Tr
2 0 2
250 V
22. For ensuring dissipation of same energy in all three resistors
(a) 800 W (b) 400 W (R1, R2, R3) connected as shown in figure, their values must be
related as
(c) 200 W (d) 600 W
15. An electric bulb rated 220 V, 100 W is connected in series with R1
another bulb rated 220 V, 60 W. If the voltage across the combination
is 220 V, the power consumed by the 100 W bulb will about
(a) 25 W (b) 14 W R2 R3
(c) 60 W (d) 100 W Vin
16. (1) The product of a volt and a coulomb is a joule.
(2) The product of a volt and an ampere is a joule/second. (a) R1 = R2 = R3
(3) The product of volt and watt is horse power. (b) R2 = R3 and R1 = 4R2
(4) Watt-hour can be measured in terms of electron volt.
1
State if (c) R2 = R3 and R1 = R2
4
(a) All four are correct (b) (1), (2) and (4) are correct (d) R1 = R2 + R3
(c) (1) and (3) are correct (d) (3) and (4) are correct 23. Each of three resistors having a resistance R can dissipate maxi-
17. Two bulbs are working in parallel order. Bulb A is brighter than mum power P. What is maximum power the circuit comprising of
bulb B. If RA and RB are their resistance respectively then three resistors can dissipate?
(a) RA > RB (b) RA < RB (a) 3P (b) 2 P
(c) RA = RB (d) None of these (c) 1.5 P (d) 2.5 P

Answer Key 12 (d) 13 (c) 14 (c) 15 (b) 16 (b) 17 (b)


Sol. from page 333
18 (a) 19 (b) 20 (b) 21 (a ) 22 (c) 23 (a)
324 ELECTRICITY & MAGNETISM
24. Consider the following two statements. 29. In Seebeck series Sb appears before Bi. In a Sb-Bi thermocouple
1. Free-electron density is different in different metals. current flows from
2. Free-electron density in a metal depends on temperature. (a) Sb to Bi at the hot junction
Seebeck effect is caused (b) Sb to Bi at the cold junction
(a) due to both statements 1 and 2 (c) Bi to Sb at the cold junction
(b) due to 1 but not due to 2 (d) none of the above
(c) due to 2 but not due to 1 30. Two different metals are joined end to end. One end is kept at
(d) neither due to 1 nor due to 2. constant temperature and the other end is heated to a very high
temperature. The graph depicting the thermo e.m.f. is
25. For a copper-iron thermocouple the values of the various
temperatures are given below : T0 = 0°C, TN = 275°C and
Ti = 550°C. If T0 is changed to 10°C, the new value of TN and Ti
will be respectively
(a) 275°C and 560°C (b) 275°C and 540°C
(c) 285°C and 540°C (d) 285°C and 560°C (a) (b)
26. The emf of a thermocouple, one junction of which is at 0°C is
given by E = at + bt2. The Peltier coefficient is given by (T is in
kelvin) :
(a) T(2 at + b) (b) T(2a + bt)
(c) T(a + bt) (d) T(a + 2bt)
27. In an electrolysis experiment, a current i passes through two
different cells in series, one containing a solution of CuSO4 and the (c) (d)
other a solution of AgNO3. The rate of increase of the weight of
the cathodes in the two cells will be
(a) in the ratio of the densities of Cu and Ag
31. The negative Zn pole of a Daniel cell, sending a constant current
(b) in the ratio of the atomic weight of Cu and Ag
through a circuit, decreases in mass by 0.13 g in 30 minutes. If the
(c) in the ratio of half the atomic weight of Cu to the atomic electrochemical equivalent of Zn and Cu are 32.5 and 31.5
weight of Ag respectively, the increase in the mass of the positive Cu pole in
(d) in the ratio of half the atomic weight of Cu to half the atomic this time is
weight of Ag (a) 0.242 g (b) 0.190 g
28. The electrochemical equivalent of a metal is 3.3×10–7 kg / coulomb. (c) 0.141 g (d) 0.126 g
The mass of the metal liberated at the cathode when a 3 A current
is passed for 2 second will be 32. Thomson coefficient of a conductor is 10 V/K. The two ends of it
are kept at 50°C and 60°C respectively. Amount of heat absorbed
(a) 19.8×10–7 kg (b) 9.39×10–7 kg
by the conductor when a charge of 10 C flows through it is
(c) 6.6×10–7 kg (d) 1.1×10–7 kg
(a) 1000 J (b) 100 J
(c) 100 mJ (d) 1 mJ

Answer Key 24 (a) 25 (b) 26 (d) 27 (c) 28 (a)


Sol. from page 333 29 (b) 30 (d) 31 (d) 32 (d)
THERMAL AND CHEMICAL EFFECT OF CURRENT 325

Level -2
Only one option correct 4. A constant current i is passed through a resistor. Taking the
1. Figure shows three resistor configuration R1, R2 and R3 connected temperature coefficient of resistance into account, indicate which
to 3V battery. If the power dissipated by the configuration R1, R2 of the plots shown in figure best represents the rate of production
of thermal energy in the resistor
and R3 is P1, P2 and P3 respectively, then

(a) a (b) b
(c) c (d) d
5. The charge flowing through a resistor R varies as Q (t) = t – t2.
The total heat produced is R is :
3 3
R R
(a) (b)
2
3 3
R R
(c) (d)
3 6
6. Consider four circuits shown in the figure below. In which circuit
power dissipated is greatest (neglect the internal resistance of the
power supply)

(a) P1 > P2 > P3 (b) P1 > P3 > P2


(a) (b)
(c) P2 > P1 > P3 (d) P3 > P2 > P1
2. Find the power of the circuit

(c) (d)

7. A torch bulb rated as 4.5 W, 1.5 V is connected as shown in the


figure. The e.m.f. of the cell needed to make the bulb glow at full
intensity is
(a) 1.5 W (b) 2 W
(c) 1W (d) None of these
3. Water of volume 2 litre in a container is heated with a coil of 1 kW
at 27 °C. The lid of the container is open and energy dissipates at
rate of 160 J/s. In how much time temperature will rise from 27°C
to 77°C [Given specific heat of water is 4.2 kJ/kg]
(a) 8 min 20 s (b) 6 min 2 s
(c) 7 min (d) 14 min
(a) 4.5 V (b) 1.5 V
(c) 2.67 V (d) 13.5 V

Answer Key 1 (c) 2 (c) 3 (a) 4 (d)


Sol. from page 334 5 (d) 6 (a) 7 (d)
326 ELECTRICITY & MAGNETISM
8. A battery of internal resistance 4 is connected to the network 11. Two voltameters, one of copper and another of silver, are joined in
parallel. When a total charge q flows through the voltameters,
of resistances as shown. In order to give the maximum power to
equal amount of metals are deposited. If the electrochemical
the network, the value of R (in ) should be equivalents of copper and silver are z1 and z2 respectively the
charge which flows through the silver voltameter is

z1 z2
(a) q (b) q z
z2 1

q q
(c) (d) z
z1 1 2
1
z2 z1
(a) 4/9 (b) 8 / 9
(c) 2 (d) 18 12. In the circuit shown in figure, the heat produced in 5 ohm resistance
is 10 cal/ s. The heat produced in 4 resistance is
9. Consider a wire of non-uniform cross-section. If the area of cross-
section at point A is double of the area of cross-section at point B.
What is ratio of heat energy dissipated in a unit volume at points
A and B?

A B
(a) 1 cal/ s (b) 2 cal/ s
(c) 3 cal/ s (d) 4 cal/ s
13. Silver and copper voltameter are connected in parallel with a battery
of e.m.f. 12 V. In 30 minutes, 1 gm of silver and 1.8 gm of copper
are liberated. The power supplied by the battery is
1 2
(a) (b) (a) 24.13 J/s (b) 2.413 J/s
2 1
(b) 0.2413 J/s (d) 2413 J/s
1 4 (ZCu = 6.6 ×10–4 gm/C and ZAg = 11.2 × 10–4gm/C)
(c) (d)
4 1 14. The thermo e.m.f. of a thermo-couple is 25 V/ °C at room
10. In the circuit shown in figure, the internal resistances of the sources temperature. A galvanometer of 40 ohm resistance, capable of
are negligible. What is maximum power that can be generated in detecting current as low as 10 –5 A, is connected with the
resistance R? thermocouple. The smallest temperature difference that can be
detected by this system is
R 10 V
(a) 20° C (b) 16° C
(c) 12º C (d) 8° C
3 6V 15. The thermo e.m.f. of a thermocouple varies with the temperature
of the hot junction as E = a + b 2 in volt where the ratio a/b is
700°C. If the cold junction is kept at 0°C, then the neutral
6 temperature is
(a) 700° C
(a) 4W (b) 4.5 W (b) 350° C
(c) 2W (d) 2.5 W (c) 1400° C
(d) no neutral temperature is possible for this thermocouple

Answer Key 8 (c) 9 (c) 10 (b) 11 (d)


Sol. from page 334 12 (b) 13 (a) 14 (b) 15 (d)
THERMAL AND CHEMICAL EFFECT OF CURRENT 327

MCQ Type 2 E xercise 4.2


Multiple correct options 4. A cell of emf 6 V and internal resistance 1 is connected across
1. For the circuit shown in the figure resistor R = 2 . Then :

2
2k R1
I
i

24 V 6k R2 RL 1.5 k

6V, 1
(a) the current I through the battery is 7.5 mA (a) Power dissipate in the source is 4 W.
(b) the potential difference across RL is 18 V (b) Chemical energy dissipated at the rate of 12 W.
(c) ratio of powers dissipated in R1 and R2 is 3 (c) Power output of the source is 8 W.
(d) if R1 and R2 are interchanged, magnitude of the power (d) Power output of the source is 12 W.
dissipated in RL will decrease by a factor of 9 5. The constants a and b for the pair silver-lead are 2.50 V/C and
2. Two bulbs : 10 W and 200 W are connected in parallel. Then : 0.012 V/C2 respectively. For a silver-lead thermocouple with
(a) The voltage through each bulb is same colder junction at 0°C,
(b) The current in each bulb is same (a) there will be no neutral temperature
(c) The current in 100 W bulb is smaller than current in 200 W (b) there will be no inversion temperature
bulbs (c) there will not be any thermo-emf even if the junctions are
(d) The voltage across 200 W is greater kept at different temperatures
3. Three identical bulbs are connected as shown in figure. (d) there will be no current in the thermocouple even if the
functions are kept at different temperatures.

B2

P Q
B1
B3

Each bulb dissipate a maximum power P. Then :


(a) The total power dissipate of the circuit is 3 P.
3P
(b) The total power dissipate of the circuit is .
2
(c) The current in each resistor is same
(d) The current in bulbs B1, B2 and B3 are in ratio 2 : 3 : 1.

Answer Key 1 (a, d) 2 (a, c) 3 (b, d) 4 (a, b,c)


Sol. from page 335 5 (a, b)
328 ELECTRICITY & MAGNETISM

Statement Questions E xercise 4.3


Read the two statements carefully to mark the correct option out of the options given below:
(a) If both the statements are true and the statement - 2 is the correct explanation of statement - 1.
(b) If both the statements are true but statement - 2 is not the correct explanation of the statement - 1.
(c) If statement - 1 true but statement - 2 is false.
(d) If statement - 1 is false but statement - 2 is true.

1. Statement 1 9. Statement 1
If a constant potential difference is applied across a bulb, the The possibility of an electric bulb fusing is higher at the time of
current slightly decreases as time passes and then becomes switching ON and OFF
constant. Statement 2
Statement 2
Inductive effects produce a surge at the time of switch ON and
The resistance of the metal increases with temperature. OFF
2. Statement 1
10. Statement 1
The 200 W bulb glows more brightly than 100 W bulb.
In the given circuit if lamp B or C fuses then light emitted by lamp
Statement 2
A decreases.
The resistance of 200 W bulb is less than the 100 W bulb.
3. Statement 1
Fuse wire must have low resistance and low melting point.
Statement 2
Fuse is used for small current only.
4. Statement 1
A domestic electrical appliance, working on a three pin will continue
working even if the top pin is removed.
Statement 2
The third pin is used only as a safety device. Statement 2
5. Statement 1 Voltage on A decreases.
A laser beam 0.2 W power can drill holes through a metal sheet, 11. Statement - 1
whereas 1000 W torchlight cannot. In an electrolyte, the positive ions move from left to right and the
Statement 2 negative ions from right to left, so there is no net current flows in
The frequency of laser light is much higher than that of torch light an electrolyte.
6. Statement 1 Statement - 2
Electric appliances with metallic body; e.g. heaters, presses etc. The current flows from cathode to anode in an electrolyte.
have three pin connections, whereas an electric bulb has a two pin
12. Statement - 1
connection.
The thermal power generated in a resistor is given by P = i2R and
Statement 2
so it is proportional to i2.
Three pin connections reduce heating of connecting cables.
7. Statement 1 Statement - 2
Neutral temperature of any thermocouple is a basic characteristic. The thermal power generated in a resistor is given by P=V i and so
it is proportion to i.
Statement 2
13. Statement - 1
All the thermocouples have a neutral temperature.
8. Statement 1 In each electrical circuit, the thermal power generated is equal to
the rate of work done by battery.
The neutral temperature does not depend on the temperature of
the cold junction. Statement - 2
Statement 2 The rate of work done by the battery is equal to the power delivered
The inversion temperature does not depend on the temperature of to the circuit.
the cold junction.

Answer Key 1 (a) 2 (a) 3 (c) 4 (a) 5 (c) 6 (c) 7 (c)


Sol. from page 336 8 (c) 9 (a) 10 (a) 11 (d) 12 (c) 13 (d)
THERMAL AND CHEMICAL EFFECT OF CURRENT 329

Passage & Matrix E xercise 4. 4

Passage for Q. 1 to Q. 3 1 2
(a) C (b) C 2
The internal resistance of a dry cell increases gradually with age, even 2
though the cell is not used. The emf, however, remains fairly constant at (c) 2C 2 (d) none of them
about 1.5 V. Dry cells may be tested for age at the time of purchase by 6. Maximum energy stored in the capacitor
connecting an ammeter directly across the terminals of the cell and reading 1 2
(a) C (b) C 2
the current. The resistance of the ammeter is so small that the cell is 2
practically short circuited. (c) 2C 2 (d) Infinite.

Passage for Q. 7 to Q. 9
1. The short circuit current of the cell is about 30 A. The internal Two batteries (2 , r) and ( , 2r) are connected as shown in diagram the
resistance of the cell approximately is : resistance R (external load) is variable, which can be adjusted using rheostat,
(a) 0.01 (b) 0.02 carefully answer the questions for this given situation.

(c) 0.05 (d) 0.1


2. The power generated by the internal resistance is :
2
(a) 45 W (b) 60 W
(c) 90 W (d) 100 W r 2r
R
3. The power generated in the circuit is :
i0
(a) 45 W (b) 60 W
(c) greater than 45 W (d) less than 45 W 7. Value of R for which power across the load (R) is maximum is
3r 2r
(a) (b)
Passage for Q. 4 to Q. 6 2 3
t / RC , where (c) r (d) none of these
The current in a charging capacitor(C) is given by i i0 e 8. Maximum power dissipation in the circuit is

2 2 3 2
i0 . For instantaneous potential V of the capacitor, power is given (a) (b)
R 3r 2r
by P = Vi.
2 3
4. Total energy supplied by the battery is : (c) (d) none of these
6r
1 2 9. Maximum value of i0 is
(a) C (b) C 2
2
(c) 2C 2 (d) zero (a) (b)
2r r
5. The instantaneous power dissipated in the resistor is i2R. The
5
total energy dissipated in the resistor is (c) (d)
2r 3r

Answer Key 1 (c) 2 (a) 3 (c) 4 (b) 5 (a)


Sol. from page 336 6 (a) 7 (b) 8 (d) 9 (c)
330 ELECTRICITY & MAGNETISM

10. Match the Column I with the Column II from the combination shown. In the left side (Column I) there are four different conditions
and in the right side (Column II), there are ratios of heat produced in each resistance of each condition :
Column I Column II
A. Two wires of same resistance are connected in series (p) 1:2
and same current is passed through them
B. Two wires of resistance R and 2R ohm are connected in (q) 4:1
series and same potential difference is applied across them
C. Two wires of same resistance are connected in parallel (r) 1:1
and same current is flowing through them
D. Two wires of resistances in the ratio 1 : 2 are connected (s) 2:1
in parallel and current i is across them

11. A cell of e.m.f and internal resistance r is connected across a variable load resistance R. Match the statements given in column I to
conductions gives in column II.
Column I Column II
A. Thermal power generated in the load (p) R=r
E2
resistance is less than
4r
E
B. Potential difference across load is more than (q) R=0
2
E2 R
C. Thermal power generated in the cell is (r) r
r 4
3R
D. Work done by battery is positive (s) r
2
R
(t) r
2

12. n cell are connected in a closed loop. Emf of the cells are 1V, 2V, 3V ....nV, here n is even. Internal resistance of the cell are 0.5 , 1.0 , 1.5 ,
n
2 ..... .Based on above facts, match column I with column II.
2

B r1

i ri r 2

Column I Column II
A. Zero (p) Current through each cell
B. Equal (q) Terminal voltage of each cell
C. Unequal (r) Potential difference between any two points
D. Non-zero (s) A and B on the circuit
(t) Current through each cell if alternate cells are
connected with reverse polarity

Answer Key 10 A-r ; B-s ; C-r ; D- s 11 A-r, s, t ; B-r, t ; C-q ; D-p, q, r, s


Sol. from page 336 12 A-q, r ; B-q, r, t ; C-s ; D-p, s, t
THERMAL AND CHEMICAL EFFECT OF CURRENT 331

Subjective Integer Type Exercise 4.5


Solution from page 337

1. An immersion heater rated 1000 W, 220 V is used to heat 0.01 m3 5. A charged capacitor of 5 × 10–2 F capacity is discharged through a
of water. Assuming that the power is supplied at 220 V and 60 % resistor R of 20 and a copper voltmeter of internal resistance 30
of the power supplied is used to heat the water, how long will it connected in series. If 4.62 × 10 –6 kg copper is deposited,
take to increase the temperature of the water from 150C to 40°C ? calculate the heat generated in the resistor R. (Electrochemical
Ans. 29 min. equivalent of copper is 3.3 × 10–7 kg/C).
2. Four resistances carrying a current as shown in the diagram are
immersed in a box containing ice at 0°C. How much ice must R=20
be put in the box every 10 minute to keep the average quantity of
ice in the box constant? Latent heat of ice = 80 cal/g.

10 5
10A
R =30
C
5 10 q

Ans. 1190 g. Ans. 784 J.


3. In circuit shown in figure the heat produced in 5 resistor due to 6. Find the time required to liberate 1.0 litre of hydrogen at STP in an
current flowing through it is 10 calorie per second.The heat gener- electrolytic cell by a current of 5.0 A.
ated in 4 resistor is : Ans. 29 min.
7. The potential difference across the terminals of a battery of emf
4 6 12 V and internal resistance 2 drops to 10 V when it is con-
nected to a silver voltmeter. Find the silver deposited at the cath-
ode in half an hour. Atomic weight of silver is 107.9 g/mole.
Ans. 2g.
5

Ans. 2 cal/s.
4. A heater is designed to operate with a power of 1000 W in 100 V
line. It is connected to two resistances of 10 and R as shown in
figure. If the heater is now given a power of 62.5 W, calculate the
value of resistance R.

Heater
10 1000W

100V

Ans. R = 5 .
332 ELECTRICITY & MAGNETISM

Subjective E xercise 4.6


Solution from page 338
1. The 2.0 resistor shown in figure is dipped into a calorimeter 5. A piece of metal weighing 20 g is to be electroplated with 5% of its
containing water. The heat capacity of the calorimeter together weight in gold. If the strength of the available current is 2 A, how
with water is 2000 J/K. long would it take to deposit the required amount of gold E C E of
(a) If the circuit is active for 15 minutes, What would be the rise H = 0.1044 × 10–4, atomic weight of gold = 197.1, atomic weight
in the temperature of the water ? of hydrogen = 1.008.
(b) Suppose the 6.0 resistor gets burnt. What would be the Ans. 4 min.27.9 s.
rise in the temperature of the water in the next 15 minutes ? 6. Figure shows an electrolyte of AgCl through which a current is
passed. It is observed that 2.68 g of silver is deposited in 10
minutes on the cathode. Find the heat developed in the 20
6V 1
resistor during this period. Atomic weight of silver is 107.9 g/mol.
+ -
20
6 + -

Ans. (a) 2.9°C (b) 3.6°C.


2. A battery of internal resistance 4 is connected to the network of
resistances as shown in figure. What must be the value of R so
that maximum power is delivered to the network ? What is the Ans. 190 kJ.
maximum power? 7. A copper wire having cross–sectional area 0.5 mm2 and a length of
0.1 m is initially at 25°C and is thermally insulated from the
A R B R surrounding. If a current of 10 A is set up in this wire.
(i) Find the time in which the wire will start melting. The change
R of resistance with the temperature of the wire may be
R 6R
neglected.
R 4R
C (ii) What will be the time taken if length of the wire is doubled ?
D
Given for copper wire, density = 9 × 103 kg/m3, specific
+ – heat = 9 × 10–2 kcal/kg–°C, melting point = 1075°C, specific
r=4 resistance 1.6 × 10–8 –m.
Ans. (i) 558 s, (ii) 558 s.
Ans. 2 , 2/16. 8. A plate of area 10 cm2 is to be electroplated with copper (density
3. An electric kettle has two coils. When one coil is switched on, it 9000 kg/m3) to a thickness of 10 micrometre on both sides, using
takes 15 minute to boil water and when the second coil is switched a cell of 12 V. Calculate the energy spent by the cell in the process
on, it takes 30 minute. How long will it take to boil water and of deposition. If this energy is used to heat 100 g of water, calculate
when both the coils are used in (i) series and (ii) parallel ? the rise in the temperature of the water. ECE of copper = 3 × 10–7
Ans. (i) 45 min. (ii) 10 min. kg/C and specific heat capacity of water = 4200 J/kg–K.
4. The temperatures of the junctions of a bismuth– silver Ans. 7.2 kJ, 17 K.
thermocouple are maintained at 0°C and 0.001°C. Find the thermo– 9. Find the amount of silver liberated at cathode of 0.500 A of current
emf (Seebeck emf) developed. For bismuth–silver, a = – 46 × is passed through AgNO3 electrolyte for 1 hour. Atomic weight of
10 –6 V/°C and b = – 0.48 × 10 –6 V/°C2 . silver is 107.9 g/mol.
Ans. –4.6 × 10–8 V. Ans. 2.01 g.
THERMAL AND CHEMICAL EFFECT OF CURRENT 333

S olutions Exercise4.1 Level -1


1. (a) Thermal energy, U = i2Rt, and so U i2, it represents a
parabola between U and i. 1 1 1
Now , and so P = 200 W
2. (a) In series, current in both the resistors is same and so P 400 400

U1 i 2 Rt 1 V2 2202 R1 R2
2 . 15. (b) R1 = 484 i
U2 i (2 R)t 2 P1 100
3. (b) In parallel, potential difference across both the resistors is 220V
same and so V2 2202
and R2 = 806.7
P2 60
U1 (V 2 / R )t 2
U2 (V 2 / 2 R)t 1 220
Current, i = 0.17 A
4. (b) When one bulb is fused, the resistance of rest 39 bulb be- 484 806.7
comes smaller and so current in the bulbs will increase, which Now, P1 = i2R1 = 0.172 × 484 = 14 W.
result increase in illumination. 16. (b)
17. (b)
R
5. (a) The resistance of half coil, R ' = .
2 V2 2002
18. (a) Resistance of the bulb, R = = 400
P 100
H' V 2 / R' R
So, ratio of heat produced, = 2 2.
H V /R R' V 100 1
Now, current i A
6. (c) R 400 4

V2 2202 m1 W
19. (b) = 1
7. (d) Resistance of the bulb, R = . m2 W2
P 100
W2
V 2supply m2 = m1
Power consumed with 110 V, P = W1
R

110 2
= 25 W . 107.88
220 2 /100 =1
63.57
H1 V 2 / 2R 1 2
8. (d) =
H2 V 2 /( R / 2) 4 = 3.4 mg
9. (d) For maximum power, R = r = 0.5 . 22.4
20. (b) To liberate litre of H2, the charge needed = 96500C
P 2
V 2 2
10. (b) = to liberate 0.224 litre of H2, the charge needed
P02 V02
11. (a) If each resistance is of value R, then 96500 2
= C
100
2R R 2R R 3R
R1 = 3R, R2 , R3 R , R4 R . = 1930C
2R R 3 3 2 2
As R1 > R4 > R2 > R3, so power P1 > P4 > P2 > P3. q 1930
Current needed , i = = 1930 A
12. (d) Resistance, R1 < R2, also R2 = R3. t 100
So, in series, W1< W2. dE
Current in B3 will be greater than current in bulb B2 and so 21. (a) We know that S .
dT
W3 >W2.
(T Tr )
120 2 1102 Given E k (T Tr ) T0 .
13. (c) P1 and P2 2
R R
T0
P1 P2 120 2 110 2 On differentiating E w.r.t. T and putting, T , we get
100 16% 2
P1 120 2
kT0
14. (c) The total power of bulbs in parallel = 400 W S .
2
334 ELECTRICITY & MAGNETISM
22. (c) For same energy dissipation in all the resistors, R2 = R3. If i
mcu W ( Acu / 2)
is the current in R1, then it will divide equally in R2 and R3, 27. (c) = cu
so m Ag W Ag AAg

2 28. (a) m = Zit = 3.3 × 10–7× 3 × 2 = 19.8 × 10–7 kg.


i 29. (b)
i 2 R1 = R2
2 30. (d)
R2 m1 Z
R1 = . 31. (d) = 1
4 m2 Z2
23. (a) For maximum power, all the three resistors must be placed in
Z2 31.5
parallel and so, m2 m1 0.13
Ptotal = 3 × P = 3P. Z1 32.5
24. (a) = 0.126 g
T0 Ti 32. (d) H= ( Q) t
25. (b) We know that, TN ,
2 = (10 10 6 ) 10 (60 50)
Ti = 2TN –T0 = 2 × 275 –10 = 540° C
= 10–3 J
dE d
26. (d) Peltier coefficient, T T (at bt 2 )
dt dt
= T(a + 2bt).

Solutions Exercise4.1 Level -2


1 /2
1. (c) R1 = 1 , R2 = and R3 = 2
2 = ( 2 t ) 2 Rdt
0
V2 V2 V2 V2
Now, P1 , P2 , P3
3
R1 1 (1/ 2) 2 R
= .
P2 P1 P3 . 6
6. (a) Resistance of the circuits are :
2 1
2. (c) The only current in 2 resistors, i A R 3R 2R
4 2 R1 , R2 2 R , R3 , and R4
2 2 3
2
1
Now, power P = i2 R = 4 1W V2 V2 2V 2
2 Pgreatest .
Rleast R/2 R
3. (a) Energy required to raise the temperature of water from 27°
to 77°C. P 4.5
Q = mC T + loss 7. (d) Allowable current in the bulb, i1 3A
V 1.5
or 1000t = 2× 4200 × (77 – 27) + 160 × t
or t = 500 s 1.5
Current in 1 resistor, i2 1.5 A
= 8 min 20 s 1
dU Total circuit current i i1 i2 4.5 A
4. (d) Power, P = i2R
dt Now , = V + ir
= 1.5 + 4.5 × 2.67
= i 2[ R0 (1 t )] . = 13.5 V
It represents a straight line in P and t with positive slope. 8. (c) The effective circuit is shown in figure.
5. (d) Current through a resistor,
R 2R
dQ d
i ( t t2)
dt dt
= 2 t.
2R 4R
The time upto current flows,
i= 2 t 0

or t= . The equivalent resistance across the battery.


2
3 6
t Req R
2
i Rdt 3 6
Total heat produced, H =
0 = 2R.
THERMAL AND CHEMICAL EFFECT OF CURRENT 335
For maximum power, 11. (d) Given mcu = msi
Req r or Z1q1 = Z 2q2
or 2R 4 Also, q1 q2 q
R=2 qZ1 q
q2 = =
Z1 Z 2 Z2
9. (c) 1
i i Z1
4R
R V2
12. (b) As P , V PR 10 5 volt = 50 volt
R
H1 i 2 R1 R 1
= 4R = 4 50
H2 i 2 R2 Now, current in 4 resistor, i A.
10
Power generated in 4 2
resistor, = i R
i1 R 10 V
50
= 4 2 cal/s
100
i2 3 6V
m
13. (a) m = Zq, q
10. (b) Z
6 1
Now, q1 = 4
C 0.89 103 C
(i1 + i2) 11.2 10
1.8
i1R – 10 + 6 – 3i2 = 0 q2 = 4
2.73 103 C
or i1R – 3i2 = 4 ...(i) 6.6 10
and 3i2 – 6 + 6 (i1 + i2) = 0 Total charge,
or 6i1 + 9i2 = 6 ...(ii) q q1 q2 (0.89 2.73) 103 C 3.62 103 C.
Solving above equations, we get
Power supplied by battery, P (Vq) / t
18 6
i1 =
6 3R 2 R 12 3.62 103
= J/s
30 60
Power in R,
= 24.13 J/s
P = i12 R 14. (b) The p.d. across galvanometer,

2 V iR 10 5 40 40 10 5 volt
6 The temperature difference that can be detected,
= R
2 R 5
40 10
T 6
16 C
dP 25 10
P to be maximum, =0
dR 2
15. (d) Given, E a b .

d 62 dE
For neutral temperature, 0,
or dR 2
R 0 d
2 R
d 2
R=2 or (a b ) 0
d
6
2 or a 2b 0,
Now Pmax 2
a 700
2 2 350 C (not possible)
or
= 4.5 2b 2

S olutions Exercise4.2
1. (a, d) Total resistance of the circuit Power, P1, = i12 × 1.5 = (6 × 10–3)2 × 1.5 × 103
6 1.5 = 54 × 10–3 J
R= 2 4.2 k After interchanged, total resistance
6 1.5
2 1.5
24 R= 6 6.86 k
Now I 3
7.5 10 3 A 2 1.5
4.2 10
24
3 Total current, I = 3.5 10 3 A
7.5 10 6 3 6.86 103
Current in RL, i1 6 10 A
(6 1.5)
336 ELECTRICITY & MAGNETISM
Power dissipate in bulb B2 or B3,
3.5 10 3 2
Now current in RL = 2 10 3 A 2
(2 1.5) i P
P R .
2 4
Power, P1 ' = (2 10 3 )2 1.5 103
= 6 × 10–3J P P 3P
Thus total power P .
4 4 2
Clearly P1 : P1 ' 9 .
2. (a,c) Resistance of 100 W bulb is greater and in parallel, both the 6
4. (a,b,c) Current, i 2 A.
bulbs have same potential, and so current in 100 W bulb will 2 1
be smaller. Electric power produced in the source = i = 6 × 2 = 12 W.
3. (b,d) If i in the current in bulb B1, then it will divide equally in Power dissipate in the source = i 2 r = 2 2 × 1 = 4 W.
bulbs B2 and B3. Also
5. (a, b) For neutral and inversion temperature, either of a or b must
P = i2R. be negative.

Solutions Exercise4.3
1. (a) With increase in temperature of filament, its resistance will 8. (c) Netural temperature is the characteristics of metals used in
increase and so current will decrease. the thermocouple. As ti = 2tn – tc, so inversion temperature
2. (a) Power consumed by 200 W bulb will be twice that of 100 W depends on cold junction temperature.
9. (a) The change in current at the time of switch ON or OFF is
1
bulb. Also R . greater.
p 10. (a) If either of bulbs B or C fuses, the effective resistance of
3. (c) Fuse wire has low resistance and low melting point, so it will circuit increases and so current and p.d. across bulb A will
melt easily due to excessive heat. decreases.
4. (a) Statement -2 is the correct explanation of statement-1. 11. (d) Both types of ions constitute the current.
5. (c) The energy of laser light is focused to small area but that of
12. (c) In P = Vi, V also depends on i , V = iR, and so P i 2 .
bulb, it spread on larger area.
6. (c) In case of short circuit third pin passes the extra current. 13. (d) Some power is dissipated by the battery itself due to internal
7. (c) Netural temperature is the characteristics of metals used in resistance.
the thermocouple. Only few thermocouple has neutral
temperature.

Solutions Exercise4.4
Passage Q.1 to Q.3 7. (b) For maximum power,
1.5 R = rnet
1. (c) r= 0.05
i 30 2r
2. (a) Power generated by internal resistance, or R= .
3
P i 2r 302 0.05 45 W net 5 /3 5 /3 5
8. (d) i
3. (c) The power generated in the circuit is the sum of power Rtotal R rnet 2 2r / 3 4r
generated in external and internal circuit, so it must be greater
2
than 45 W. 5 4r 25 2
Pmax i 2 Rtotal
Passage of Q.4 to Q.6 4r 3 12r
4. (b) Total energy supplied by the battery = q (C ) C 2 . 9. (c) Maximum value of i0 ;
5. (a) Half the work done by battery will store in electrical energy net 5 /3 5
i0 [R = 0]
1 Rmin 2r / 3 2r
2
of the capacitor, U C and rest half converts into the 10. A-r : When same current is passed through them,
2
heat energy. H1 i 2 R and H 2 i 2 R , and so H1 / H 2 1:1 .
1 2
6. (a) The maximum energy stored in the capacitor, U C . H1 V2/R
2 B -s : 2.
Passage of Q.7 to Q.9 H2 V 2 / 2R
The equivalent circuit is shown in figure.
H1 i 2R
2 2r r 5 C-r : 1
net 5
R
H2 i 2R
2r r 3 2r
3
2r r 2r 3 i0 H1 i12 R1 (2i / 3)2 R
rnet D-q : 2
2.
2r r 3 H2 i2 R2 (2i / 3)2 2 R
THERMAL AND CHEMICAL EFFECT OF CURRENT 337
11. A-r, s, t : The maximum power across external circuit generated 12. A-q, r : Current in the circuit,
when R = r. For any other value of r, the power generated
1 2 ..... n 1 2 ..... n
will be less than maximum. i
B-r,t : For V > / 2, the value of r should be less than R. r1 r2 ...... rn 0.5 1 ... n / 2
2 = 2
C- q : If R = 0, then P . p.d. across any cell V = ir
r
= 1 – 2 ×0.5 = 0
D- p, q, r, s : W.d. by battery in each case is positive.
B- q, r, t : As all the cells are connected in series and so current in
each cell will be same in both the cases.
C- s :

S olutions Exercise-4.5
1. Mass of the water, m
= V 1000 0.01 62.5
= 2.5 A
= 10 kg. 10
The amount of heat required to raise the temperature of water,
2.5 10 25
H = mC T The current in resistor R, i2 =
R R.
= 10 4200 40 15
The total current, i = i1 i2
= 1.05 106 J.
25
If t be the required time, then = 2.5
R
0.60 Pt = H
Thus i RTotal = 100
H 1.05 106
or t = 0.60 P 0.6 1000 25 10 R
or 2.5 10 = 100
R 10 R
29 min. Ans.
2. The equivalent resistance of the circuit After solving, we get R = 5 . Ans.
10 5
5. By Faraday law, we have m = zq
R = 2 6.67 . 6
10 5 m 4.62 10
q = 14 C
The heat evolved in 10 minute, z 3.3 10 7

H = i 2 Rt 102 6.67 10 60 q2 142


The total heat generatedH = 2
1960 J .
= 4 105 J. 2C 2 5 10
= 95238 Cal. If H1 and H2 are the heat generated in 20 and 30 resistors,
The amount of ice required then
H 95238 H1 H 2 = 1960 ...(i)
m =
L 80
H1 2
= 1190 g Ans. i 20 2
Also H2 = ...(ii)
3. If V is the p.d. across 5 resistor, then i 2 30 3
After solving above equations, we get
V2 H 1 = 784 J. Ans.
P =
R 6. Charge required to liberate 22.4 litre of hydrogen
V = PR 10 5 50 = 2 96500 C
4 6 Charge required to liberate 1 litre of hydrogen
2 96500
q = C.
22.4
If t is the required time, then
5 q 2 96500 / 22.4
t =
i 5
The current in 4 resister,
= 29 min. Ans.
V 50 7. If i be the current in the circuit, then
i = .
4 6 10 V = ir
Thus heat generated in 4 resistor, or 10 = 12 i 2
2 i = 1A.
50 The charge flows in half hour,
H = i2 4 4 2 cal. Ans.
10 q = it 1 1800 1800 C.
V2 1002 The charge required to liberate 107.9 g of silver
4. The resistance of heater,R = 10 = 96500 C
P 1000
If i is the current in heater, then 107.9 1800
1800 C will liberate, =
P 96500
i1 = = 2 g. Ans.
R
338 ELECTRICITY & MAGNETISM

S olutions Exercise-4.6
1. The total resistance of the circuit 6. The charge needed to deposit 107.9 g (1 mol) of silver
2 6 5 = 96500 C
R = 1 Charge needed to deposit 2.68 g
2 6 2
The current drawn from the battery 96500 2.68
q = = 2397 C.
V 6 107.9
i = 2.4 A.
R 5/ 2 q 2397
The current in the resistor i = 4A
6 t 10 60
The current in 2 resistor, i ' = 2.4 1.8 A.
6 2 The heat produced, H = i 2 Rt
The heat produced through 2 resistor in 15 minute is,
= 42 20 10 60
H = i 2 Rt 1.82 2 15 60 = 192 kJ. Ans.
= 5832 J 7. Mass of the wire, m = A
(a) If T be the size in temperature of water, then
= 9 103 0.5 10 6
0.1
H 5832
T = 2.9 C. Ans. = 0.45 10 3
kg
C 2000
(b) If 6 resistor gets burnt, then current in 2 resistor,
Resistance of the wire,R =
6 A
i = 2A
2 1 1.6 10 8
0.1
The heat produced in it in 15 minute = 6
0.5 10
H = 22 2 15 60 = 7200 J.
= 0.32 10 2
7200
The rise in temperature T = 3.6 C. Ans. The amount of heat needed to melt the wire
2000
H = mC T
2. The given circuit is balanced wheat stone bridge type, so its resis-
tance = 0.45 10 3
9 10 2
4200 1075 25
3R 6 R = 178.6 J
Rt = 2 R.
3R 6 R (i) If t is the required time, then
For the maximum power, Rt = r i 2 Rt = H
2R = 4 or 10 2
0.32 10 t = 178.6
2
R = 2 . t = 558 s.
2 2 2 (ii) If mass of the wire is doubled, the heat required becomes zH
The maximum power, P = = . Ans. and resistance of wire becomes zR, so the time required will
4 Rt 4 4 16 be same.
3. (i) In series, time taken to boil water 8. The mass of copper needed for electroplating,
t = t1 t2 15 30 45 minute m = 2 At
(ii) In parallel, time taken to boil water
4 6
t1t2 15 30 = 2 9000 10 10 10 10
t = 10 minute. 5
t1 t2 15 30 = 18 10 kg
4. Given, t = Charge required to deposit copper
0.001 0 0.001 C
5
b 2 m 18 10
The Seebeck emf, e = at t q = 7
600 C
2 z 3 10
6
The energy spent by the cell
6 0.48 10
= 46 10 0.001 0.0012 H = Vq 12 600
2
= 7.2 kJ Ans.
= 8 Ans. If T be the rise in temperature, then
4.6 10 V.
5 H = mC T
5. Mass of gold required, m = 20 1g.
100 H 7.2 103
Z Au WAu or T = = 3
mC 100 10 4200
We know that ZH = WH = 17 K. Ans.
WAu 9. The charge flows in one hour
Z Au = z H = 197.1 0.1044 10 4
WH 1.008 q = i t 0.5 3600
= 1800 C.
= 20.41 10 4
96500 C of charge liberates the silver
We know that, m = Z Au i t = 107.9 g
3
1800 C of charge liberate
m 1 10
t = 7 107.9 1800
Z Au i 20.41 10 2 = = 2.01 g Ans.
96500
4 min. Ans.

Вам также может понравиться